You are on page 1of 204

Situation 1 Nurse Minette is an independent nurse practitioner following-up

referred clients in their respective homes. Here she handles a case of


POSTPARTUM MOTHER AND FAMILY focusing on HOME CARE.
1. Nurse Minette needs to schedule a first home visit to OB client Leah. When is a first homecare visit typically made?
a. Within 4 days after discharge
b. Within 24 hours after discharge
c. Within 1 hour after discharge
d. Within 1 week of discharge
2. Leah is developing constipation from being on bed rest. What measures would you suggest
she take to help prevent this?
a. Eat more frequent small meals instead of three large one daily
b. Walk for at least half an hour daily to stimulate peristalsis
c. Drink more milk, increased calcium intake prevents constipation
d. Drink eight full glasses of fluid such as water daily
3. If you were Minette, which of the following actions, would alert you that a new mother is
entering a postpartum at taking-hold phase?
a. She urges the baby to stay awake so that she can breastfeed him in her
b. She tells you she was in a lot of pain all during labor
c. She says that she has not selected a name for the baby as yet
d. She sleeps as if exhausted from the effort of labor
4. At 6-week postpartum visit what should this postpartum mothers fundic height be?
a. Inverted and palpable at the cervix
b. Six fingerbreadths below the umbilicus
c. No longer palpable on her abdomen
d. One centimeter above the symphysis pubis

5. This postpartum mother wants to lose the weight she gained in pregnancy, so she is reluctant
to increase her caloric intake for breast-feeding. By how much should a lactating mother
increase her caloric intake during the first 6 months after birth?
a. 350 kcal/day
b. 5CO kcal/day
c. 200 kcal/day
d. 1,000 kcal/day
Situation 2 As the CPES is applicable for all professional nurse, the professional growth and
development of Nurses with specialties shall be addressed by a Specialty Certification Council. The
following questions apply to these special groups of nurses.
6. Which of the following serves as the legal basis and statute authority for the Board of
nursing to promulgate measures to effect the creation of a Specialty Certification Council and
promulgate professional development programs for this group of nurse-professionals?
a. R.A. 7610
b. R.A. 223
c. R.A. 9173
d. R.A. 7164
7. By force of law, therefore, the PRC-Board of Nursing released Resolution No. 14 Series of the
entitled: Adoption of a Nursing Specialty Certification Program and Creation of Nursing
Specialty Certification Council. This rule-making power is called:
a. Quasi-Judicial Power
b. Regulatory Power
c. Quasi/Legislative Power
d. Executive/Promulgation Power
8. Under the PRC-Board of Nursing Resolution promulgating the adoption of a Nursing
Specialty-Certification Program and Council, which two (2) of the following serves as the
strongest for its enforcement?

(a) Advances made in science and technology have provided the


climate for specialization in almost all aspects of human endeavor
and

(b) As necessary consequence, there has emerged a new concept


known as globalization which seeks to remove barriers in trade,
industry and services imposed by the national laws of countries all
over the world; and

(c) Awareness of this development should impel the nursing


sector to prepare our people in the services sector to meet .the
above challenges; and

(d) Current trends of specialization in nursing practice recognized


by; the International Council of Nurses (ICN) of which the
Philippines is a member for the benefit of the Filipino in terms of
deepening and refining nursing practice and enhancing the quality
of nursing care.

a. b & c are strong justification


b. a & b are strong justification
c. a & c are strong justification
d. a & d are strong justification
9. Which of the following is NOT a correct statement as regards Specialty Certification?
a. The Board of Nursing intended to create the Nursing Specialty Certification Program as a means of
perpetuating the creation of an elite force of Filipino Nurse Professionals.
b. The Board of Nursing shall oversee the administration of the NSCP through the various Nursing
Specialty Boards which will eventually, be created.
c. The Board of Nursing at the time exercised their powers under R.A. 7164 in order to adopt the
creation of the Nursing Specialty Certification /council and Program.
d. The Board of Nursing consulted nursing leaders of national nursing associations and other
concerned nursing groups which later decided to ask a special group of nurses of the program for
nursing specialty certification

10. The NSCC was created for the purpose of implementing the Nursing Specialty policy under
the direct supervision and stewardship of the Board of Nursing. Who shall comprise the
NSCC?
a. A Chairperson who is the current President of the APO a member from .the Academe, and the last
member coming from the Regulatory Board
b. The Chairperson and members of the Regulatory Board ipso facto acts as the CPE Council
c. A Chairperson, chosen from among the Regulatory Board Members, a Vice Chairperson appointed
by the BON at-large; two other members also chosen at-large; and one representing the consumer
group
d. A Chairperson who is the President of the Association from the Academe; a member from the
Regulatory Board, and the last member coming from the APO
Situation 3 Nurse Anna is a new BSEN graduate and has just passed her Licensure Examination for
Nurses in the Philippines. She has likewise been hired as a new Community Health Nurse in one of
the Rural Health Units in their City, which of the following conditions may be acceptable TRUTHS
applied to Community Health Nursing Practice.
11. Which of the following is the primary focus of community health nursing practice?
a. Cure of illnesses
b. Prevention of illness
c. Rehabilitation back to health
d. Promotion of health
12. In Community Health Nursing, which of the following is our unit of service as nurses?
a. The community.
b. The extended members of every family.
c. The individual members of the Barangay.
d. The family.
13. A very important part of the Community Health Nursing Assessment Process includes:

a. the application of professional judgment in estimating importance of facts to family and


community.
b. evaluation structures arid qualifications of health center team.
c. coordination with other sectors in relation to health concerns.
d. carrying out nursing procedures as per plan of action.
14. In community health nursing it is important to take into account the family health with an
equally important need to perform ocular inspection of the areas activities which are powerful
elements of:
a. evaluation
b. assessment
c. implementation
d. planning
15. The initial step in the PLANNING process in order to engage in any nursing project or
parties at the community level involves:
a. goal-setting
b. monitoring
c. evaluation of data
d. provision of data
Situation 4 Please continue responding as a professional nurse in these other health situations
through the following questions.
16. Transmission of HIV from an infected individual to another person occurs:
a. Most frequency in nurses with needlesticks.
b. Only if there is a large viral load in the blood.
c. Most commonly as a result of sexual contact.
d. In all infants born to women with HIV infection.
17. The medical record of a client reveals a condition in which the fetus cannot pass through
the maternal pelvis. The nurse interprets this as:

a. Contracted pelvis
b. Maternal disproportion
c. Cervical insufficiency
d. Fetopelvic disproportion
18. The nurse would anticipate a cesarean birth for a client who has which infection present at
the onset of labor?
a. Herpes simplex virus
b. Human papillomavirus
c. Hepatitis
d. Toxoplasmosis
19. After a vaginal examination, the nurse determines that the clients fetus is in an occiput
posterior position. The nurse would anticipate that the client will have:
a. A precipitous birth
b. Intense back pain
c. Frequent leg cramps
d. Nausea and vomiting
20. The rationales for using a prostaglandin gel for a client prior to the induction of labor is to:
a. Soften and efface the cervix
b. Numb cervical pain receptors
c. Prevent cervical lacerations
d. Stimulate uterine contractions
Situation 5 Nurse Lorena is a Family Planning and Infertility Nurse Specialist and is currently
attending to a meeting. The following conditions pertain to meeting the nursing of this particular
population group.
21. Dina, 17 years old, asks you how a tubal ligation prevents pregnancy. Which would be the
best answer?

a. Prostaglandins released from the cut fallopian tubes can kill sperm
b. Sperm cannot enter the uterus, because the cervical entrance is blocked
c. Sperm can no longer reach the ova, because the fallopian tubes are blocked
d. The ovary no longer releases ova, as there is no where for them to go
22. The McMichaels are a couple undergoing testing for infertility. Infertility is said to exist
when:
a. a woman has no uterus
b. a woman has no children
c. a couple has been trying to conceive for 1 year
d. a couple has wanted a child for 6 months
23. Another client names Lilia is diagnosed as having endometriosis. This condition interferes
with the fertility because:
a. endometrial implants can block the fallopian tubes
b. the uterine cervix becomes inflamed and swollen
c. ovaries stop producing adequate estrogen
d. pressure on the pituitary leads to decreased FSH levels
24. Lilia is scheduled to have a hysterosalpingogram. Which of the following, instructions
would you give her regarding this procedure?
a. She will not be able to conceive for 3 months after the procedure
b. The sonogram of the uterus will reveal any tumors present
c. Many women experience mild bleeding as an after effect
d. She may feel some cramping when the dye is inserted
25. Lilias cousin on the other hand, knowing nurse Lorenas specialization asks what artificial
insemination by donor entails. Which would be your best answer if you were Nurse Lorena?
a. Donor sperm are introduced vaginally into the uterus or cervix
b. Donor sperm are injected intra-abdominally into each ovary

c. Artificial sperm are injected vaginally to test tubal patency


d. The husbands sperm is administered intravenously weekly
Situation 6 There are other important basic knowledge in the performance of our task as
Community Health Nurse in relation to IMMUNIZATION these include:
26. The correct temperature to store vaccines in a refrigerator is:
a. between -4 deg C and +8 deg C
b. between 2 deg C and +8 deg C
c. between -8 deg C and 0 deg C
d. between -8 deg C and +8 deg C
27. Which of the following vaccines is not done by intramuscular (IM) injection?
a. Measles vaccine
b. DPT
c. Hepa B vaccines
d. DPT
28. This vaccine content is derived from RNA recombinants:
a. Measles
b. Tetanus toxoids
c. Hepatitis B vaccines
d. DPT
29. This is the vaccine needed before a child reaches one (1) year in order for him/her to qualify
as a fully immunized child.
a. DPT
b. Measles
c. Hepatitis B
d. BCG

30. Which of the following dose of tetanus toxoid is given to the mother to protect her infant
from neonatal tetanus and likewise provide 10 years protection for the mother?
a. Tetanus toxoid 3
b. Tetanus toxoid 2
c. Tetanus toxoid 1
d. Tetanus toxoid 4
Situation 7 Records contain those, comprehensive descriptions of patients health conditions and
needs and at the same serve as evidences of every nurses accountability in the, caregiving process.
Nursing records normally differ from institution to, institution nonetheless they follow similar
patterns of meeting needs for specifics, types of information. The following pertains to
documentation/records management.
31. This special form used when the patient is admitted to the unit. The nurse completes, the
information in this records particularly his/her basic personal data, current illness, previous
health history, health history of the family, emotional profile, environmental history as well as
physical assessment together with nursing diagnosis on admission. What do you call this
record?
a. Nursing Kardex
b. Nursing Health History and Assessment Worksheet
c. Medicine and Treatment Record
d. Discharge Summary
32. These, are sheets/forms which provide an efficient and time saving way to record
information that must be obtained repeatedly at regular and/or short intervals, of time. This
does not replace the progress notes; instead this record of information on vital signs, intake and
output, treatment, postoperative care, postpartum care, and diabetic regimen, etc., this is used
whenever specific measurements or observations are needed to-be documented repeatedly.
What is this?
a. Nursing Kardex
b. Graphic Flowsheets

c. Discharge Summary
d. Medicine and Treatment Record
33. These records show all medications and treatment provided on a repeated basis. What do
you call this record?
a. Nursing Health History and Assessment Worksheet
b. Discharge Summary
c. Nursing Kardex
d. Medicine and Treatment Record
34. This flip-over card is usually kept in a portable file at the Nurses Station. It has 2-parts: the
activity and treatment section and a nursing care plan section. This carries information about
basic demographic data, primary medical diagnosis, current orders of the physician to be
carried out by the nurse, written nursing care plan, nursing orders, scheduled tests and
procedures, safety precautions in-patient care and factors related to daily living activities/ this
record is used in the charge-of-shift reports or during the bedside rounds or walking rounds.
What record is this?
a. Discharge Summary
b. Medicine and Treatment Record
c. Nursing Health History and Assessment Worksheet
d. Nursing Kardex
35. Most nurses regard this as conventional recording of the date, time and mode by which the
patient leaves a healthcare unit but this record includes importantly, directs of planning for
discharge that starts soon after the person is admitted to a healthcare institution, it is accepted
that collaboration or multidisciplinary involvement (of all members of the health team) in
discharge results in comprehensive care. What do you call this?
a. Discharge Summary
b. Nursing Kardex
c. Medicine and Treatment Record
d. Nursing Health History and Assessment Worksheet

Situation 8 As Filipino Professional Nurses we must be knowledgeable, about the Code of Ethics
for Filipino Nurses and practice these by heart. The next questions pertain to this Code of Ethics.
36. Which of the following is TRUE about the Code of Ethics of Filipino Nurses?
a. The Philippine Nurses Association for being the accredited professional organization was given the
privilege to formulate a Code of Ethics which the Board of Nurses promulgated
b. Code of Nurses was first formulated in 1982 published in the Proceedings of the Third Annual
Convention of the PNA House of Delegates
c. The present code utilized the Code of Good Governance for the Professions in the Philippines
d. Certificate of Registration of registered nurses; may be revoked or suspended for violations of any
provisions of the Code of Ethics
37. Based on the Code of Ethics for Filipino Nurses, what is regarded as the hallmark of
nursing responsibility and accountability?
a. Human rights of clients, regardless of creed and gender
b. The privilege of being a registered professional nurses
c. Health, being a fundamental right of every individual
d. Accurate documentation of actions and outcomes
38. Which of the following nurses behavior is regarded as a violation of the Code of Ethics of
Filipino Nurses?
a. A nurse withholding harmful information to the family members of a patient
b. A nurse declining commission sent by a doctor for her referral
c. A nurse endorsing a person running for congress
d. Nurse Reviewers and/or nurse review center managers who pays a considerable amount of cash for
reviewees who would memorize items from the Licensure exams and submit these to them after the
examination
39. A nurse should be cognizant that professional programs for specialty certification by the
Board of Nursing are accredited through the

a. Professional Regulation Commission


b. Nursing Specialty Certification Council
c. Association of Deans of Philippine Colleges of Nursing
d. Philippine Nurse Association
40. Mr. Santos, R.N. works in a nursing home, and he knows that one of his duties is to be an
advocate for his patients. Mr. Santos knows a primary duty of an advocate is to:
a. act as the patients legal representative
b. complete all nursing responsibilities on time
c. safeguard the wellbeing of every patient
d. maintain the patients right to privacy
Situation 9 Nurse Joanna works as an OB-Gyn Nurse and attends to several HIGH-RISK
PREGNANCIES: Particularly women with preexisting or newly acquired illness. The following
conditions apply.
41. Bernadette is a 22-year old woman. Which condition would make her more prone than
others to developing a Candida infection during pregnancy?
a. Her husband plays golf 6 days a week
b. She was over 35 when she became pregnant
c. She usually drinks tomato juice for breakfast
d. She has developed gestational diabetes
42. Bernadette develops a deep-vein thrombosis following an auto accident and is prescribed
heparin sub-Q. What should Joanna educate her about in regard to this?
a. Some infants will be born with allergic symptoms to heparin
b. Her infant will be born with scattered petechiae on his trunk
c. Heparin can cause darkened skin in newborns
d. Heparin does not cross the placenta and so does not affect a fetus
43. The cousin of Bernadette with sickle-cell anemia alerted Joanna that she may need further
instruction on prenatal care. Which statement signifies this fact?

a. Ive stopped jogging so I dont risk becoming dehydrated


b. I take an iron pull every day to help grow new red blood cells
c. I am careful to drink at least eight glasses of fluid everyday
d. 1 understand why folic acid is important for red cell formation
44. Bernadette routinely takes acetylsalicylic acid (aspirin) for arthritis. Why should she limit
or discontinue this toward the end of pregnancy?
a. Aspirin can lead to deep vein thrombosis following birth
b. Newborns develop a red rash from salicylate toxicity
c. Newborns develop withdrawal headaches from salicylates
d. Salicylates can lead to increased maternal bleeding at childbirth
45. Bernadette received a laceration on her leg from her automotive accident. Why are
lacerations of lower extremities potentially more serious in pregnant women than others?
a. Lacerations can provoke allergic responses because of gonadotropic hormone
b. Increased bleeding can occur from uterine pressure on leg veins
c. A woman is less able to keep the laceration clean because o f her fatigue
d. Healing is limited during pregnancy, so these will not heal until after birth
Situation 10 Still in your self-managed Child Health Nursing Clinic, your encounter these cases
pertaining to the CARE OF CHILDREN WITH PULMONARY AFFECTIONS.
46. Josie brought her 3-months old child to your clinic because of cough and colds. Which of
the following is your primary action?
a. Give cotrimoxazole tablet or syrup
b. Assess the patient using the chart on management of children with cough
c. Refer to the doctor
d. Teach the mother how to count her childs bearing
47. In responding to the care concerns of children with severe disease, referral to the hospital of
the essence especially if the child manifests which of the following?

a. Wheezing
b. Stopped bleeding
c. Fast breathing
d. Difficulty to awaken
48. Which of the following is the most important responsibility of a nurse in the prevention of
necessary deaths from pneumonia and other severe diseases?
a. Giving of antibiotics
b. Taking of the temperature of the sick child
c. Provision of Careful Assessment
d. Weighing of the sick child
49. You were able to identify factors that lead to respiratory problems in the community where
your health facility serves. Your primary role therefore in order to reduce morbidity due to
pneumonia is to:
a. Teach mothers how to recognize early signs and symptoms of pneumonia
b. Make home visits to sick children
c. Refer cases to hospitals
d. Seek assistance and mobilize the BHWs to have a meeting with mothers
50. Which of the following is the principal focus on the CARI program of the Department of
Health?
a. Enhancement of health team capabilities
b. Teach mothers how to detect signs and where to refer
c. Mortality reduction through early detection
d. Teach other community health workers how to assess patients
Situation 11 You are working as a Pediatric Nurse in your own Child Health Nursing Clinic, the
following cases pertain to ASSESSMENT AND CARE OF THE NEWBORN AT RISK conditions.

51. Theresa, a mother with a 2 year old daughter asks, At what are can I be able to take the
blood pressure of my daughter as a routine procedure since hypertension is common in the
family? Your answer to this is:
a. At 2 years you may
b. As early as 1 year old
c. When shes 3- years old
d. When shes 6 years old
52. You typically gag children to inspect the back of their throat. When is it important NOT to
solicit a gag reflex?
a. when a girl has a geographic tongue
b. when a boy has a possible inguinal hernia
c. when a child has symptoms of epiglottitis
d. when children are under 5 years of age
53. Baby John was given a drug at birth to reverse the effects of a narcotic given to his mother
in labor. What drug is commonly used for this?
a. Naloxone (Narcan)
b. Morphine Sulfate
c. Sodium Chloride
d. Penicillin G
54. Why are small-for-gestational-age newborns at risks for difficulty maintaining body
temperature?
a. They do not have as many fat stores as other infants
b. They are more active than usual so throw off covers
c. Their skin is more susceptible to conduction of cold
d. They are preterm so are born relatively small in size
55. Baby John develops hyperbilirubinemia. What is a method used to treat
hyperbilirubinemia in a newborn?

a. Keeping infants in a warm arid dark environment


b. Administration of a cardiovascular stimulant
c. Gentle exercise to stop muscle breakdown
d. Early feeding to speed passage of meconium
Situation 12 You are the nurse in the Out-Patient-Department and during your shift you
encountered multiple childrens condition. The following questions apply.
56. You assessed a child with visible severe wasting, he has:
a. edema
b. LBM
c. kwashiorkor
d. marasmus
57. Which of the following conditions is NOT true about contraindication to immunization?
a. do not give DPT2 or DPT3 to a child who has convulsions within 3 days of DPT1
b. do not give BOG if the child has known hepatitis .
c. do not give OPT to a child who has recurrent convulsion or active neurologic disease
d. do not give BCG if the child has known AIDS
58. Which of the following statements about immunization is NOT true:
a. A child with diarrhea who is due for OPV should receive the OPV and make extra dose on the next
visit
b. There is no contraindication to immunization if the child is well enough to go home
c. There is no contraindication to immunization if the child is well enough to go home and a child
should be immunized in the health center before referrals are both correct
d. A child should be immunized in the center before referral
59. A child with visible severe wasting or severe palmar pallor may be classified as:
a. moderate malnutrition/anemia
b. severe malnutrition/anemia

c. not very tow weight no anemia


d. anemia/very low weight
60. A child who has some palmar pallor can be classified as:
a. moderate anemia/normal weight
b. severe malnutrition/anemia
c. anemia/very low weight
d. not very low eight to anemia
Situation 13 Nette, a nurse palpates the abdomen of Mrs. Medina, a primigravida. She is unsure of
the date of her last menstrual period. Leopolds Maneuver is done. The obstetrician told mat she
appears to be 20 weeks pregnant. .
61. Nette explains this because the fundus is:
a. At the level the umbilicus, and the fetal heart can be heard with a fetoscope
b. 18 cm, and the baby is just about to move
c. is just over the symphysis, and fetal heart cannot be heard
d. 28 cm, and fetal heart can be heard with a Doppler
62. In doing Leopolds maneuver palpation which among the following is NOT considered a
good preparation?
a. The woman should lie in a supine position with her knees flexed slightly
b. The hands of the nurse should be cold so that abdominal muscles would contract and tighten
c. Be certain that your hands are warm (by washing them in warm water first if necessary)
d. The woman empties her bladder before palpation
63. In her pregnancy, she experienced fatigue and drowsiness. This probably occurs because:
a. of high blood pressure
b. she is expressing pressure
c. the fetus utilizes her glucose stores and leaves her with a Sow blood glucose
d. of the rapid growth of the fetus

64. The nurse assesses the woman at 20 weeks gestation and expects the woman to report:
a. Spotting related to fetal implantation
b. Symptoms of diabetes as human placental lactogen is released
c. Feeling fetal kicks
d. Nausea and vomiting related HCG production
65. If Mrs. Medina comes to you for check-up on June 2, her EDO is June 11, what do you
expect during assessment?
a. Fundic ht 2 fingers below xyphoid process, engaged
b. Cervix close, uneffaced, FH-midway between the umbilicus and symphysis pubis
c. Cervix open, fundic ht. 2 fingers below xyphoid process, floating .
d. Fundal height at least at the level of the xyphoid process, engaged
Situation 14: Please continue responding as a professional nurse in varied health situations through
the following questions.
66. Which of the following medications would the nurse expect the physician to order for
recurrent convulsive seizures of a 10-year old child brought to your clinic?
a. Phenobarbital
b. Nifedipine
c. Butorphanol
d. Diazepam
67. RhoGAM is given to Rh-negative women to prevent maternal sensitization from occurring.
The nurse is aware that in addition to pregnancy, Rh-negative women would also receive this
medication after which of the following?
a. Unsuccessful artificial insemination procedure
b. Blood transfusion after hemorrhage
c. Therapeutic or spontaneous abortion
d. Head injury from a car accident

68. Which of the following would the nurse include when describing the pathophysiology of
gestational diabetes?
a. Glucose levels decrease to accommodate fetal growth
b. Hypoinsulinemia develops early in the first trimester
c. Pregnancy fosters the development of carbohydrate cravings
d. There is progressive resistance to the effects of insulin
69. When providing prenatal education to a pregnant woman with asthma, which of the
following would be important for the nurse to do?
a. Demonstrate how to assess her blood glucose
b. Teach correct administration of subcutaneous bronchodilators
c. Ensure she seeks treatment for any acute exacerbation
d. Explain that she should avoid steroids during her pregnancy
70. Which of the following conditions would cause an insulin-dependent diabetic client the most
difficulty during her pregnancy?
a. Rh incompatibility
b. Placenta previa
c. Hyperemesis gravidarum
d. Abruptio placentae
Situation 15 One important toot a community health nurse uses in the conduct of his/her activities
is the CHN Bag. Which of the following BEST DESCRIBES the use of this vital facility for our
practice?
71. The Community/Public Health Bag is:
a. a requirement for home visits
b. an essential and indispensable equipment of the community health nurse
c. contains basic medications and articles used by the community health nurse
d. a tool used by the Community health nurse is rendering effective nursing procedure during a home
visit

72. What is the rationale in the use of bag technique during home visit?
a. It helps render effective nursing care to clients or other members of the family
b. It saves time and effort of the nurse in the performance of nursing procedures
c. It should minimize or prevent the spread of infection from individuals to families
d. It should not overshadow concerns for the patient
73. Which among the following is important in the use of the bag technique during home visit?
a. Arrangement of the bags contents must be convenient to the nurse
b. The bag should contain all necessary supplies and equipment ready for use
c. Be sure to thoroughly clean your bag especially when exposed to communicable disease cases
d. Minimize if not totally prevent the spread of infection
74. This is an important procedure of the nurse during home visits?
a. protection of the CHN bag
b. arrangement of the contents of the CHM bag
c. cleaning of the CHN bag
d. proper handwashing
75. In consideration of the steps in applying the bag technique, which side of the paper lining of
the CHN bag is considered clean to make a non-contaminated work area?
a. The lower lip
b. The outer surface
c. The upper lip
d. The inside surface
Situation 16 As a Community Health Nurse relating with people in different communities, and in
the implementation of health programs and projects you experience vividly as well the varying forms
of leadership and management from the Barangay Level to the Local Government/Municipal City
Level.
76. The following statements can correctly be made about Organization and management?

A. An organization (or company) is people. Values make people persons: values give vitality,
meaning and direction to a company. As the people of an organization value, so the company
becomes.
B. Management is the process by which administration achieves its mission, goals, and objectives
C. Management effectiveness can be measured in terms of accomplishment of the purpose of the
organization while management efficiency is measured in terms of the satisfaction of individual
motives
D. Management principles are universal therefore one need not be concerned about people, culture,
values, traditions and human relations.
a. B and C only
b. A, B and D only
c. A and D only
d. B, A, and C only
77. Management by Filipino values advocates the consideration of the Filipino goals trilogy
according to the Filipino priority-values which are:
a. Family goals, national goals, organizational goals
b. Organizational goats, national goals, family goals
c. National goals, organizational goals, family goals
d. Family goals, organizational goals, national goals
78. Since the advocacy for the utilization of Filipino value-system in management has been
encouraged, the Nursing sector is no except, management needs to examine Filipino values and
discover its positive potentials and harness them to achieve:
a. Employee satisfaction
b. Organizational commits .ants, organizational objectives and employee satisfaction
c. Employee objectives/satisfaction, commitments and organizational objectives
d. Organizational objectives, commitments and employee objective/satisfaction
79. The following statements can correctly be made about an effective and efficient community
or even agency managerial-leader.

A. Considers the achievement and advancement of the organization she/he represents as well as his
people
B. Considers the recognition of individual efforts toward the realization of organizational goals as
well as the welfare of his people
C. Considers the welfare of the organization above all other consideration by higher administration
D. Considers its own recognition by higher administration for purposes of promotion and prestige
a. Only C and D are correct
b. A, C and D are correct
c. B, C, and D are correct
d. Only A and B are correct
80. Whether management at the community or agency level, there are 3 essential types of skills
managers must have, these are:
A. Human relation skills, technical skills, and cognitive skills
B. Conceptual skills, human relation/behavioral skills, and technical skills
C. Technical skills, budget and accounting skills, skills in fund-raising
D. Manipulative skills, technical skills, resource management skills
a. A and D are correct
b. B is correct
c. A is correct
d. C and D are correct
Situation 17 You are actively practicing nurse who just finished your Graduate Studies. You earned
the value of Research and would like to utilize the knowledge and skills gained in the application of
research to Nursing service. The following questions apply to research.
81. Which type of research Inquiry investigates the issue of human complexity (e.g.
understanding the human expertise)
a. Logical position
b. Naturalistic inquiry

c. Positivism
d. Quantitative Research
82. Which of the following studies is based on quantitative research?
a. A study examining the bereavement process in spouses of clients with terminal cancer
b. A study exploring factors influencing weight control behavior
c. A study measuring the effects of sleep deprivation on wound healing
d. A study examining clients feelings before, during and after a bone marrow aspiration
83. Which of the following studies is based on qualitative research?
a. A study examining clients reactions to stress after open heart surgery
b. A study measuring nutrition and weight, loss/gain in clients with cancer
c. A study examining oxygen levels after endotracheal suctioning
d. A study measuring differences in blood pressure before during and after a procedure
84. An 85 year old client in a nursing home tells a nurse, I signed the papers for that research
study because the doctor was so insistent and I want: him to continue taking care of me.
Which client right is being violated?
a. Right of self determination
b. Right to privacy and confidentiality
c. Right to full disclosure
d. Right not to be harmed
85. A supposition or system of ideas that is proposed to explain a given phenomenon, best
defines:
a. a paradigm
b. a concept
c. a theory
d. a conceptual framework

Situation 18 Nurse Michelle works with a Family Nursing Team in Calbayog Province specifically
handling a UNICEF project for children. The following conditions pertain, to CARE OP THE
FAMILIES PRESCHOOLERS.
86. Ronnie asks constant questions. How many does a typical 3-year-old ask in a days time?
a. 1,200 or more
b. Less than 50
c. 100-200
d. 300-400
87. Ronnie will need to change to a new bed because his baby sister will need Ronnies old crib.
What measure would you suggest that his parents take to help decrease sibling rivalry between
Ronnie and his new sister?
a. Move him to the new bed before the baby arrives
b. Explain that new sisters grow up to become best friends
c. Tell him he will have to share with the new baby
d. Ask him to get his crib ready for the new baby
88. Ronnies parents want to know how to react to him when he begins to masturbate while
watching television. What would you suggest?
a. They refuse to allow him to watch television
b. They schedule a health check-up for sex-related disease
c. They remind him that some activities are private
d. They give him timeout when this begins
89. How many words does a typical 12-month-old infant use?
a. About 12 words
b. Twenty or more words
c. About 50 words
d. Two, plus mama and dada

90. As a nurse. You reviewed infant safety procedures with Bryans mother. What are two of the
most common types of accidents among infants?
a. Aspiration and falls
b. Falls and auto accidents
c. Poisoning and burns
d. Drowning and homicide
Situation 19 Among common conditions found in children especially among poor communities are
ear infection/problems. The following questions apply.
91. A child with ear problem should be assessed for the following EXCEPT:
a is there any fever?
b. ear discharge
c. if discharge is present for how long?
d. ear pain
92. If the child does not have ear problem, using IMCI, what should you as the nurse do?
a. Check for ear discharge
b. Check for tender swellings, behind the ear
c. Check for ear pain
d. Go to the next question, check for malnutrition
93. An ear discharge that has been present for more than 14 days can be classified as:
a. mastoiditis
b. chronic ear infection
c. acute ear infection
d. complicated ear infection
94. An ear discharge that has been present for less than 14 days can be classified as:
a. chronic ear infection
b. mastoiditis

c. acute ear infection


d. complicated ear infection
95. If the child has severe classification because of ear problem, what would be the best thing
that you as the nurse can do?
a. instruct mother when to return immediately
b. refer urgently
c. give an antibiotic for 5 days
d. dry the ear by wicking
Situation 20 If a child with diarrhea registers one sign in the pink row and one in the yellow; row in
the IMCI Chart.
96. We can classify the patient as:
a. moderate dehydration
b. some dehydration
c. no dehydration
d. severe dehydration
97. The child with no dehydration needs home treatment Which of the following is not included
the rules for home treatment in this case:
a. continue feeding the child
b. give oresol every 4 hours
c. know when to return to the health center
d. give the child extra fluids
98. A child who has had diarrhea for 14 days but has no sign of dehydration is classified as:
a. severe persistent diarrhea
b. dysentery
c. severe dysentery b. dysentery
d. persistent diarrhea

99. If the child has sunken eyes, drinking eagerly, thirsty and skin pinch goes back slowly, the
classification would be:
a. no dehydration
b. moderate dehydration
c. some dehydration
d. severe dehydration
100. Carlo has had diarrhea for 5 days. There is no blood in the stool, he is irritable. His eyes
are sunken the nurse offers fluid to Carlo and he drinks eagerly. When the nurse pinched the
abdomen, it goes back slowly. How will you classify Carlos illness?
a. severe dehydration
b. no dehydration
c. some dehydration
d. moderate dehydration

Answers
Here are the answers for the exam. Unfortunately, rationales are not given. If you need clarifications
or disputes, please direct them to the comments section and well be glad to give you an explanation.
1. A
2. B
3. A
4. C
5. B
6. D
7. C
8. D
9. A
10. B
11. D
12. D
13. A

14. B
15. A
16. C
17. D
18. A
19. B
20. D
21. C
22. C
23. A
24. C
25. A
26. B
27. A
28. C
29. B
30. D
31. B
32. B
33. D
34. D
35. A
36. C
37. C
38. A
39. B
40. C
41. D
42. D
43. B
44. D
45. B

46. B
47. D
48. C
49. A
50. C
51. C
52. C
53. A
54. A
55. D
56. D
57. B
58. A
59. B
60.C
61. A
62. B
63. D
64. C
65. A
66. A
67. C
68. D
69. C
70. C
71. B
72. A
73. D
74. D
75. B
76. D
77. D

78. D
79. D
80. C
81. B
82. C
83. A
84. A
85. C
86. D
87. A
88. C
89. A
90. A
91. A
92. D
93. B
94. C
95. B
96. D
97. B
98. D
99. C
100. C
1. May arrives at the health care clinic and tells the nurse that her last menstrual period was 9
weeks ago. She also tells the nurse that a home pregnancy test was positive but she began to
have mild cramps and is now having moderate vaginal bleeding. During the physical
examination of the client, the nurse notes that May has a dilated cervix. The nurse determines
that May is experiencing which type of abortion?
a. Inevitable
b. Incomplete

c. Threatened
d. Septic
2. Nurse Reese is reviewing the record of a pregnant client for her first prenatal visit. Which of
the following data, if noted on the clients record, would alert the nurse that the client is at risk
for a spontaneous abortion?
a. Age 36 years
b. History of syphilis
c. History of genital herpes
d. History of diabetes mellitus
3. Nurse Hazel is preparing to care for a client who is newly admitted to the hospital with a
possible diagnosis of ectopic pregnancy. Nurse Hazel develops a plan of care for the client and
determines that which of the following nursing actions is the priority?
a. Monitoring weight
b. Assessing for edema
c. Monitoring apical pulse
d. Monitoring temperature
4. Nurse Oliver is teaching a diabetic pregnant client about nutrition and insulin needs during
pregnancy. The nurse determines that the client understands dietary and insulin needs if the
client states that the second half of pregnancy require:
a. Decreased caloric intake
b. Increased caloric intake
c. Decreased Insulin
d. Increase Insulin
5. Nurse Michelle is assessing a 24 year old client with a diagnosis of hydatidiform mole. She is
aware that one of the following is unassociated with this condition?

a. Excessive fetal activity.


b. Larger than normal uterus for gestational age.
c. Vaginal bleeding
d. Elevated levels of human chorionic gonadotropin.
6. A pregnant client is receiving magnesium sulfate for severe pregnancy induced hypertension
(PIH). The clinical findings that would warrant use of the antidote , calcium gluconate is:
a. Urinary output 90 cc in 2 hours.
b. Absent patellar reflexes.
c. Rapid respiratory rate above 40/min.
d. Rapid rise in blood pressure.
7. During vaginal examination of Janna who is in labor, the presenting part is at station plus
two. Nurse, correctly interprets it as:
a. Presenting part is 2 cm above the plane of the ischial spines.
b. Biparietal diameter is at the level of the ischial spines.
c. Presenting part in 2 cm below the plane of the ischial spines.
d. Biparietal diameter is 2 cm above the ischial spines.
8. A pregnant client is receiving oxytocin (Pitocin) for induction of labor. A condition that
warrant the nurse in-charge to discontinue I.V. infusion of Pitocin is:
a. Contractions every 1 minutes lasting 70-80 seconds.
b. Maternal temperature 101.2
c. Early decelerations in the fetal heart rate.
d. Fetal heart rate baseline 140-160 bpm.
9. Calcium gluconate is being administered to a client with pregnancy induced hypertension
(PIH). A nursing action that must be initiated as the plan of care throughout injection of the
drug is:

a. Ventilator assistance
b. CVP readings
c. EKG tracings
d. Continuous CPR
10. A trial for vaginal delivery after an earlier caesareans, would likely to be given to a gravida,
who had:
a. First low transverse cesarean was for active herpes type 2 infections; vaginal culture at 39 weeks
pregnancy was positive.
b. First and second caesareans were for cephalopelvic disproportion.
c. First caesarean through a classic incision as a result of severe fetal distress.
d. First low transverse caesarean was for breech position. Fetus in this pregnancy is in a vertex
presentation.
11. Nurse Ryan is aware that the best initial approach when trying to take a crying toddlers
temperature is:
a. Talk to the mother first and then to the toddler.
b. Bring extra help so it can be done quickly.
c. Encourage the mother to hold the child.
d. Ignore the crying and screaming.
12. Baby Tina a 3 month old infant just had a cleft lip and palate repair. What should the nurse
do to prevent trauma to operative site?
a. Avoid touching the suture line, even when cleaning.
b. Place the baby in prone position.
c. Give the baby a pacifier.
d. Place the infants arms in soft elbow restraints.
13. Which action should nurse Marian include in the care plan for a 2 month old with heart
failure?

a. Feed the infant when he cries.


b. Allow the infant to rest before feeding.
c. Bathe the infant and administer medications before feeding.
d. Weigh and bathe the infant before feeding.
14. Nurse Hazel is teaching a mother who plans to discontinue breast feeding after 5 months.
The nurse should advise her to include which foods in her infants diet?
a. Skim milk and baby food.
b. Whole milk and baby food.
c. Iron-rich formula only.
d. Iron-rich formula and baby food.
15. Mommy Linda is playing with her infant, who is sitting securely alone on the floor of the
clinic. The mother hides a toy behind her back and the infant looks for it. The nurse is aware
that estimated age of the infant would be:
a. 6 months
b. 4 months
c. 8 months
d. 10 months
16. Which of the following is the most prominent feature of public health nursing?
a. It involves providing home care to sick people who are not confined in the hospital.
b. Services are provided free of charge to people within the catchments area.
c. The public health nurse functions as part of a team providing a public health nursing services.
d. Public health nursing focuses on preventive, and not curative, services.
17. When the nurse determines whether resources were maximized in implementing Ligtas
Tigdas (Measles Prevention), she is evaluating:
a. Effectiveness
b. Efficiency

c. Adequacy
d. Appropriateness
18. Vangie is a new B.S.N. graduate. She wants to become a Public Health Nurse. Where should
she apply?
a. Department of Health
b. Provincial Health Office
c. Regional Health Office
d. Rural Health Unit
19. Tony is aware the Chairman of the Municipal Health Board is:
a. Mayor
b. Municipal Health Officer
c. Public Health Nurse
d. Any qualified physician
20. Myra is the public health nurse in a municipality with a total population of about 20,000.
There are 3 rural health midwives among the RHU personnel. How many more midwife items
will the RHU need?
a. 1
b. 2
c. 3
d. The RHU does not need any more midwife item.
21. According to Freeman and Heinrich, community health nursing is a developmental service.
Which of the following best illustrates this statement?
a. The community health nurse continuously develops himself personally and professionally.
b. Health education and community organizing are necessary in providing community health
services.
c. Community health nursing is intended primarily for health promotion and prevention and

treatment of disease.
d. The goal of community health nursing is to provide nursing services to people in their own places
of residence.
22. Nurse Tina is aware that the disease declared through Presidential Proclamation No. 4 as a
target for eradication in the Philippines is?
a. Poliomyelitis
b. Measles
c. Rabies
d. Neonatal tetanus
23. May knows that the step in community organizing that involves training of potential
leaders in the community is:
a. Integration
b. Community organization
c. Community study
d. Core group formation
24. Beth a public health nurse takes an active role in community participation. What is the
primary goal of community organizing?
a. To educate the people regarding community health problems
b. To mobilize the people to resolve community health problems
c. To maximize the communitys resources in dealing with health problems.
d. To maximize the communitys resources in dealing with health problems.
25. Tertiary prevention is needed in which stage of the natural history of disease?
a. Pre-pathogenesis
b. Pathogenesis
c. Prodromal
d. Terminal

26. The nurse is caring for a primigravid client in the labor and delivery area. Which condition
would place the client at risk for disseminated intravascular coagulation (DIC)?
a. Intrauterine fetal death.
b. Placenta accreta.
c. Dysfunctional labor.
d. Premature rupture of the membranes.
27. A full term client is in labor. Nurse Betty is aware that the fetal heart rate would be:
a. 80 to 100 beats/minute
b. 100 to 120 beats/minute
c. 120 to 160 beats/minute
d. 160 to 180 beats/minute
28. The skin in the diaper area of a 7 month old infant is excoriated and red. Nurse Hazel
should instruct the mother to:
a. Change the diaper more often.
b. Apply talc powder with diaper changes.
c. Wash the area vigorously with each diaper change.
d. Decrease the infants fluid intake to decrease saturating diapers.
29. Nurse Carla knows that the common cardiac anomalies in children with Down Syndrome
(trisomy 21) is:
a. Atrial septal defect
b. Pulmonic stenosis
c. Ventricular septal defect
d. Endocardial cushion defect
30. Malou was diagnosed with severe preeclampsia is now receiving I.V. magnesium sulfate.
The adverse effects associated with magnesium sulfate is:

a. Anemia
b. Decreased urine output
c. Hyperreflexia
d. Increased respiratory rate
31. A 23 year old client is having her menstrual period every 2 weeks that last for 1 week. This
type of menstrual pattern is bets defined by:
a. Menorrhagia
b. Metrorrhagia
c. Dyspareunia
d. Amenorrhea
32. Jannah is admitted to the labor and delivery unit. The critical laboratory result for this
client would be:
a. Oxygen saturation
b. Iron binding capacity
c. Blood typing
d. Serum Calcium
33. Nurse Gina is aware that the most common condition found during the second-trimester of
pregnancy is:
a. Metabolic alkalosis
b. Respiratory acidosis
c. Mastitis
d. Physiologic anemia
34. Nurse Lynette is working in the triage area of an emergency department. She sees that
several pediatric clients arrive simultaneously. The client who needs to be treated first is:
a. A crying 5 year old child with a laceration on his scalp.
b. A 4 year old child with a barking coughs and flushed appearance.

c. A 3 year old child with Down syndrome who is pale and asleep in his mothers arms.
d. A 2 year old infant with stridorous breath sounds, sitting up in his mothers arms and drooling.
35. Maureen in her third trimester arrives at the emergency room with painless vaginal
bleeding. Which of the following conditions is suspected?
a. Placenta previa
b. Abruptio placentae
c. Premature labor
d. Sexually transmitted disease
36. A young child named Richard is suspected of having pinworms. The community nurse
collects a stool specimen to confirm the diagnosis. The nurse should schedule the collection of
this specimen for:
a. Just before bedtime
b. After the child has been bathe
c. Any time during the day
d. Early in the morning
37. In doing a childs admission assessment, Nurse Betty should be alert to note which signs or
symptoms of chronic lead poisoning?
a. Irritability and seizures
b. Dehydration and diarrhea
c. Bradycardia and hypotension
d. Petechiae and hematuria
38. To evaluate a womans understanding about the use of diaphragm for family planning,
Nurse Trish asks her to explain how she will use the appliance. Which response indicates a need
for further health teaching?
a. I should check the diaphragm carefully for holes every time I use it
b. I may need a different size of diaphragm if I gain or lose weight more than 20 pounds

c. The diaphragm must be left in place for at least 6 hours after intercourse
d. I really need to use the diaphragm and jelly most during the middle of my menstrual cycle.
39. Hypoxia is a common complication of laryngotracheobronchitis. Nurse Oliver should
frequently assess a child with laryngotracheobronchitis for:
a. Drooling
b. Muffled voice
c. Restlessness
d. Low-grade fever
40. How should Nurse Michelle guide a child who is blind to walk to the playroom?
a. Without touching the child, talk continuously as the child walks down the hall.
b. Walk one step ahead, with the childs hand on the nurses elbow.
c. Walk slightly behind, gently guiding the child forward.
d. Walk next to the child, holding the childs hand.
41. When assessing a newborn diagnosed with ductus arteriosus, Nurse Olivia should expect
that the child most likely would have an:
a. Loud, machinery-like murmur.
b. Bluish color to the lips.
c. Decreased BP reading in the upper extremities
d. Increased BP reading in the upper extremities.
42. The reason nurse May keeps the neonate in a neutral thermal environment is that when a
newborn becomes too cool, the neonate requires:
a. Less oxygen, and the newborns metabolic rate increases.
b. More oxygen, and the newborns metabolic rate decreases.
c. More oxygen, and the newborns metabolic rate increases.
d. Less oxygen, and the newborns metabolic rate decreases.

43. Before adding potassium to an infants I.V. line, Nurse Ron must be sure to assess whether
this infant has:
a. Stable blood pressure
b. Patent fontanelles
c. Moros reflex
d. Voided
44. Nurse Carla should know that the most common causative factor of dermatitis in infants
and younger children is:
a. Baby oil
b. Baby lotion
c. Laundry detergent
d. Powder with cornstarch
45. During tube feeding, how far above an infants stomach should the nurse hold the syringe
with formula?
a. 6 inches
b. 12 inches
c. 18 inches
d. 24 inches
46. In a mothers class, Nurse Lynnette discussed childhood diseases such as chicken pox.
Which of the following statements about chicken pox is correct?
a. The older one gets, the more susceptible he becomes to the complications of chickenpox.
b. A single attack of chickenpox will prevent future episodes, including conditions such as shingles.
c. To prevent an outbreak in the community, quarantine may be imposed by health authorities.
d. Chicken pox vaccine is best given when there is an impending outbreak in the community.

47. Barangay Pinoy had an outbreak of German measles. To prevent congenital rubella, what is
the BEST advice that you can give to women in the first trimester of pregnancy in the
Barangay Pinoy?
a. Advise them on the signs of German measles.
b. Avoid crowded places, such as markets and movie houses.
c. Consult at the health center where rubella vaccine may be given.
d. Consult a physician who may give them rubella immunoglobulin.
48. Myrna a public health nurse knows that to determine possible sources of sexually
transmitted infections, the BEST method that may be undertaken is:
a. Contact tracing
b. Community survey
c. Mass screening tests
d. Interview of suspects
49. A 33-year old female client came for consultation at the health center with the chief
complaint of fever for a week. Accompanying symptoms were muscle pains and body malaise.
A week after the start of fever, the client noted yellowish discoloration of his sclera. History
showed that he waded in flood waters about 2 weeks before the onset of symptoms. Based on
her history, which disease condition will you suspect?
a. Hepatitis A
b. Hepatitis B
c. Tetanus
d. Leptospirosis
50. Mickey a 3-year old client was brought to the health center with the chief complaint of
severe diarrhea and the passage of rice-watery stools. The client is most probably suffering
from which condition?
a. Giardiasis
b. Cholera

c. Amebiasis
d. Dysentery
51. The most prevalent form of meningitis among children aged 2 months to 3 years is caused
by which microorganism?
a. Hemophilus influenzae
b. Morbillivirus
c. Streptococcus pneumoniae
d. Neisseria meningitidis
52. The student nurse is aware that the pathognomonic sign of measles is Kopliks spot and you
may see Kopliks spot by inspecting the:
a. Nasal mucosa
b. Buccal mucosa
c. Skin on the abdomen
d. Skin on neck
53. Angel was diagnosed as having Dengue fever. You will say that there is slow capillary refill
when the color of the nail bed that you pressed does not return within how many seconds?
a. 3 seconds
b. 6 seconds
c. 9 seconds
d. 10 seconds
54. In Integrated Management of Childhood Illness, the nurse is aware that the severe
conditions generally require urgent referral to a hospital. Which of the following severe
conditions DOES NOT always require urgent referral to a hospital?
a. Mastoiditis
b. Severe dehydration

c. Severe pneumonia
d. Severe febrile disease
55. Myrna a public health nurse will conduct outreach immunization in a barangay Masay with
a population of about 1500. The estimated number of infants in the barangay would be:
a. 45 infants
b. 50 infants
c. 55 infants
d. 65 infants
56. The community nurse is aware that the biological used in Expanded Program on
Immunization (EPI) should NOT be stored in the freezer?
a. DPT
b. Oral polio vaccine
c. Measles vaccine
d. MMR
57. It is the most effective way of controlling schistosomiasis in an endemic area?
a. Use of molluscicides
b. Building of foot bridges
c. Proper use of sanitary toilets
d. Use of protective footwear, such as rubber boots
58. Several clients is newly admitted and diagnosed with leprosy. Which of the following clients
should be classified as a case of multibacillary leprosy?
a. 3 skin lesions, negative slit skin smear
b. 3 skin lesions, positive slit skin smear
c. 5 skin lesions, negative slit skin smear
d. 5 skin lesions, positive slit skin smear

59. Nurses are aware that diagnosis of leprosy is highly dependent on recognition of symptoms.
Which of the following is an early sign of leprosy?
a. Macular lesions
b. Inability to close eyelids
c. Thickened painful nerves
d. Sinking of the nosebridge
60. Marie brought her 10 month old infant for consultation because of fever, started 4 days
prior to consultation. In determining malaria risk, what will you do?
a. Perform a tourniquet test.
b. Ask where the family resides.
c. Get a specimen for blood smear.
d. Ask if the fever is present everyday.
61. Susie brought her 4 years old daughter to the RHU because of cough and colds. Following
the IMCI assessment guide, which of the following is a danger sign that indicates the need for
urgent referral to a hospital?
a. Inability to drink
b. High grade fever
c. Signs of severe dehydration
d. Cough for more than 30 days
62. Jimmy a 2-year old child revealed baggy pants. As a nurse, using the IMCI guidelines,
how will you manage Jimmy?
a. Refer the child urgently to a hospital for confinement.
b. Coordinate with the social worker to enroll the child in a feeding program.
c. Make a teaching plan for the mother, focusing on menu planning for her child.
d. Assess and treat the child for health problems like infections and intestinal parasitism.

63. Gina is using Oresol in the management of diarrhea of her 3-year old child. She asked you
what to do if her child vomits. As a nurse you will tell her to:
a. Bring the child to the nearest hospital for further assessment.
b. Bring the child to the health center for intravenous fluid therapy.
c. Bring the child to the health center for assessment by the physician.
d. Let the child rest for 10 minutes then continue giving Oresol more slowly.
64. Nikki a 5-month old infant was brought by his mother to the health center because of
diarrhea for 4 to 5 times a day. Her skin goes back slowly after a skin pinch and her eyes are
sunken. Using the IMCI guidelines, you will classify this infant in which category?
a. No signs of dehydration
b. Some dehydration
c. Severe dehydration
d. The data is insufficient.
65. Chris a 4-month old infant was brought by her mother to the health center because of
cough. His respiratory rate is 42/minute. Using the Integrated Management of Child Illness
(IMCI) guidelines of assessment, his breathing is considered as:
a. Fast
b. Slow
c. Normal
d. Insignificant
66. Maylene had just received her 4th dose of tetanus toxoid. She is aware that her baby will
have protection against tetanus for
a. 10 year
b. 5 years
c. 3 years
d. Lifetime

67. Nurse Ron is aware that unused BCG should be discarded after how many hours of
reconstitution?
a. 2 hours
b. 4 hours
c. 8 hours
d. At the end of the day
68. The nurse explains to a breastfeeding mother that breast milk is sufficient for all of the
babys nutrient needs only up to:
a. 5 months
b. 6 months
c. 1 year
d. 2 years
69. Nurse Ron is aware that the gestational age of a conceptus that is considered viable (able to
live outside the womb) is:
a. 8 weeks
b. 12 weeks
c. 24 weeks
d. 32 weeks
70. When teaching parents of a neonate the proper position for the neonates sleep, the nurse
Patricia stresses the importance of placing the neonate on his back to reduce the risk of which
of the following?
a. Aspiration
b. Sudden infant death syndrome (SIDS)
c. Suffocation
d. Gastroesophageal reflux (GER)
71. Which finding might be seen in baby James a neonate suspected of having an infection?

a. Flushed cheeks
b. Increased temperature
c. Decreased temperature
d. Increased activity level
72. Baby Jenny who is small-for-gestation is at increased risk during the transitional period for
which complication?
a. Anemia probably due to chronic fetal hypoxia
b. Hyperthermia due to decreased glycogen stores
c. Hyperglycemia due to decreased glycogen stores
d. Polycythemia probably due to chronic fetal hypoxia
73. Marjorie has just given birth at 42 weeks gestation. When the nurse assessing the neonate,
which physical finding is expected?
a. A sleepy, lethargic baby
b. Lanugo covering the body
c. Desquamation of the epidermis
d. Vernix caseosa covering the body
74. After reviewing the Myrnas maternal history of magnesium sulfate during labor, which
condition would nurse Richard anticipate as a potential problem in the neonate?
a. Hypoglycemia
b. Jitteriness
c. Respiratory depression
d. Tachycardia
75. Which symptom would indicate the Baby Alexandra was adapting appropriately to extrauterine life without difficulty?
a. Nasal flaring
b. Light audible grunting

c. Respiratory rate 40 to 60 breaths/minute


d. Respiratory rate 60 to 80 breaths/minute
76. When teaching umbilical cord care for Jennifer a new mother, the nurse Jenny would
include which information?
a. Apply peroxide to the cord with each diaper change
b. Cover the cord with petroleum jelly after bathing
c. Keep the cord dry and open to air
d. Wash the cord with soap and water each day during a tub bath.
77. Nurse John is performing an assessment on a neonate. Which of the following findings is
considered common in the healthy neonate?
a. Simian crease
b. Conjunctival hemorrhage
c. Cystic hygroma
d. Bulging fontanelle
78. Dr. Esteves decides to artificially rupture the membranes of a mother who is on labor.
Following this procedure, the nurse Hazel checks the fetal heart tones for which the following
reasons?
a. To determine fetal well-being.
b. To assess for prolapsed cord
c. To assess fetal position
d. To prepare for an imminent delivery.
79. Which of the following would be least likely to indicate anticipated bonding behaviors by
new parents?
a. The parents willingness to touch and hold the newborn.
b. The parents expression of interest about the size of the newborn.

c. The parents indication that they want to see the newborn.


d. The parents interactions with each other.
80. Following a precipitous delivery, examination of the clients vagina reveals a fourth-degree
laceration. Which of the following would be contraindicated when caring for this client?
a. Applying cold to limit edema during the first 12 to 24 hours.
b. Instructing the client to use two or more peri pads to cushion the area.
c. Instructing the client on the use of sitz baths if ordered.
d. Instructing the client about the importance of perineal (kegel) exercises.
81. A pregnant woman accompanied by her husband, seeks admission to the labor and delivery
area. She states that shes in labor and says she attended the facility clinic for prenatal care.
Which question should the nurse Oliver ask her first?
a. Do you have any chronic illnesses?
b. Do you have any allergies?
c. What is your expected due date?
d. Who will be with you during labor?
82. A neonate begins to gag and turns a dusky color. What should the nurse do first?
a. Calm the neonate.
b. Notify the physician.
c. Provide oxygen via face mask as ordered
d. Aspirate the neonates nose and mouth with a bulb syringe.
83. When a client states that her water broke, which of the following actions would be
inappropriate for the nurse to do?
a. Observing the pooling of straw-colored fluid.
b. Checking vaginal discharge with nitrazine paper.
c. Conducting a bedside ultrasound for an amniotic fluid index.
d. Observing for flakes of vernix in the vaginal discharge.

84. A baby girl is born 8 weeks premature. At birth, she has no spontaneous respirations but is
successfully resuscitated. Within several hours she develops respiratory grunting, cyanosis,
tachypnea, nasal flaring, and retractions. Shes diagnosed with respiratory distress syndrome,
intubated, and placed on a ventilator. Which nursing action should be included in the babys
plan of care to prevent retinopathy of prematurity?
a. Cover his eyes while receiving oxygen.
b. Keep her body temperature low.
c. Monitor partial pressure of oxygen (Pao2) levels.
d. Humidify the oxygen.
85. Which of the following is normal newborn calorie intake?
a. 110 to 130 calories per kg.
b. 30 to 40 calories per lb of body weight.
c. At least 2 ml per feeding
d. 90 to 100 calories per kg
86. Nurse John is knowledgeable that usually individual twins will grow appropriately and at
the same rate as singletons until how many weeks?
a. 16 to 18 weeks
b. 18 to 22 weeks
c. 30 to 32 weeks
d. 38 to 40 weeks
87. Which of the following classifications applies to monozygotic twins for whom the cleavage
of the fertilized ovum occurs more than 13 days after fertilization?
a. conjoined twins
b. diamniotic dichorionic twins
c. diamniotic monochorionic twin
d. monoamniotic monochorionic twins

88. Tyra experienced painless vaginal bleeding has just been diagnosed as having a placenta
previa. Which of the following procedures is usually performed to diagnose placenta previa?
a. Amniocentesis
b. Digital or speculum examination
c. External fetal monitoring
d. Ultrasound
89. Nurse Arnold knows that the following changes in respiratory functioning during
pregnancy is considered normal:
a. Increased tidal volume
b. Increased expiratory volume
c. Decreased inspiratory capacity
d. Decreased oxygen consumption
90. Emily has gestational diabetes and it is usually managed by which of the following therapy?
a. Diet
b. Long-acting insulin
c. Oral hypoglycemic
d. Oral hypoglycemic drug and insulin
91. Magnesium sulfate is given to Jemma with preeclampsia to prevent which of the following
condition?
a. Hemorrhage
b. Hypertension
c. Hypomagnesemia
d. Seizure
92. Cammile with sickle cell anemia has an increased risk for having a sickle cell crisis during
pregnancy. Aggressive management of a sickle cell crisis includes which of the following
measures?

a. Antihypertensive agents
b. Diuretic agents
c. I.V. fluids
d. Acetaminophen (Tylenol) for pain
93. Which of the following drugs is the antidote for magnesium toxicity?
a. Calcium gluconate (Kalcinate)
b. Hydralazine (Apresoline)
c. Naloxone (Narcan)
d. Rho (D) immune globulin (RhoGAM)
94. Marlyn is screened for tuberculosis during her first prenatal visit. An intradermal injection
of purified protein derivative (PPD) of the tuberculin bacilli is given. She is considered to have
a positive test for which of the following results?
a. An indurated wheal under 10 mm in diameter appears in 6 to 12 hours.
b. An indurated wheal over 10 mm in diameter appears in 48 to 72 hours.
c. A flat circumcised area under 10 mm in diameter appears in 6 to 12 hours.
d. A flat circumcised area over 10 mm in diameter appears in 48 to 72 hours.
95. Dianne, 24 year-old is 27 weeks pregnant arrives at her physicians office with complaints
of fever, nausea, vomiting, malaise, unilateral flank pain, and costovertebral angle tenderness.
Which of the following diagnoses is most likely?
a. Asymptomatic bacteriuria
b. Bacterial vaginosis
c. Pyelonephritis
d. Urinary tract infection (UTI)
96. Rh isoimmunization in a pregnant client develops during which of the following conditions?
a. Rh-positive maternal blood crosses into fetal blood, stimulating fetal antibodies.
b. Rh-positive fetal blood crosses into maternal blood, stimulating maternal antibodies.

c. Rh-negative fetal blood crosses into maternal blood, stimulating maternal antibodies.
d. Rh-negative maternal blood crosses into fetal blood, stimulating fetal antibodies.
97. To promote comfort during labor, the nurse John advises a client to assume certain
positions and avoid others. Which position may cause maternal hypotension and fetal hypoxia?
a. Lateral position
b. Squatting position
c. Supine position
d. Standing position
98. Celeste who used heroin during her pregnancy delivers a neonate. When assessing the
neonate, the nurse Lynnette expects to find:
a. Lethargy 2 days after birth.
b. Irritability and poor sucking.
c. A flattened nose, small eyes, and thin lips.
d. Congenital defects such as limb anomalies.
99. The uterus returns to the pelvic cavity in which of the following time frames?
a. 7th to 9th day postpartum.
b. 2 weeks postpartum.
c. End of 6th week postpartum.
d. When the lochia changes to alba.
100. Maureen, a primigravida client, age 20, has just completed a difficult, forceps-assisted
delivery of twins. Her labor was unusually long and required oxytocin (Pitocin) augmentation.
The nurse whos caring for her should stay alert for:
a. Uterine inversion
b. Uterine atony
c. Uterine involution
d. Uterine discomfort

Answers & Rationale


Gauge your performance by counter checking your answers to the answers below. Learn more about
the question by reading the rationale. If you have any disputes or questions, please direct them to the
comments section.
1. Answer: (A) Inevitable
Rationale: An inevitable abortion is termination of pregnancy that cannot be prevented. Moderate to
severe bleeding with mild cramping and cervical dilation would be noted in this type of abortion.
2. Answer: (B) History of syphilis
Rationale: Maternal infections such as syphilis, toxoplasmosis, and rubella are causes of spontaneous
abortion.
3. Answer: (C) Monitoring apical pulse
Rationale: Nursing care for the client with a possible ectopic pregnancy is focused on preventing or
identifying hypovolemic shock and controlling pain. An elevated pulse rate is an indicator of shock.
4. Answer: (B) Increased caloric intake
Rationale: Glucose crosses the placenta, but insulin does not. High fetal demands for glucose,
combined with the insulin resistance caused by hormonal changes in the last half of pregnancy can
result in elevation of maternal blood glucose levels. This increases the mothers demand for insulin
and is referred to as the diabetogenic effect of pregnancy.
5. Answer: (A) Excessive fetal activity.
Rationale: The most common signs and symptoms of hydatidiform mole includes elevated levels of
human chorionic gonadotropin, vaginal bleeding, larger than normal uterus for gestational age,
failure to detect fetal heart activity even with sensitive instruments, excessive nausea and vomiting,
and early development of pregnancy-induced hypertension. Fetal activity would not be noted.
6. Answer: (B) Absent patellar reflexes
Rationale: Absence of patellar reflexes is an indicator of hypermagnesemia, which requires
administration of calcium gluconate.

7. Answer: (C) Presenting part in 2 cm below the plane of the ischial spines.
Rationale: Fetus at station plus two indicates that the presenting part is 2 cm below the plane of the
ischial spines.
8. Answer: (A) Contractions every 1 minutes lasting 70-80 seconds.
Rationale: Contractions every 1 minutes lasting 70-80 seconds, is indicative of hyperstimulation of
the uterus, which could result in injury to the mother and the fetus if Pitocin is not discontinued.
9. Answer: (C) EKG tracings
Rationale: A potential side effect of calcium gluconate administration is cardiac arrest. Continuous
monitoring of cardiac activity (EKG) throught administration of calcium gluconate is an essential
part of care.
10. Answer: (D) First low transverse caesarean was for breech position. Fetus in this pregnancy
is in a vertex presentation.
Rationale: This type of client has no obstetrical indication for a caesarean section as she did with her
first caesarean delivery.
11. Answer: (A) Talk to the mother first and then to the toddler.
Rationale: When dealing with a crying toddler, the best approach is to talk to the mother and ignore
the toddler first. This approach helps the toddler get used to the nurse before she attempts any
procedures. It also gives the toddler an opportunity to see that the mother trusts the nurse.
12. Answer: (D) Place the infants arms in soft elbow restraints.
Rationale: Soft restraints from the upper arm to the wrist prevent the infant from touching her lip but
allow him to hold a favorite item such as a blanket. Because they could damage the operative site,
such as objects as pacifiers, suction catheters, and small spoons shouldnt be placed in a babys
mouth after cleft repair. A baby in a prone position may rub her face on the sheets and traumatize the
operative site. The suture line should be cleaned gently to prevent infection, which could interfere
with healing and damage the cosmetic appearance of the repair.
13. Answer: (B) Allow the infant to rest before feeding.
Rationale: Because feeding requires so much energy, an infant with heart failure should rest before
feeding.

14. Answer: (C) Iron-rich formula only.


Rationale: The infants at age 5 months should receive iron-rich formula and that they shouldnt
receive solid food, even baby food until age 6 months.
15. Answer: (D) 10 months
Rationale: A 10 month old infant can sit alone and understands object permanence, so he would look
for the hidden toy. At age 4 to 6 months, infants cant sit securely alone. At age 8 months, infants can
sit securely alone but cannot understand the permanence of objects.
16. Answer: (D) Public health nursing focuses on preventive, and not curative, services.
Rationale: The catchments area in PHN consists of a residential community, many of whom are well
individuals who have greater need for preventive rather than curative services.
17. Answer: (B) Efficiency
Rationale: Efficiency is determining whether the goals were attained at the least possible cost.
18. Answer: (D) Rural Health Unit
Rationale: R.A. 7160 devolved basic health services to local government units (LGUs ). The public
health nurse is an employee of the LGU.
19. Answer: (A) Mayor
Rationale: The local executive serves as the chairman of the Municipal Health Board.
20. Answer: (A) 1
Rationale: Each rural health midwife is given a population assignment of about 5,000.
21. Answer: (B) Health education and community organizing are necessary in providing
community health services.
Rationale: The community health nurse develops the health capability of people through health
education and community organizing activities.
22. Answer: (B) Measles
Rationale: Presidential Proclamation No. 4 is on the Ligtas Tigdas Program.

23. Answer: (D) Core group formation


Rationale: In core group formation, the nurse is able to transfer the technology of community
organizing to the potential or informal community leaders through a training program.
24. Answer: (D) To maximize the communitys resources in dealing with health problems.
Rationale: Community organizing is a developmental service, with the goal of developing the
peoples self-reliance in dealing with community health problems. A, B and C are objectives of
contributory objectives to this goal.
25. Answer: (D) Terminal
Rationale: Tertiary prevention involves rehabilitation, prevention of permanent disability and
disability limitation appropriate for convalescents, the disabled, complicated cases and the terminally
ill (those in the terminal stage of a disease).
26. Answer: (A) Intrauterine fetal death.
Rationale: Intrauterine fetal death, abruptio placentae, septic shock, and amniotic fluid embolism
may trigger normal clotting mechanisms; if clotting factors are depleted, DIC may occur. Placenta
accreta, dysfunctional labor, and premature rupture of the membranes arent associated with DIC.
27. Answer: (C) 120 to 160 beats/minute
Rationale: A rate of 120 to 160 beats/minute in the fetal heart appropriate for filling the heart with
blood and pumping it out to the system.
28. Answer: (A) Change the diaper more often.
Rationale: Decreasing the amount of time the skin comes contact with wet soiled diapers will help
heal the irritation.
29. Answer: (D) Endocardial cushion defect
Rationale: Endocardial cushion defects are seen most in children with Down syndrome, asplenia, or
polysplenia.
30. Answer: (B) Decreased urine output
Rationale: Decreased urine output may occur in clients receiving I.V. magnesium and should be

monitored closely to keep urine output at greater than 30 ml/hour, because magnesium is excreted
through the kidneys and can easily accumulate to toxic levels.
31. Answer: (A) Menorrhagia
Rationale: Menorrhagia is an excessive menstrual period.
32. Answer: (C) Blood typing
Rationale: Blood type would be a critical value to have because the risk of blood loss is always a
potential complication during the labor and delivery process. Approximately 40% of a womans
cardiac output is delivered to the uterus, therefore, blood loss can occur quite rapidly in the event of
uncontrolled bleeding.
33. Answer: (D) Physiologic anemia
Rationale: Hemoglobin values and hematocrit decrease during pregnancy as the increase in plasma
volume exceeds the increase in red blood cell production.
34. Answer: (D) A 2 year old infant with stridorous breath sounds, sitting up in his mothers
arms and drooling.
Rationale: The infant with the airway emergency should be treated first, because of the risk of
epiglottitis.
35. Answer: (A) Placenta previa
Rationale: Placenta previa with painless vaginal bleeding.
36. Answer: (D) Early in the morning
Rationale: Based on the nurses knowledge of microbiology, the specimen should be collected early
in the morning. The rationale for this timing is that, because the female worm lays eggs at night
around the perineal area, the first bowel movement of the day will yield the best results. The specific
type of stool specimen used in the diagnosis of pinworms is called the tape test.
37. Answer: (A) Irritability and seizures
Rationale: Lead poisoning primarily affects the CNS, causing increased intracranial pressure. This
condition results in irritability and changes in level of consciousness, as well as seizure disorders,
hyperactivity, and learning disabilities.

38. Answer: (D) I really need to use the diaphragm and jelly most during the middle of my
menstrual cycle.
Rationale: The woman must understand that, although the fertile period is approximately midcycle, hormonal variations do occur and can result in early or late ovulation. To be effective, the
diaphragm should be inserted before every intercourse.
39. Answer: (C) Restlessness
Rationale: In a child, restlessness is the earliest sign of hypoxia. Late signs of hypoxia in a child are
associated with a change in color, such as pallor or cyanosis.
40. Answer: (B) Walk one step ahead, with the childs hand on the nurses elbow.
Rationale: This procedure is generally recommended to follow in guiding a person who is blind.
41. Answer: (A) Loud, machinery-like murmur.
Rationale: A loud, machinery-like murmur is a characteristic finding associated with patent ductus
arteriosus.
42. Answer: (C) More oxygen, and the newborns metabolic rate increases.
Rationale: When cold, the infant requires more oxygen and there is an increase in metabolic rate.
Non-shievering thermogenesis is a complex process that increases the metabolic rate and rate of
oxygen consumption, therefore, the newborn increase heat production.
43. Answer: (D) Voided
Rationale: Before administering potassium I.V. to any client, the nurse must first check that the
clients kidneys are functioning and that the client is voiding. If the client is not voiding, the nurse
should withhold the potassium and notify the physician.
44. Answer: (c) Laundry detergent
Rationale: Eczema or dermatitis is an allergic skin reaction caused by an offending allergen. The
topical allergen that is the most common causative factor is laundry detergent.
45. Answer: (A) 6 inches
Rationale: This distance allows for easy flow of the formula by gravity, but the flow will be slow
enough not to overload the stomach too rapidly.

46. Answer: (A) The older one gets, the more susceptible he becomes to the complications of
chickenpox.
Rationale: Chickenpox is usually more severe in adults than in children. Complications, such as
pneumonia, are higher in incidence in adults.
47. Answer: (D) Consult a physician who may give them rubella immunoglobulin.
Rationale: Rubella vaccine is made up of attenuated German measles viruses. This is contraindicated
in pregnancy. Immune globulin, a specific prophylactic against German measles, may be given to
pregnant women.
48. Answer: (A) Contact tracing
Rationale: Contact tracing is the most practical and reliable method of finding possible sources of
person-to-person transmitted infections, such as sexually transmitted diseases.
49. Answer: (D) Leptospirosis
Rationale: Leptospirosis is transmitted through contact with the skin or mucous membrane with
water or moist soil contaminated with urine of infected animals, like rats.
50. Answer: (B) Cholera
Rationale: Passage of profuse watery stools is the major symptom of cholera. Both amoebic and
bacillary dysentery are characterized by the presence of blood and/or mucus in the stools. Giardiasis
is characterized by fat malabsorption and, therefore, steatorrhea.
51. Answer: (A) Haemophilus influenzae
Rationale: Hemophilus meningitis is unusual over the age of 5 years. In developing countries, the
peak incidence is in children less than 6 months of age. Morbillivirus is the etiology of measles.
Streptococcus pneumoniae and Neisseria meningitidis may cause meningitis, but age distribution is
not specific in young children.
52. Answer: (B) Buccal mucosa
Rationale: Kopliks spot may be seen on the mucosa of the mouth or the throat.

53. Answer: (A) 3 seconds


Rationale: Adequate blood supply to the area allows the return of the color of the nailbed within 3
seconds.
54. Answer: (B) Severe dehydration
Rationale: The order of priority in the management of severe dehydration is as follows: intravenous
fluid therapy, referral to a facility where IV fluids can be initiated within 30 minutes, Oresol or
nasogastric tube. When the foregoing measures are not possible or effective, than urgent referral to
the hospital is done.
55. Answer: (A) 45 infants
Rationale: To estimate the number of infants, multiply total population by 3%.
56. Answer: (A) DPT
Rationale: DPT is sensitive to freezing. The appropriate storage temperature of DPT is 2 to 8 C only.
OPV and measles vaccine are highly sensitive to heat and require freezing. MMR is not an
immunization in the Expanded Program on Immunization.
57. Answer: (C) Proper use of sanitary toilets
Rationale: The ova of the parasite get out of the human body together with feces. Cutting the cycle at
this stage is the most effective way of preventing the spread of the disease to susceptible hosts.
58. Answer: (D) 5 skin lesions, positive slit skin smear
Rationale: A multibacillary leprosy case is one who has a positive slit skin smear and at least 5 skin
lesions.
59. Answer: (C) Thickened painful nerves
Rationale: The lesion of leprosy is not macular. It is characterized by a change in skin color (either
reddish or whitish) and loss of sensation, sweating and hair growth over the lesion. Inability to close
the eyelids (lagophthalmos) and sinking of the nosebridge are late symptoms.
60. Answer: (B) Ask where the family resides.
Rationale: Because malaria is endemic, the first question to determine malaria risk is where the
clients family resides. If the area of residence is not a known endemic area, ask if the child had

traveled within the past 6 months, where she was brought and whether she stayed overnight in that
area.
61. Answer: (A) Inability to drink
Rationale: A sick child aged 2 months to 5 years must be referred urgently to a hospital if he/she has
one or more of the following signs: not able to feed or drink, vomits everything, convulsions,
abnormally sleepy or difficult to awaken.
62. Answer: (A) Refer the child urgently to a hospital for confinement.
Rationale: Baggy pants is a sign of severe marasmus. The best management is urgent referral to a
hospital.
63. Answer: (D) Let the child rest for 10 minutes then continue giving Oresol more slowly.
Rationale: If the child vomits persistently, that is, he vomits everything that he takes in, he has to be
referred urgently to a hospital. Otherwise, vomiting is managed by letting the child rest for 10
minutes and then continuing with Oresol administration. Teach the mother to give Oresol more
slowly.
64. Answer: (B) Some dehydration
Rationale: Using the assessment guidelines of IMCI, a child (2 months to 5 years old) with diarrhea
is classified as having SOME DEHYDRATION if he shows 2 or more of the following signs: restless
or irritable, sunken eyes, the skin goes back slow after a skin pinch.
65. Answer: (C) Normal
Rationale: In IMCI, a respiratory rate of 50/minute or more is fast breathing for an infant aged 2 to
12 months.
66. Answer: (A) 10 years
Rationale: The baby will have passive natural immunity by placental transfer of antibodies. The
mother will have active artificial immunity lasting for about 10 years. 5 doses will give the mother
lifetime protection.
67. Answer: (B) 4 hours
Rationale: While the unused portion of other biologicals in EPI may be given until the end of the day,

only BCG is discarded 4 hours after reconstitution. This is why BCG immunization is scheduled only
in the morning.
68. Answer: (B) 6 months
Rationale: After 6 months, the babys nutrient needs, especially the babys iron requirement, can no
longer be provided by mothers milk alone.
69. Answer: (C) 24 weeks
Rationale: At approximately 23 to 24 weeks gestation, the lungs are developed enough to sometimes
maintain extrauterine life. The lungs are the most immature system during the gestation period.
Medical care for premature labor begins much earlier (aggressively at 21 weeks gestation)
70. Answer: (B) Sudden infant death syndrome (SIDS)
Rationale: Supine positioning is recommended to reduce the risk of SIDS in infancy. The risk of
aspiration is slightly increased with the supine position. Suffocation would be less likely with an
infant supine than prone and the position for GER requires the head of the bed to be elevated.
71. Answer: (C) Decreased temperature
Rationale: Temperature instability, especially when it results in a low temperature in the neonate,
may be a sign of infection. The neonates color often changes with an infection process but generally
becomes ashen or mottled. The neonate with an infection will usually show a decrease in activity
level or lethargy.
72. Answer: (D) Polycythemia probably due to chronic fetal hypoxia
Rationale: The small-for-gestation neonate is at risk for developing polycythemia during the
transitional period in an attempt to decrease hypoxia. The neonates are also at increased risk for
developing hypoglycemia and hypothermia due to decreased glycogen stores.
73. Answer: (C) Desquamation of the epidermis
Rationale: Postdate fetuses lose the vernix caseosa, and the epidermis may become desquamated.
These neonates are usually very alert. Lanugo is missing in the postdate neonate.
74. Answer: (C) Respiratory depression
Rationale: Magnesium sulfate crosses the placenta and adverse neonatal effects are respiratory

depression, hypotonia, and bradycardia. The serum blood sugar isnt affected by magnesium sulfate.
The neonate would be floppy, not jittery.
75. Answer: (C) Respiratory rate 40 to 60 breaths/minute
Rationale: A respiratory rate 40 to 60 breaths/minute is normal for a neonate during the transitional
period. Nasal flaring, respiratory rate more than 60 breaths/minute, and audible grunting are signs of
respiratory distress.
76. Answer: (C) Keep the cord dry and open to air
Rationale: Keeping the cord dry and open to air helps reduce infection and hastens drying. Infants
arent given tub bath but are sponged off until the cord falls off. Petroleum jelly prevents the cord
from drying and encourages infection. Peroxide could be painful and isnt recommended.
77. Answer: (B) Conjunctival hemorrhage
Rationale: Conjunctival hemorrhages are commonly seen in neonates secondary to the cranial
pressure applied during the birth process. Bulging fontanelles are a sign of intracranial pressure.
Simian creases are present in 40% of the neonates with trisomy 21. Cystic hygroma is a neck mass
that can affect the airway.
78. Answer: (B) To assess for prolapsed cord
Rationale: After a client has an amniotomy, the nurse should assure that the cord isnt prolapsed and
that the baby tolerated the procedure well. The most effective way to do this is to check the fetal
heart rate. Fetal well-being is assessed via a nonstress test. Fetal position is determined by vaginal
examination. Artificial rupture of membranes doesnt indicate an imminent delivery.
79. Answer: (D) The parents interactions with each other.
Rationale: Parental interaction will provide the nurse with a good assessment of the stability of the
familys home life but it has no indication for parental bonding. Willingness to touch and hold the
newborn, expressing interest about the newborns size, and indicating a desire to see the newborn are
behaviors indicating parental bonding.
80. Answer: (B) Instructing the client to use two or more peri pads to cushion the area
Rationale: Using two or more peripads would do little to reduce the pain or promote perineal healing.

Cold applications, sitz baths, and Kegel exercises are important measures when the client has a
fourth-degree laceration.
81. Answer: (C) What is your expected due date?
Rationale: When obtaining the history of a client who may be in labor, the nurses highest priority is
to determine her current status, particularly her due date, gravidity, and parity. Gravidity and parity
affect the duration of labor and the potential for labor complications. Later, the nurse should ask
about chronic illnesses, allergies, and support persons.
82. Answer: (D) Aspirate the neonates nose and mouth with a bulb syringe.
Rationale: The nurses first action should be to clear the neonates airway with a bulb syringe. After
the airway is clear and the neonates color improves, the nurse should comfort and calm the neonate.
If the problem recurs or the neonates color doesnt improve readily, the nurse should notify the
physician. Administering oxygen when the airway isnt clear would be ineffective.
83. Answer: (C) Conducting a bedside ultrasound for an amniotic fluid index.
Rationale: It isnt within a nurses scope of practice to perform and interpret a bedside ultrasound
under these conditions and without specialized training. Observing for pooling of straw-colored fluid,
checking vaginal discharge with nitrazine paper, and observing for flakes of vernix are appropriate
assessments for determining whether a client has ruptured membranes.
84. Answer: (C) Monitor partial pressure of oxygen (Pao2) levels.
Rationale: Monitoring PaO2 levels and reducing the oxygen concentration to keep PaO2 within
normal limits reduces the risk of retinopathy of prematurity in a premature infant receiving oxygen.
Covering the infants eyes and humidifying the oxygen dont reduce the risk of retinopathy of
prematurity. Because cooling increases the risk of acidosis, the infant should be kept warm so that his
respiratory distress
isnt aggravated.
85. Answer: (A) 110 to 130 calories per kg.
Rationale: Calories per kg is the accepted way of determined appropriate nutritional intake for a
newborn. The recommended calorie requirement is 110 to 130 calories per kg of newborn body

weight. This level will maintain a consistent blood glucose level and provide enough calories for
continued growth and development.
86. Answer: (C) 30 to 32 weeks
Rationale: Individual twins usually grow at the same rate as singletons until 30 to 32 weeks
gestation, then twins dont gain weight as rapidly as singletons of the same gestational age. The
placenta can no longer keep pace with the nutritional requirements of both fetuses after 32 weeks, so
theres some growth retardation in twins if they remain in utero at 38 to 40 weeks.
87. Answer: (A) conjoined twins
Rationale: The type of placenta that develops in monozygotic twins depends on the time at which
cleavage of the ovum occurs. Cleavage in conjoined twins occurs more than 13 days after
fertilization. Cleavage that occurs less than 3 day after fertilization results in diamniotic dichorionic
twins. Cleavage that occurs between days 3 and 8 results in diamniotic monochorionic twins.
Cleavage that occurs between days 8 to 13 result in monoamniotic monochorionic twins.
88. Answer: (D) Ultrasound
Rationale: Once the mother and the fetus are stabilized, ultrasound evaluation of the placenta should
be done to determine the cause of the bleeding. Amniocentesis is contraindicated in placenta previa.
A digital or speculum examination shouldnt be done as this may lead to severe bleeding or
hemorrhage. External fetal monitoring wont detect a placenta previa, although it will detect fetal
distress, which may result from blood loss or placenta separation.
89. Answer: (A) Increased tidal volume
Rationale: A pregnant client breathes deeper, which increases the tidal volume of gas moved in and
out of the respiratory tract with each breath. The expiratory volume and residual volume decrease as
the pregnancy progresses. The inspiratory capacity increases during pregnancy. The increased oxygen
consumption in the pregnant client is 15% to 20% greater than in the nonpregnant state.
90. Answer: (A) Diet
Rationale: Clients with gestational diabetes are usually managed by diet alone to control their
glucose intolerance. Oral hypoglycemic drugs are contraindicated in pregnancy. Long-acting insulin
usually isnt needed for blood glucose control in the client with gestational diabetes.

91. Answer: (D) Seizure


Rationale: The anticonvulsant mechanism of magnesium is believes to depress seizure foci in the
brain and peripheral neuromuscular blockade. Hypomagnesemia isnt a complication of
preeclampsia. Antihypertensive drug other than magnesium are preferred for sustained hypertension.
Magnesium doesnt help prevent hemorrhage in preeclamptic clients.
92. Answer: (C) I.V. fluids
Rationale: A sickle cell crisis during pregnancy is usually managed by exchange transfusion oxygen,
and L.V. Fluids. The client usually needs a stronger analgesic than acetaminophen to control the pain
of a crisis. Antihypertensive drugs usually arent necessary. Diuretic wouldnt be used unless fluid
overload resulted.
93. Answer: (A) Calcium gluconate (Kalcinate)
Rationale: Calcium gluconate is the antidote for magnesium toxicity. Ten milliliters of 10% calcium
gluconate is given L.V. push over 3 to 5 minutes. Hydralazine is given for sustained elevated blood
pressure in preeclamptic clients. Rho (D) immune globulin is given to women with Rh-negative
blood to prevent antibody formation from RH-positive conceptions. Naloxone is used to correct
narcotic toxicity.
94. Answer: (B) An indurated wheal over 10 mm in diameter appears in 48 to 72 hours.
Rationale: A positive PPD result would be an indurated wheal over 10 mm in diameter that appears in
48 to 72 hours. The area must be a raised wheal, not a flat circumcised area to be considered positive.
95. Answer: (C) Pyelonephritis
Rational: The symptoms indicate acute pyelonephritis, a serious condition in a pregnant client. UTI
symptoms include dysuria, urgency, frequency, and suprapubic tenderness. Asymptomatic bacteriuria
doesnt cause symptoms. Bacterial vaginosis causes milky white vaginal discharge but no systemic
symptoms.
96. Answer: (B) Rh-positive fetal blood crosses into maternal blood, stimulating maternal
antibodies.
Rationale: Rh isoimmunization occurs when Rh-positive fetal blood cells cross into the maternal

circulation and stimulate maternal antibody production. In subsequent pregnancies with Rh-positive
fetuses, maternal antibodies may cross back into the fetal circulation and destroy the fetal blood cells.
97. Answer: (C) Supine position
Rationale: The supine position causes compression of the clients aorta and inferior vena cava by the
fetus. This, in turn, inhibits maternal circulation, leading to maternal hypotension and, ultimately,
fetal hypoxia. The other positions promote comfort and aid labor progress. For instance, the lateral,
or side-lying, position improves maternal and fetal circulation, enhances comfort, increases maternal
relaxation, reduces muscle tension, and eliminates pressure points. The squatting position promotes
comfort by taking advantage of gravity. The standing position also takes advantage of gravity and
aligns the fetus with the pelvic angle.
98. Answer: (B) Irritability and poor sucking.
Rationale: Neonates of heroin-addicted mothers are physically dependent on the drug and experience
withdrawal when the drug is no longer supplied. Signs of heroin withdrawal include irritability, poor
sucking, and restlessness. Lethargy isnt associated with neonatal heroin addiction. A flattened nose,
small eyes, and thin lips are seen in infants with fetal alcohol syndrome. Heroin use during
pregnancy hasnt been linked to specific congenital anomalies.
99. Answer: (A) 7th to 9th day postpartum
Rationale: The normal involutional process returns the uterus to the pelvic cavity in 7 to 9 days. A
significant involutional complication is the failure of the uterus to return to the pelvic cavity within
the prescribed time period. This is known as subinvolution.
100. Answer: (B) Uterine atony
Rationale: Multiple fetuses, extended labor stimulation with oxytocin, and traumatic delivery
commonly are associated with uterine atony, which may lead to postpartum hemorrhage. Uterine
inversion may precede or follow delivery and commonly results from apparent excessive traction on
the umbilical cord and attempts to deliver the placenta manually. Uterine involution and some uterine
discomfort are normal after delivery.

Situation : Nurse Macarena is a Family Planning and Infertility Nurse Specialist and currently attends
to FAMILY PLANNING CLIENTS AND INFERTILE COUPLES. The following conditions pertain
to meeting the nursing needs of this particular population group.
1. Daphne, 19 years old, asks nurse Macarena how can pregnancy be prevented through tubal
ligation. Which would be the best answer?
A. Prostaglandins released from the cut fallopian tubes will lead to permanent closure of the vagina.
B. Sperm can not enter the uterus because the cervical entrance is blocked.
C. Sperm can no longer reach the ova, because the fallopian tubes are blocked
D. The ovary no longer releases ova as there is no where for them to go.
2. The Stevens are a couple undergoing testing for infertility. Infertility is said to exist when:
A. A woman has no uterus
B. A woman has no children
C. A couple has been trying to conceive for 1 year
D. A couple has wanted a child for 6 months
3. Another client named Cindy is diagnosed as having endometriosis. This condition interferes
with fertility because:
A. Endometrial implants can block the fallopian tubes
B. The uterine cervix becomes inflamed and swollen
C. The endometrial lining becomes inflamed leading to narrowing of the cervix.
D. Inflammation of the endometrium causes release of substance P which kills the sperm.
4. Cindy submits herself to Fatima Medical Center and is scheduled to have
a hysterosalpingogram. Which of the following instructions would you give her regarding this
procedure?
A. Menstruation will be irregular for few months as an effect of the dye but it is just normal
B. The sonogram of the uterus will reveal any tumors present
C. The women may experience some itchiness in the vagina as an after effect.
D. Cramping may be felt when the dye is inserted

5. Cindys cousin on the other hand, knowing nurse Macarenas specialization asks what
artificial insemination by donor entails. Which would be your best answer if you were Nurse
Macarena?
A. Donor sperm are introduced vaginally into the uterus or cervix
B. Donor sperm are injected intra-abdominally into each ovary
C. Artificial sperm are injected vaginally to test tubal patency
D. The husbands sperm is administered intravenously weekly
6. Pain in geriatric clients require careful assessment because they:
A. experienced reduce sensory perception
B. have increased sensory perception
C. are expected to experience chronic pain
D. have a increased pain sensitivity
7. Administration of aminoglycosides to the older persons requires careful patient assessment
because older people:
A. are more sensitive to drugs
B. have increased hepatic, renal and gastrointestinal function
C. have increased sensory perception
D. mobilize drugs more rapidly
8. Elder clients are often at risk of having impaired skin integrity. One factor is that they often
experience urinary incontinence. The elderly patient is at higher risk for urinary incontinence
because of:
A. increased glomerular filtration
B. decrease elasticity of blood vessels
C. decreased bladder capacity
D. dilated urethra
9. Which of the following is the MOST COMMON sign of infection among the elderly?

A. decreased breath sounds with crackles


B. Increase body temperature
C. pain
D. Restlessness, confusion, irritability
10. Prioritization is important to test a nurses good judgment towards different situations.
Priorities when caring for the elderly trauma patient:
A. circulation, airway, breathing
B. disability (neurologic), airway, breathing
C. airway, breathing, disability (neurologic)
D. airway, breathing, circulation
11. The nurse assessing newborn babies and infants during their hospital stay after birth will
notice which of the following symptoms as a primary manifestation of Achalasia?
A. Olive shaped mass on abdomen
B. Failure to pass meconium during the first 24 to 48 hours after birth
C. The skin turns yellow and then brown over the first 48 hours of life
D. Effortless and non-projectile vomiting
12. A client is 7 months pregnant and has just been diagnosed as having a partial placenta
previa. She is stable and has minimal spotting and is being sent home. Which of these
instructions to the client may indicate a need for further teaching?
A. Maintain bed rest with bathroom privileges
B. Avoid intercourse for three days.
C. Call if contractions occur.
D. Stay on left side as much as possible when lying down.
13. Ms. Anna has been rushed to the hospital with ruptured membrane. Which of the following
should the nurse check first?
A. Check for the presence of infection
B. Assess for Prolapse of the umbilical cord

C. Check the maternal heart rate


D. Assess the color of the amniotic fluid
14. The nurse notes that the infant is wearing a plastic-coated diaper. If a topical medication
were to be prescribed and it were to go on the stomachs or buttocks, the nurse would teach the
caregivers to:
A. avoid covering the area of the topical medication with the diaper
B. avoid the use of clothing on top of the diaper
C. put the diaper on as usual
D. apply an icepack for 5 minutes to the outside of the diaper
15. Which of the following factors is most important in determining the success of relationships
used in delivering nursing care?
A. Type of illness of the client
B. Transference and countertransference
C. Effective communication
D. Personality of the participants
16. Grace sustained a laceration on her leg from automobile accident. Why are lacerations of
lower extremities potentially more serious among pregnant women than other?
A. lacerations can provoke allergic responses due to gonadotropic hormone release
B. a woman is less able to keep the laceration clean because of her fatigue
C. healing is limited during pregnancy so these will not heal until after birth
D. increased bleeding can occur from uterine pressure on leg veins
17. You are the nurse assigned to work with a child with acute glomerulonephritis. By following
the prescribed treatment regimen, the child experiences a remission. You are now checking to
make sure the child does not have a relapse. Which finding would most lead you to the
conclusion that a relapse is happening?
A. Elevated temperature, cough, sore throat, changing complete blood count (CBC) with differential
B. A urine dipstick measurement of 2+ proteinuria or more for 3 days, or the child found to have 3-

4+ proteinuria plus edema.


C. The urine dipstick showing glucose in the urine for 3 days, extreme thirst, increase in urine output,
and a moon face.
D. A temperature of 37.8 degrees (100 degrees F), flank pain, burning frequency, urgency on voiding,
and cloudy urine.
18. The painful phenomenon known as back labor occurs in a client whose fetus in what
position?
A. Brow position
B. Right Occipito-Anterior Position
C. Breech position
D. Left Occipito-Posterior Position
19. Which among the following is the primary focus of prevention of cancer?
A. Elimination of conditions causing cancer
B. Diagnosis and treatment
C. Treatment at early stage
D. Early detection
20. In the prevention and control of cancer, which of the following activities is the most
important function of the community health nurse?
A. Conduct community assemblies.
B. Referral to cancer specialist those clients with symptoms of cancer.
C. Use the nine warning signs of cancer as parameters in our process of detection, control and
treatment modalities.
D. Teach women about proper/correct nutrition.
21. Who among the following are recipients of the secondary level of care for cancer cases?
A. Those under early case detection
B. Those under post case treatment

C. Those scheduled for surgery


D. Those undergoing treatment
22. Who among the following are recipients of the tertiary level of care for cancer cases?
A. Those under early treatment C. Those under early detection
B. Those under supportive care D. Those scheduled for surgery
23. Being a community health nurse, you have the responsibility of participating in protecting
the health of people. Consider this situation: Vendors selling bread with their bare hands. They
receive money with these hands. You do not see them washing their hands. What should you
say/do?
A. Miss, may I get the bread myself because you have not washed your hands
B. All of these
C. Miss, it is better to use a pick up forceps/ bread tong
D. Miss, your hands are dirty. Wash your hands first before getting the bread
24. A female client asks the nurse about the use of a cervical cap. Which statement is correct
regarding the use of the cervical cap?
A. It may affect Pap smear results.
B. It does not need to be fitted by the physician.
C. It does not require the use of spermicide.
D. It must be removed within 24 hours.
25. The major components of the communication process are:
A. Verbal, written and nonverbal
B. Speaker, listener and reply
C. Facial expression, tone of voice and gestures
D. Message, sender, channel, receiver and feedback

26. The school nurse notices a child who is wearing old, dirty, poor-fitting clothes is always
hungry, has no lunch money, and is always tired. When the nurse asks the boy his tiredness, he
talks of playing outside until midnight. The nurse will suspect that this child is:
A. Being raised by a parent of low intelligence quotient (IQ)
B. An orphan
C. A victim of child neglect
D. The victim of poverty
Situation: Milo, a 16-year-old male, has been diagnosed to have AIDS. He worked as entertainer in a
cruise ship.
27. Which method of transmission is common to contract AIDS:
A. Syringe and needles
B. Body fluids
C. Sexual contact
B. Transfusion
28. Causative organism in AIDS is one of the following:
A. Fungus
B. Bacteria
C. retrovirus
D. Parasites
29. You are assigned in a private room of Milo. Which procedure should be of utmost
importance?
A. Alcohol wash
B. Universal precaution
C. Washing Isolation
D. Gloving technique
30. What primary health teaching would you give to Milo?

A. Daily exercise
B. Prevent infection
C. Reverse isolation
D. Proper nutrition
31. Exercise precaution must be taken to protect health worker dealing with the AIDS patients.
Which among these must be done as priority:
A. Boil used syringe and needles
B. Use gloves when handling specimen
C. Label personal belonging
D. Avoid accidental wound
Situation: Michelle is a 6 year old preschooler. She was reported by her sister to have measles but she
is at home because of fever, upper respiratory problem and white sports in her mouth.
32. Rubeola is an Arabic term meaning Red, the rash appears on the skin in invasive stage
prior to eruption behind the ears. As a nurse, your physical examination must determine
complication especially:
A. Otitis media
B. Bronchial pneumonia
C. Inflammatory conjunctiva
D. Membranous laryngitis
33. To render comfort measure is one of the priorities, Which includes care of the skin, eyes,
ears, mouth and nose. To clean the mouth, your antiseptic solution is in some form of which one
below?
A. Water
B. Sulfur
C. Alkaline
D. Salt

34. As a public health nurse, you teach mother and family members the prevention of
complication of measles. Which of the following should be closely watched?
A. Temperature fails to drop
B. Inflammation of the conjunctiva
C. Inflammation of the nasopharynx
D. Ulcerative stomatitis
35. Source of infection of measles is secretion of nose and throat of infection person. Filterable
virus of measles is transmitted by:
A. Water supply
B. Droplet
C. Food ingestion
D. Sexual contact
36. Method of prevention is to avoid exposure to an infected person. Nursing responsibility for
rehabilitation of patient includes the provision of:
A. Terminal disinfection
B. Injection of gamma globulin
C. Immunization
D. Comfort measures
SITUATION: Sexually Transmitted Diseases are important to identify during pregnancy because of
their potential effect on the pregnancy, fetus, or newborn. The following questions pertain to STDs.
37. Ms. Reynaldita is a promiscuous woman in Manila submits herself to the clinic for certain
examinations. She is experiencing vaginal irritation, redness, and a thick cream cheese vaginal
discharge. As a nurse, you will suspect that Ms. Reynaldita is having what disease?
A. Gardnerella Vaginalis
B. Candida Albicans
C. Treponema Pallidum
D. Moniliasis

38. As a knowledgeable nurse, you know that the doctor may prescribe a certain medications
for Ms. Reynaldita. What is the drug of choice for Reynalditas infection?
A. haloperidol
B. miconazole
C. benzathine penicillin
D. metronidazole
39. Based on your learnings, you know that the causative agent of Reynalditas infection is:
A. Monistat Candida
B. Candida Albicans
C. Albopictus Candidiasis
D. Monakiki
40. The microorganism that causes Reynalditas infection is a:
A. Bacteria
B. Protozoa
C. Fungus
D. Virus
41. Another client in the Maternal Clinic was Ms. Celbong. Her doctor examined Ms. Celbongs
vaginal secretions and found out that she has a Trichomoniasis infection. Trichomoniasis is
diagnosed through which of the following method?
A. Vaginal secretions are examined on a wet slide that has been treated with potassium hydroxide.
B. Vaginal speculum is used to obtain secretions from the cervix.
C. A litmus paper is used to test if the vaginal secretions are infected with trichomoniasis.
D. Vaginal secretions are examined on a wet slide treated with zephiran solution.
42. Daphne who is on her first trimester of pregnancy is also infected with trichomoniasis. You
know that the drug of choice for Daphne is:

A. Flagyl
B. Clotrimazole (topical)
C. Monistat
D. Zovirax
43. Syphilis is another infection that may impose risk during pregnancy. Since we are under the
practice of health science, you know that Syphilis is caused by:
A. Treponema Syphilis
B. Neisseria gonorrhoeae
C. Chlamydia Trachomatis
D. Treponema Pallidum
44. What type of microorganism is the causative agent of syphilis?
A. Spirochete
B. Fungus
C. Bacteria
D. Protozoan
45. Under the second level of prevention, you know that one of the focuses of care is the
screening of diseases. What is the screening test for syphilis?
A. VDRL
B. Western blot
C. PSA
D. ELISA
46. Jarisch-Herxheimer reaction may be experienced by the client with syphilis after therapy
with benzathine penicillin G. The characteristic manifestations of Jarisch-Herxheimer reaction
are:
A. Rashes, itchiness, hives and pruritus
B. Confusion, drowsiness and numbness of extremities

C. sudden episode of hypotension, fever, tachycardia, and muscle aches


D. Episodes of nausea and vomiting, with bradypnea and bradycardia
47. A pregnant woman is in the clinic for consultation with regards to STDs. She inquires
about Venereal warts and asks you about its specific lesion appearance. Which of the following
is your correct response to the client?
A. Why are you asking about it? You might be a prostitute woman.
B. The lesions appear as cauliflower like lesions.
C. It appears as pinpoint vesicles surrounded by erythema.
D. The lesions can possibly obstruct the birth canal.
48. Based on your past learnings in communicable diseases, you know that the causative agent
of venereal warts is:
A. Chlamydia Trachomatis
B. Candida Moniliasis
C. Human Papillomavirus
D. Staphylococcus Aureus
49. As a nurse in charge for this woman, you anticipate that the doctor will prescribe what
medication for this type of infection?
A. Podophyllum (Podofin)
B. Flagyl
C. Monistat
D. Trichloroacetic acid
50. Cryocautery may also be used to remove large lesions. The healing period after cryocautery
may be completed in 4-6 weeks but may cause some discomforts to the woman. What measures
can alleviate these discomforts?
A. Kegels Exercise
B. Cool air

C. Topical steroids
D. Sitz baths and lidocaine cream
51. In order to prevent acquiring sexually transmitted diseases, what is the BEST way to
consider?
A. Condom use
B. Withdrawal
C. vasectomy
D. Abstinence
SITUATION: The Gastrointestinal System is responsible for taking in and processing nutrients for all
parts of the body, any problem can quickly affect other body systems and, if not adequately treated,
can affect overall health, growth, and development. The following questions are about
gastrointestinal disorders in a child.
52. Mr. & Mrs. Alcaras brought their son in the hospital for check up. The child has failure to
thrive and was diagnosed with pyloric stenosis. Which among the following statements are the
characteristic manifestations of pyloric stenosis?
A. Vomiting in the early morning
B. Bile containing vomitus immediately after meal
C. sausage shaped mass in the abdomen
D. Projectile vomiting with no bile content
53. The exact cause of pyloric stenosis is unknown, but multifactorial inheritance is the likely
cause. Being knowledgeable about this disease, you know that pyloric stenosis is more common
in which gender?
A. Male
B. Female
C. Incidence is equal for both sexes
D. None of the above

54. To rule out pyloric stenosis, the definitive diagnosis is made by watching the infant drink.
After the infant drinks, what will be the characteristic sign that will describe pyloric stenosis?
A. An olive-size lump can be palpated
B. There is gastric peristaltic waves from left to right across the abdomen
C. A hypertrophied sphincter can be seen on ultrasound.
D. A tingling sensation is felt on the lower extremities
55. Shey a 10 months old infant was admitted to the hospital for severe abdominal pain. The
doctor found out that the distal ileal segment of the childs bowel has invaginated into the
cecum. The nurse will suspect what disease condition?
A. Intussusception
B. Pyloric stenosis
C. Hirschsprungs disease
D. Vaginismus
56. In intussusceptions, children suddenly draw up their legs and cry as if they are in severe
pain and possibly vomit. Another manifestation of such disease is the presence of blood in the
stool. What is the characteristic stool of client with intussusception?
A. Coffee ground
B. Black and Tarry
C. Currant jelly stool
D. Watery stool
57. A 4-year-old child is hospitalized because of persistent vomiting. As a nurse, you must
monitor the child closely for:
A. Diarrhea
B. Metabolic Acidosis
C. Metabolic Alkalosis
D. Hyperactive bowel sounds

58. A nurse is monitoring for signs of dehydration in a 1-year-old child who has been
hospitalized for diarrhea. The nurse prepares to take the childs temperature and avoids which
method of measurement?
A. Tympanic
B. Axillary
C. Rectal
D. Electronic
59. A home care nurse provides instructions to the mother of an infant with cleft palate
regarding feeding. Which statement if made by the mother indicates a need for further
instructions?
A. I will use a nipple with a small hole to prevent choking.
B. I will stimulate sucking by rubbing the nipple on the lower lip.
C. I will allow the infant time to swallow.
D. I will allow the infant to rest frequently to provide time for swallowing what has been placed in
the mouth.
60. An infant has just returned to the nursing unit following a surgical repair of a cleft lip
located at the right side of the lip. The nurse places the infant in which most appropriate
position?
A. On the right side
B. On the left side
C. Prone
D. Supine
61. A clinic nurse reviews the record of an infant seen in the clinic. The nurse notes that a
diagnosis of esophageal atresia with tracheoesophageal fistula (TEF) is suspected. The nurse
expects to note which most likely sign of this condition documented in the record?
A. Severe projectile vomiting
B. Coughing at night time

C. Choking with feedings


D. Incessant crying
SITUATION: Human development is one of the important concepts that a nurse should learn to be
able to deal appropriately with their clients of different developmental stages.
62. Which statement best describes when fertilization occurs?
A. When the spermatozoon passes into the ovum and the nuclei fuse into a single cell.
B. When the ovum is discharged from the ovary near the fimbriated end of the fallopian tube.
C. When the embryo attaches to the uterine wall.
D. When the sperm and ova undergo developmental changes resulting in a reduction in the number of
chromosomes.
63. A pregnant client asks you about fetal development. At approximately what gestational age
does the fetuss single chambered heart begin to pump its own blood cells through main blood
vessels?
A. 10 weeks
B. 8 weeks
C. 5 weeks
D. 3 weeks
64. At 17 weeks gestation, a fetus isnt considered to be ballotable. Ballottement means that:
A. The examiner feels rebound movement of the fetus.
B. The examiner feels fetal movement.
C. The client feels irregular, painless uterine contractions.
D. The client feels fetal movement.
65. Which hormone stimulates the development of the ovum?
A. Follicle stimulating hormone (FSH)
B. Human Chorionic Gonadotropin (HCG)

C. Luteinizing Hormone (LH)


D. Gonadotropin Releasing Hormone (GnRH)
66. How long is the gestational period of a full term pregnancy?
A. Ranging from 245 days to 259 days
B. around 5,554 hours to 5,880 hours
C. More than 294 days
D. Averaging of 266 to 294 days
67. An 18 year old woman in her 18th week of pregnancy is being evaluated. Which positive
sign of pregnancy should the nurse expect to be present?
A. Fetal heart tones detectable by Doppler stethoscope
B. Fetal movement detectable by palpation
C. Visualization of the fetus by ultrasound examination.
D. Fetal heart tones detectable by a fetoscope.
68. During her prenatal visit, a 28 year old client expresses concern about nutrition during
pregnancy. She wants to know what foods she should be eating to ensure the proper growth and
development of her baby. Which step should the nurse take first?
A. Give the client a sample diet plan for a 2,400 calorie diet.
B. Emphasize the importance of avoiding salty and fatty foods.
C. Instruct the client to continue to eat a normal diet.
D. Assess the clients current nutritional status by taking a diet history.
69. A nurse is teaching a class about the reproductive system. She explains that fertilization
most often takes place in the:
A. Ovary
B. Fallopian tubes
C. Uterus
D. vagina

70. A large number of neural tube defects may be prevented if a pregnant woman includes
which supplement in her diet?
A. Vit. A
B. Vit. E
C. Vit. D
D. Vit. B9
71. A 22 year old client is at 20 weeks gestation. She asks the nurse about the development of
her fetus at this stage. Which of the following developments occurs at 20 weeks gestation?
A. The pancreas starts producing insulin and the kidneys produce urine.
B. The fetus follows a regular schedule of turning, sleeping, sucking, and kicking.
C. Swallowing reflex has been mastered, and the fetus sucks its thumb.
D. Surfactant forms in the lungs.
SITUATION: Developing countries such as the Philippines suffer from high infant and child
mortality rates. Thus, as a management to the existing problem, the WHO and UNICEF launched the
IMCI.
72. A 6 month old baby Len was brought to the health center because of fever and cough for 2
days. She weighs 5 kg. Her temperature is 38.5 taken axillary. Further examination revealed
that she has general rashes, her eyes are red and she has mouth ulcers non deep and non
extensive, There was no pus draining from her eyes. Most probably Baby Len has:
a. Severe complicated measles
b. Fever: No MALARIA
c. Very severe febrile disease
d. Measles
e. Measles with eye or mouth complications
73. The dosage of Vit. A supplement given to Baby Len would be:
a. 100,000 IU
b. 10,000 IU

c. 200,000 IU
d. 20,000 IU
74. Using IMCI Chart, this child can be manage with:
a. Treat the child with paracetamol and follow up in 2 days if the fever persist
b. Give the first dose of antibiotic, give Vit. A, apply Gentian Violet for mouth ulcers and refer
urgently to hospital
c. Give100, 000 international units of Vit. A
d. Give 200, 000 international units of Vit. A
e. Give Vit. A, apply Gentian violet for mouth ulcers and follow up in 2 days
75. The following are signs of severe complicated measles:
a. Clouding of the cornea
b. Deep or extensive mouth ulcers
c. Pus draining from the eyes
d. A and b only
e. All of the above
76. If the child is having 2 weeks ear discharges, how would you classify and treat the child:
1. Green
2. Yellow
3. Pink
4. Red
5. Dry the ear by wicking
6. 5 days antibiotic
7. Urgent referral with first dose of antibiotic
a. 4,7
b. 2,5,6
c. 1,5
d. 3,7
e. 2,5

77. The following are treatments for acute ear infections:


a. Dry the ear by wicking
b. Give antibiotics for 5 days
c. Follow up in 5 days
d. A and c only
e. All of the above
78. A child with ear problem should be assessed for the following, except:
a. Ear pain
b. If discharge is present for how long?
c. Ear discharge
d. Is there any fever?
e. None of the above
79. If the child does not have ear problem, using IMCI, what should you do as a nurse?
a. Go to the next question, check for malnutrition
b. Check for ear pain
c. Check for tender swelling behind the ear
d. Check for ear discharge
80. An ear discharge that has been present for more than 14 days can be classified as:
a. Complicated ear infection
b. Acute ear infection
c. Chronic ear infection
d. Mastoiditis
81. An ear discharge that has been present for less than 14 days can be classified as:
a. Complicated ear infection
b. Acute ear infection

c. Chronic ear infection


d. Mastoiditis
82. If the child has severe classification because of ear problem, what would be the best thing
that you should do as a nurse?
a. Dry the ear by wicking
b. Give an antibiotic for 5 days
c. Refer urgently
d. Instruct mother when to return immediately
Situation: Primary Health Care (PHC) is defined by the WHO as essential health care made
universally accessible to individuals, families and communities.
83. The WHO held a meeting in this place where Primary health Care was discussed. What is
this place?
A. Alma Ata
B. Russia
C. Vienna
D. Geneva
Situation: The national objective for maintaining the health of all Filipinos is a primary responsibility
of the DOH.
84. The following are mission of the DOH except:
a. Ensure accessibility
b.Quality of health care
c.Health for all Filipinos
d.Quality of Life of all Filipinos
e. None of the above
85. The basic principles to achieve improvement in health include all BUT:

a. Universal access to basic health services must be ensured


b. The health and nutrition of vulnerable groups must be prioritized
c. Performance of the health sector must be enhanced
d. Support the frontline workers and the local health system
e. None of the above
86. Which of the following is not a primary strategy to achieve health goals:
a. Support of local health system development
b. Development of national standards for health
c. Assurance of health care for all
d. Support the frontline workers
e. None of the above
87. According to the WHO health is:
A. state of complete physical, mental and social well being not merely the absence of disease
B. A science and art of preventing disease and prolonging life
C. A science that deals the optimum level of functioning of the Individual, family and community
D. All of the above
88. Assistance in physical therapy of a trauma patient is a:
A. Primary level of prevention
B. Secondary level of prevention
C. Tertiary level of prevention
D. Specialized level of prevention
89. Local health boards were established at the provincial, city and municipal levels. At the
municipal level,the chairman of the board is the:
A.Rural Health physician
B.Governor
C.Mayor
D.Chairman of the Committee on Health

90. The emphasis of community health nursing is on:


A. Treatment of health problems
B. Preventing health problems and promoting optimum health
C. Identification and assessment of health problems
D. Illness end of the wellness-illness continuum.
91. In asking the mother about her childs problem the following communication skills should
be used except:
a. Use words that the mother understand
b. Give time for the mother to answer the questions
c. Listen attentively
d. Ask checking questions
e. None of the above
92. Which of the following is the principal focus of the CARI program of the Department of
Health?
a. Teach other community health workers how to assess patients
b. Mortality reduction through early detection
c. Teach mothers how to detect signs and where to refer
d. Enhancement of health team capabilities
93. You were able to identify factors that lead to respiratory problems in the community where
your health facility serves. Your primary role therefore in order to reduce morbidity due to
pneumonia is to?
a. Seek assistance and mobilize the BHWs to have a meeting with mothers
b. Refer cases to hospitals
c. Make home visits to sick children
d. Teach mothers how to recognize early signs and symptoms of pneumonia
94. Which of the following is the most important responsibility of a nurse in the prevention of
unnecessary deaths from pneumonia and other severe disease?

a. Weighing of the child


b. Provision of careful assessment
c. Taking of the temperature of the sick child
d. Giving of antibiotics
95. A 4-month-old child was brought to your clinic because of cough and colds. Which of the
following is your primary action?
a. Teach the mother how to count her childs breathing?
b. Refer to the doctor
c. Assess the patient using the chart on management of children with cough
d. Give cotrimoxazole tablet or syrup
e. All of the above
96. In responding to the care concerns to children with severe disease, referral to the hospital is
of the essence especially if the child manifests which of the following?
a. Stopped feeding well
b. Fast breathing
c. Wheezing
d. Difficulty to awaken
SITUATION: Elvira is a 26 year old woman you admit to a birthing room. Shes been having
contractions 45 seconds long and 3 minutes apart for the last 6 hours. She tells you she wants to have
her baby naturally without any analgesia or anesthesia. Her husband is in the Army and assigned
overseas, so he is not with her. Although her sister lives only two blocks from the hospital, Elvira
doesnt want her called. She asks if she can talk to her mother on the telephone instead.
97. Elvira didnt recognize for over an hour that she was in labor. A sign of true labor is:
A. Sudden increase energy from epinephrine release
B. Nagging but constant pain in the lower back.
C. Urinary urgency from increased bladder pressure.
D. Show or release of the cervical mucus plug.
98. Elvira asks you which fetal position and presentation are ideal. Your best answer would be:

A. Right occipitoanterior with full flexion.


B. Left transverse anterior in moderate flexion.
C. Right occipitoposterior with no flexion.
D. Left sacroanterior with full flexion.
99. Elvira is having long and hard uterine contractions. What length of contraction would you
report as abnormal?
A. Any length over 30 seconds.
B. A contraction over 70 seconds in length.
C. A contraction that peaks at 20 seconds.
D. A contraction shorter than 60 seconds.
100. You assess Elviras uterine contractions. In relation to the contraction, when does a late
deceleration begin?
A. Forty-five seconds after the contraction is over.
B. Thirty seconds after the start of a contraction.
C. After every tenth or more contraction.
D. After a typical contraction ends.

Answers
Here are the answers for the exam. Unfortunately, rationales are not given. If you need clarifications
or disputes, please direct them to the comments section and well be glad to give you an explanation.
1.

C. Sperm can no longer reach the ova, because the fallopian tubes are
blocked

2.

C. A couple has been trying to conceive for 1 year

3.

A. Endometrial implants can block the fallopian tubes

4.

D. Cramping may be felt when the dye is inserted

5.

A. Donor sperm are introduced vaginally into the uterus or cervix

6.

A. experienced reduce sensory perception

7.

A. are more sensitive to drugs

8.

C. decreased bladder capacity

9.

D. Restlessness, confusion, irritability

10.

D. airway, breathing, circulation

11.

D. Effortless and non-projectile vomiting

12.

B. Avoid intercourse for three days.

13.

B. Assess for Prolapse of the umbilical cord

14.

A. avoid covering the area of the topical medication with the diaper

15.

C. Effective communication

16.

D. increased bleeding can occur from uterine pressure on leg veins

17.

B. A urine dipstick measurement of 2+ proteinuria or more for 3 days, or

the child found to have 3-4+ proteinuria plus edema.


18.

D. Left Occipito-Posterior Position

19.

A. Elimination of conditions causing cancer

20.

A. Conduct community assemblies.

21.

A. Those under early case detection

22.

B. Those under supportive care

23.

C. Miss, it is better to use a pick up forceps/ bread tong

24.

A. It may affect Pap smear results.

25.

D. Message, sender, channel, receiver and feedback

26.

C. A victim of child neglect

27.

C. Sexual contact

28.

C. retrovirus

29.

B. Universal precaution

30.

B. Prevent infection

31.

B. Use gloves when handling specimen

32.

B. Bronchial pneumonia

33.

D. Salt

34.

C. Inflammation of the nasopharynx

35.

B. Droplet

36.

D. Comfort measures

37.

D. Moniliasis

38.

B. miconazole

39.

B. Candida Albicans

40.

C. Fungus

41.

A. Vaginal secretions are examined on a wet slide that has been treated

with potassium hydroxide.


42.

B. Clotrimazole (topical)

43.

D. Treponema Pallidum

44.

A. Spirochete

45.

A. VDRL

46.

C. sudden episode of hypotension, fever, tachycardia, and muscle aches

47.

B. The lesions appear as cauliflower like lesions.

48.

C. Human Papillomavirus

49.

D. Trichloroacetic acid

50.

D. Sitz baths and lidocaine cream

51.

A. Condom use

52.

D. Projectile vomiting with no bile content

53.

A. Male

54.

B. There is gastric peristaltic waves from left to right across the abdomen

55.

A. Intussusception

56.

C. Currant jelly stool

57.

C. Metabolic Alkalosis

58.

C. Rectal

59.

B. I will stimulate sucking by rubbing the nipple on the lower lip.

60.

B. On the left side

61.

A. Severe projectile vomiting

62.

A. When the spermatozoon passes into the ovum and the nuclei fuse into

a single cell.
63.

A. 10 weeks

64.

A. The examiner feels rebound movement of the fetus.

65.

A. Follicle stimulating hormone (FSH)

66.

D. Averaging of 266 to 294 days

67.

C. Visualization of the fetus by ultrasound examination.

68.

D. Assess the clients current nutritional status by taking a diet history.

69.

B. Fallopian tubes

70.

D. Vit. B9

71.

B. The fetus follows a regular schedule of turning, sleeping, sucking, and

kicking.
72.

E. Measles with eye or mouth complications

73.

A. 100,000 IU

74.

E. Give Vit. A, apply Gentian violet for mouth ulcers and follow up in 2

days
75.

D. A and b only

76.

E. 2,5

77.

D. A and c only

78.

D. Is there any fever?

79.

A. Go to the next question, check for malnutrition

80.

C. Chronic ear infection

81.

B. Acute ear infection

82.

C. Refer urgently

83.

A. Alma Ata

84.

C. Health for all Filipinos

85.

D. Support the frontline workers and the local health system

86.

E. None of the above

87.

A. State of complete physical, mental and social well being not merely the

absence of disease
88.

C. Tertiary level of prevention

89.

C. Mayor

90.

B. Preventing health problems and promoting optimum health

91.

D. Ask checking questions

92.

B. Mortality reduction through early detection

93.

D. Teach mothers how to recognize early signs and symptoms of

pneumonia
94.

B. Provision of careful assessment

95.

C. Assess the patient using the chart on management of children with

cough
96.

D. Difficulty to awaken

97.

D. Show or release of the cervical mucus plug.

98.

A. Right occipitoanterior with full flexion.

99.

B. A contraction over 70 seconds in length.

100. B. Thirty seconds after the start of a contraction.


Situation 1: Mariah is a 31 year old lawyer who has been married for 6 moths. She consults you for
guidance in relation with her menstrual cycle and her desire to get pregnant.
1. She wants to know the length of her menstrual cycle. Her periodic menstrual period is
October 22 to 26. Her LMB is November 21. Which of the following number of days will be
your correct response?
A. 29
B. 28
C. 30
D. 31
2. You advised her to observe and record the signs of ovulation. Which of the following signs
will she likely note down?
1. A 1 degree Fahrenheit rise in basal body temperature
2. Cervical mucus becomes copious and clear
3. One pound increase in weight
4. Mittelschmerz
A. 1,2,4
B. 1,2,3
C. 2,3,4
D. 1,3,4
3. You instruct Mariah to keep record of her basal temperature everyday, which of the
following instructions is incorrect?
A. If coitus has occurred, this should be reflected in the chart
B. It is best to have coitus on the evening following a drop in BBT to become pregnant

C. Temperature should be taken immediately after waking and before getting put of bed
D. BBT is lowest during the secretory phase
4. She reports an increase in BBT on December 16. Which hormone brings about this change in
her BBT?
A. Estrogen
B. Progesterone
C. Gonadootrophine
D. Follicle Stimulating Hormone
5. The following month, Mariah suspects she is pregnant. Her urine is positive for Human
Chorionic Gonadotrophin. Which structure produce HCG?
A. Pituitary Gland
B. Trophoblastic cells of the embryo
C. Uterine deciduas
D. Ovarian follicles
Situation 2: Mariah came back and she is now pregnant.
6. At 5 months gestation, which of the following fetal development would probably be expected:
A. Fetal development are felt by Mariah
B. Vernix caseosa covers the entire body
C. Viable if delivered within this period
D. Braxton Hicks contractions are observed
7. The nurse palpates the abdomen of Mariah. Now at 5 month gestation, what level of the
abdomen can be the fundic height be palpated?
A. Symphysis pubis
B. Midpoint between the umbilicus and the xiphoid process
C. Midpoint between the symphysis pubis the umbilicus
D. Umbilicus

8. She worries about her small breast, thinking that she probably will incapable to breastfeed
her baby. Which of the following responses of the nurse is correct?
A. The size of your breast will not affect your lactation.
B. You can switch to bottle feeding.
C. You can try to have exercise to increase the size of your breast.
D. Manual expression of milk is possible.
9. She tells the nurse that she does not take milk regularly. She claims that she does not want to
gain too much weight during her pregnancy. Which of the following nursing diagnosis is a
priority?
A. Potential self-esteem disturbance related to physiologic changes in pregnancy
B. Ineffective individual coping related to physiologic changes in pregnancy
C. Fear related to the effects of pregnancy
D. Knowledge deficit regarding nutritional requirements pregnancies related to lack of information
sources.
10. Which of the following interventions will likely ensure compliance of Mariah?
A. Incorporate her food preferences that are adequately nutritious in her meal plan.
B. Consistently counsel toward optimum nutritional intake
C. Respect her right to reject dietary information if she chooses
D. Information of the adverse effects of inadequate nutrition to her fetus
Situation 3: Susan is a patient in the clinic where you work. She is inquiring about pregnancy.
11. Susan tells you she is worried because she develops breast later than most of her friends.
Breast development is termed as:
A. Adrenarche
B. Mamarche
C. Thelarche
D. Menarche

12. Kevin, Susans husband, tells you that he is considering vasectomy. After the birth of their
new child. Vasectomy involves the incision of which organ?
A. The testes
B. The vas deferens
C. The epididymis
D. The scrotum
13. On examination, Susan has been found of having a cystocele. A cystocele is:
A. A sebaceous cyst arising from the vulvar fold.
B. Protrusion of intestines into the vagina
C. Prolapse of the uterus into the vagina
D. Herniation of the bladder into the vaginal wall.
14. Susan typically has menstrual cycle of 34 days. She told you she had a coitus on days 8, 10
and 20 of her menstrual cycle. Which is the day on which she is most likely to conceive?
A. 8th day
B. 10th day
C. Day 15
D. Day 20
15. While talking with Susan, 2 new patients arrived and they are covered with large towels
and the nurse noticed that there are many cameraman and news people outside of the OPD.
Upon assessment the nurse noticed that both of them are still nude and the male clients penis
is still inside the female clients vagina and the male client said that I cant pull it.
Vaginismus was your first impression. You know that the psychological cause of Vaginismus is
related to:
A. The male client inserted the penis too deeply that it stimulates vaginal closure
B. The penis was too large thats why the vagina triggered its defense to attempt to close
C. The vagina do not want to be penetrated
D. It is due to learning patterns of the female client where she views sex as bad or sinful.

Situation 4: Overpopulation is one problem in the Philippines that case economic drain. Most
Filipinos are against in legalizing abortion. As a nurse, Mastery of contraception is needed to
contribute to the society and economic growth.
16. Supposed that Dana, 17 years old, tells you she wants to use fertility awareness method of
contraception. How will she determine her fertile days?
A. She will notice that she feels hot as if she has an elevated temperature
B. She should assess whether her cervical mucus is thin colour, clear and watery.
C. She should monitor her emotions fro sudden anger or crying
D. She should assess whether her breast feel sensitive to cool air.
17. Dana chooses to use COC as her family planning method, what is the danger sign of COC
you would ask her to report?
A. A stuffy or runny nose
B. Arthritis like symptoms
C. Slight weight gain
D. Migraine headache
18. Dana asks about subcutaneous implants and she asks how long will these implants be
effective. Your best answer is:
A. One month
B. Twelve month
C. Five years
D. 10 years
19. Dana asks about female condoms. Which of the following is true with regards to female
condoms?
A. The hormone the condom releases might cause mild weight gain.
B. She should insert the condom before any penile penetration
C. She should coat the condom with spermicide before use
D. Female condoms unlike male condoms are reusable.

20. Dana has asked about GIFT procedure. What makes her a good candidate for GIFT?
A. She has patent fallopian tubes, so fertilized ova can be implanted on them.
B. She is RH negative, a necessary stipulation to rule out RH incompatibility.
C. She has normal uterus, so the sperm can be injected through the cervix into it.
D. Her husband is taking sildenafil (Viagra), so all sperms will be motile.
Situation 5 Nurse Lorena is a Family Planning and Infertility Nurse Specialist and currently attends
to FAMILY PLANNING CLIENTS AND INFERTILE COUPLES. The following conditions pertain
to meeting the nursing needs of this particular population group.
21. Dina, 17 years old, asks you how a tubal ligation prevents pregnancy. Which would be the
best answer?
A. Prostaglandins released from the cut fallopian tubes can kill sperm
B. Sperm can not enter the uterus because the cervical entrance is blocked.
C. Sperm can no longer reach the ova, because the fallopian tubes are blocked
D. The ovary no longer releases ova as there is nowhere for them to go.
22. The Dators are a couple undergoing testing for infertility. Infertility is said to exist when:
A. a woman has no uterus
B. a woman has no children
C. a couple has been trying to conceive for 1 year
D. a couple has wanted a child for 6 months
23. Another client named Lilia is diagnosed as having endometriosis. This condition interferes
with fertility because:
A. endometrial implants can block the fallopian tubes
B. the uterine cervix becomes inflamed and swollen
C. the ovaries stop producing adequate estrogen
D. pressure on the pituitary leads to decreased FSH levels

24. Lilia is scheduled to have a hysterosalpingogram. Which of the following instructions would
you give her regarding this procedure?
A. She will not be able to conceive for 3 months after the procedure
B. The sonogram of the uterus will reveal any tumors present
C. Many women experience mild bleeding as an after effect
D. She may feel some cramping when the dye is inserted
25. Lilias cousin on the other hand, knowing nurse Lorenas specialization asks what artificial
insemination by donor entails. Which would be your best answer if you were Nurse Lorena?
A. Donor sperm are introduced vaginally into the uterus or cervix
B. Donor sperm are injected intra-abdominally into each ovary
C. Artificial sperm are injected vaginally to test tubal patency
D. The husbands sperm is administered intravenously weekly
Situation 6: You are assigned to take care of a group of patients across the lifespan.
26. Pain in the elder persons require careful assessment because they:
A. Experienced reduce sensory perception
B. Have increased sensory perception
C. Are expected to experience chronic pain
D. Have decreased pain threshold
27. Administration of analgesics to the older persons require careful patient assessment because
older people:
A. Are more sensitive drugs
B. Have increased hepatic, renal, and gastrointestinal function
C. Have increased sensory perception
D. Mobilize drugs more rapidly
28. The elderly patient is at higher risk for urinary incontinence because:

A. Increased glomerular filtration


B. Diuretic use
C. Decreased bladder capacity
D. Decreased glomerular filtration
29. Which of the following is the MOST COMMON sign of infection among the elderly?
A. Decreased breath sounds with crackles
B. Fever
C. Pain
D. Change in the mental status
30. Priorities when caring for the elderly trauma patient:
A. Circulation, airway, breathing
B. Disability(Neurologic), airway, breathing
C. Airway, Breathing, Disability(Neurologic),
D. Airway, breathing, Circulation
31. Preschoolers are able to see things from which of the following perspectives?
A. Their peers
B. Their own and their caregivers
C. Their own and their mothers
D. Only their own
32. In conflict management, the win-win approach occurs when:
A. There are two conflicts and the parties agree to each one
B. Each party gives in on 50% of the disagreement making the conflict
C. Both parties involved are committed in solving the conflict
D. The conflict is settled out of court so the legal system mandates parties win.

33. According to the social-International perspective of child abuse and neglect, four factors
place the family members at risk for abuse, these risk factors are the family members at risk
for abuse. These risk factors are the family itself, the caregiver, the child and:
A. The presence of a family crisis
B. Genetics
C. The national emphasis on sex
D. Chronic poverty
34. Which of the following signs and symptoms would you most likely find when assessing an
infant with Arnold-Chiari malformation?
A. Weakness of the leg muscles, loss of sensation in the legs, and restlessness
B. Difficulty swallowing, diminished or absent gag reflex and respiratory distress
C. Difficulty sleeping, hypervigilant and an arching of the back
D. Paradoxical irritability, diarrhea and vomiting
35. A parent calls you and frantically reports that her child has gotten into her famous ferrous
sulfate pills and ingested a number of these pills. Her child is now vomiting, has bloody
diarrhea and is complaining of abdominal pain. You will tell the mother to:
A. Call emergency medical services (EMS) and get the child to the emergency room
B. Relax because these symptoms will pass and the child will be fine
C. Administer syrup of ipecac
D. Call the poison control center
36. A client says she heard from a friend that you stop having periods once you are on the
pill. The most appropriate response would be:
A. The pill prevents the uterus from making such endometrial lining, that is why period may often
be scant or skipped occasionally.
B. If your friend has missed her period, she should stop taking the pills and get a pregnancy test.
C. The pill should cause a normal menstrual period every month. It sounds like your friend has not
been taking the pills properly.
D. Missed period can be very dangerous and may lead to the formation of precancerous cells.

37. The nurse assessing newborn babies and infants during their hospital stay notice which of
the following symptoms as a primary manifestation of Hirschsprungs disorder?
A. A fine rash over the trunk
B. Failure to pass meconium during the first 24 hours after birth
C. The skin turns yellow and then brown over the first 24 hours to 46 hours after birth.
D. High grade fever.
38. A client is 7 months pregnant and has just been diagnosed as having a partial placenta
previa. She is able and has minimal spotting and is being sent home. Which of these
instructions to the client may indicate a need for further teaching?
A. Maintain bed rest with bathroom privileges
B. Avoid intercourse for three days
C. Call if contractions occur.
D. Stay on left side as much as possible when lying down.
39. A woman has been rushed to the hospital with ruptured membrane. Which of the following
should the nurse check first?
A. Check for the presence of infarction.
B. Assess for Prolapse of the umbilical cord
C. Check the maternal heart rate
D. Assess the color of the amniotic fluid
40. The nurse notes that the infant is wearing a plastic-coated diaper. If a topical medication
were to be prescribed and it were to go on the stomachs or buttocks, the nurse would teach the
caregivers to:
A. Avoid covering the area of the topical medication with the diaper
B. Avoid the use of clothing on top of the diaper
C. Put the diaper on as usual
D. Apply an icepack for 5 minutes to the outside of the diaper

41. Which of the following factors is most important in determining the success of relationships
used in delivering nursing care?
A. Type of illness of the client
B. Transference and countertransference
C. Effective communication
D. Personality of the participants
42. Grace sustained a laceration on her leg from automobile accident. Why are lacerations of
lower extremities potentially more serious among pregnant women than other?
A. lacerations can provoke allergic responses due to gonadotropic hormone release
B. a woman is less able to keep the laceration clean because of her fatigue
C. healing is limited during pregnancy so these will not heal until after birth
D. increased bleeding can occur from uterine pressure on leg veins
43. In working with the caregivers of a client with an cute or chronic illness, the nurse would:
A. Teach care daily and let the caregivers do a return demonstration just before discharge
B. Difficulty swallowing, diminished or absent gag reflex and respiratory distress
C. Difficulty sleeping, hypervigilant and arching of the back
D. Paradoxical irritability, diarrhea and vomiting
44. Which of the following roles BEST exemplifies the expanded role of the nurse?
A. Circulating nurse in surgery
B. Medication nurse
C. Obstetrical nurse
D. Pediatric nurse practitioner
45. According to De Rosa and Kochuras (2006) article entitled Implement Culturally Health
Care in your workplace, cultures have different patterns of verbal and nonverbal
communication. Which difference does NOT necessarily belong?

A. Personal behaviour
B. Eye contact
C. Subject Matter
D. Conversational style
46. You are the nurse assigned to work with a child with acute glomerulonephritis. By following
the prescribed treatment regimen, the child experiences a remission. You are now checking to
make sure the child does not have a relapse. Which finding most lead you to the conclusion that
a relapse is happening?
A. Elevated temperature, cough, sore throat, changing complete blood count (CBC) with differential
count
B. A urine dipstick measurement of 2+ proteinuria or more for 3 days or the child found to have 3-4+
proteinuria plus edema.
C. The urine dipstick showing glucose in the urine for 3 days, extreme thirst, increase in urine output
and a moon face.
D. A temperature of 37.8 degrees (100 degrees F) flank pain, burning frequency, urgency on voiding
and cloudy urine.
47. The nurse is working with an adolescent who complains of being lonely and having a lack of
fulfilment in her life. This adolescent shies away from intimate relationships at times yet at
other times she appears promiscuous. The nurse will likely work with this adolescent in which
of the areas?
A. Isolation
B. Loneliness
C. Lack of fulfilment
D. Identity
48. The use of interpersonal decision making psychomotor skills and application of knowledge
expected in the role of a licensed healthcare professional in the context of public health welfare
and safety as an example of?

A. Delegation
B. Supervision
C. Responsibility
D. Competence
49. The painful phenomenon known as back labor occurs in a client whose fetus in what
position?
A. Brow position
B. Right occipito-Anterior Position
C. Breech position
D. Left occipito-Posterior Position
50. Focus methodology stands for?
A. Focus, Organize, Clarify, Understand and Solution
B. Focus, Opportunity, Continuous, Utilize, Substantiate
C. Focus, Organize, Clarify, Understand, Substantiate
D. Focus, Opportunity, Continuous (process), Understand, Solution
Situation 7: The infant and child mortality rate in the low to middle income countries is ten times
higher than industrialized countries. In response to this the WHO and UNICEF launched protocol
Integrated Management of Childhood Illness to reduce the morbidity and mortality against childhood
illnesses.
51. If a child with diarrhea registers two signs in the yellow row in the IMCI chart, we can
classify the patient as:
A. Moderate dehydration
B. Some dehydration
C. Severe dehydration
D. No dehydration
52. Celeste has had diarrhea for 8 days. There is no blood in the stool, he is irritable, his eyes
are sunken, the nurse offers fluid to Celeste and he drinks eagerly. When the nurse pinched the
abdomen it goes back slowly. How will you classify Celestes Illness?

A. Moderate dehydration
B. Some dehydration
C. Severe dehydration
D. No dehydration
53. A child who is 7 weeks has had diarrhea for 14 days but has no sign of dehydration is
classified as?
A. Persistent diarrhea
B. Severe dysentery
C. Dysentery
D. Severe Persistent diarrhea
54. The child with no dehydration needs home treatment. Which of the following is not
included in the rules for home treatment in this case?
A. Forced fluids
B. When to return
C. Give Vitamin A supplement
D. Feeding more
55. Fever as used in IMCI includes:
A. Axillary temperature of 37.5 or higher
B. Rectal temperature of 38 or higher
C. Feeling hot to touch
D. All of the above
E. A and C only
Situation: Prevention of Dengue is an important nursing responsibility and controlling its spread is
priority once outbreak has been observed.
56. An important role of the community health nurse in the prevention and control of Dengue
H-fever includes:

A. Advising the elimination of vectors by keeping water containers covered


B. Conducting strong health education drives/campaign directed toward proper garbage disposal
C. Explaining to the individuals, families, groups and community the nature of the disease and its
causation.
D. Practicing residual spraying with insecticides
57. Community health nurses should be alert in observing a Dengue suspect. The following is
NOT an indicator for hospitalization of H-fever suspects?
A. Marked anorexia, abdominal pain and vomiting
B. Increasing hematocrit count
C. Cough of 30 days
D. Persistent headache
58. The community health nurses primary concern in the immediate control of hemorrhage
among patients with dengue is:
A. Advising low fiber and non-fat diet
B. Providing warmth through light weight covers
C. Observing closely the patient for vital signs leading to shock
D. Keeping the patient at rest.
59. Which of these signs may NOT be REGARDED as a truly positive signs indicative of
Dengue H-fever?
A. Prolonged Bleeding Time
B. Appearance of at least 20 petechiae within 1 cm square
C. Steadily increasing hematocrit count
D. Fall in the platelet count
60. Which of the following is the most important treatment of patients with Dengue H-fever?
A. Give aspirin for fever
B. Replacement of body fluids

C. Avoid unnecessary movement


D. Ice cap over abdomen in case of melena
Situation 9: Health education and Health Promotion is an important part of nursing responsibility in
the community. Immunization is a form of health promotion that aims at preventing the common
childhood illnesses.
61. In correcting misconceptions and myths about certain diseases and their management, the
health worker should first:
A. Identify the myths and misconceptions prevailing in the community
B. Identify the source of these myths and misconceptions
C. Explain how and why these myths came about
D. Select the appropriate IEC strategies to correct them.
62. How many percent of measles are prevented by immunization at 9 months age?
A. 80 %
B. 90%
C. 99 %
D. 95 %
63. After TT3 vaccination a mother is said to be protected to tetanus by around?
A. 80 %
B. 85 %
C. 99 %
D. 90 %
64. If ever convulsion occurs after administering DPT, what should nurse best suggest to the
mother?
A. Do not continue DPT vaccination anymore
B. Advise mother to come back aster 1 week

C. Give DT instead of DPT


D. Give pertussis of the DPT and remove DT
65. These vaccines are given 3 doses at one month intervals:
A. DPT, BCG, TT
B. DPT, TT, OPV
C. OPV, Hep. B, DPT
D. Measles, OPV, DPT
Situation 10: With the increasing documented cases of CANCER the best alternative to treatment still
remains to be PREVENTION. The following conditions apply.
66. Which among the following is the primary focus of prevention of cancer?
A. Elimination of conditions causing cancer
B. Diagnosis and treatment
C. Treatment at early stage
D. Early detection
67. In the prevention and control of cancer, which of the following activity is the most
important function of the community health nurse?
A. Conduct community assemblies
B. Referral to cancer specialist those clients with symptoms of cancer
C. Use the nine warning signs of cancer as parameters in our process of detection; control and
treatment modalities.
D. Teach women about proper/correct nutrition.
68. Who among the following are recipients of the secondary level of care for cancer cases?
A. Those under early case detection
B. Those under supportive care
C. Those scheduled for surgery
D. Those under going treatment

69. Who among the following are recipients of the tertiary level of care for cancer cases?
A. Those under early treatment
B. Those under supportive care
C. Those under early detection
D. Those scheduled for surgery
70. In Community Health Nursing, despite the availability and use of many equipment and
devices to facilitate the job of the community health nurse, the nurse should be prepared to
apply is a scientific approach. This approach ensures quality of care even at the community
setting. This nursing parlance is nothing less than the:
A. Nursing diagnosis
B. Nursing protocol
C. Nursing research
D. Nursing process
Situation 11 Two children were brought to you. One with chest indrawing and the other had
diarrhea. The following questions apply:
71. Using Integrated Management and Childhood Illness (IMCI) approach, how would you
classify the 1st child?
A. Bronchopneumonia
B. No pneumonia: cough or cold
C. Severe pneumonia
D. Pneumonia
72. The 1st child who is 13 months has fast breathing using IMCI parameters he has:
A. 40 breaths per minute or more
B. 50 breaths per minute
C. 30 breaths per minute or more
D. 60 breaths per minute

73. Nina, the 2nd child has diarrhea for 5 days. There is no blood in the stool. She is irritable
and her eyes are sunken. The nurse offered fluids and the child drinks eagerly. How would you
classify Ninas illness?
A. Some dehydration
B. Dysentery
C. Severe dehydration
D. No dehydration
74. Ninas treatment should include the following EXCEPT:
A. Reassess the child and classify him for dehydration
B. For infants under 6 months old who are not breastfed, give 100-200 ml clean water as well during
this period.
C. Give in the health center the recommended amount of ORS for 4 hours.
D. Do not give any other foods to the child for home treatment
75. While on treatment, Nina 18 months old weighed 18 kgs and her temperature registered at
37 degrees C. Her mother says she developed cough 3 days ago. Nina has no general danger
signs. She has 45 breaths/minute, no chest indrawing, no stridor. How would you classify Ninas
manifestation.
A. No pneumonia
B. Severe pneumonia
C. Pneumonia
D. Bronchopneumonia
76. Carol is 15 months old and weighs 5.5 kgs and it is her initial visit. Her mother says that
Carol is not eating well and unable to breastfeed, he has no vomiting, has no convulsion and not
abnormally sleepy or difficult to awaken. Her temperature is 38.9 deg C. Using the integrated
management of childhood illness or IMCI strategy, if you were the nurse in charge of Carol,
how will you classify her illness?
A. a child at a general danger sign
B. very severe febrile disease

C. severe pneumonia
D. severe malnutrition
77. Why are small for gestational age newborns at risk for difficulty maintaining body
temperature?
A. their skin is more susceptible to conduction of cold
B. they are preterm so are born relatively small in size
C. they do not have as many fat stored as other infants
D. they are more active than usual so they throw off comes
78. Oxytocin is administered to Rita to augment labor. What are the first symptoms of water
intoxication to observe for during this procedure?
A. headache and vomiting
B. a swollen tender tongue
C. a high choking voice
D. abdominal bleeding and pain
79. Which of the following treatment should NOT be considered if the child has severe dengue
hemorrhagic fever?
A. use plan C if there is bleeding from the nose or gums
B. give ORS if there is skin petechiae, persistent vomiting, and positive tourniquet test
C. give aspirin
D. prevent low blood sugar
80. In assessing the patients condition using the Integrated Management of Childhood Illness
approach strategy, the first thing that a nurse should do is to:
A. ask what are the childs problem
B. check the patients level of consciousness
C. check for the four main symptoms
D. check for the general danger signs

81. A child with diarrhea is observed for the following EXCEPT:


A. how long the child has diarrhea
B. skin petechiae
C. presence of blood in the stool
D. signs of dehydration
82. The child with no dehydration needs home treatment. Which of the following is NOT
included in the care for home management at this case?
A. give drugs every 4 hours
B. continue feeding the child
C. give the child more fluids
D. inform when to return to the health center
83. Ms. Jordan, RN, believes that a patient should be treated as individual. This ethical
principle that the patient referred to:
A. beneficence
B. nonmaleficence
C. respect for person
D. autonomy
84. When patients cannot make decisions for themselves, the nurse advocate relies on the
ethical principle of:
A. justice and beneficence
B. fidelity and nonmaleficence
C. beneficence and nonmaleficence
D. fidelity and justice
85. Being a community health nurse, you have the responsibility of participating in protecting
the health of people. Consider this situation: Vendors selling bread with their bare hands. They
receive money with these hands. You do not see them washing their hands. What should you say
or do?

A. Miss, may I get the bread myself because you have not washed your hands
B. All of these
C. Miss, it is better to use a pick up forceps/ bread tong
D. Miss, your hands are dirty. Wash your hands first before getting the bread
Situation 12: The following questions refer to common clinical encounters experienced by an entry
level nurse.
86. A female client asks the nurse about the use of cervical cap. Which statement is correct
regarding the use of the cervical cap?
A. It may affect Pap smear results
B. It does not need to be fitted by the physician
C. It does not require the use of spermicide
D. It must be removed within 24 hours
87. The major components of the communication process are?
A. Verbal, written, and nonverbal
B. Speaker, Listener and reply
C. Facial expression, tone of voice and gestures
D. Message, sender, channel, Receiver and Feedback
88. The extent of burns in children are normally assessed and expressed in terms of:
A. The amount of body surface that is unburned
B. Percentages of total body surface area (TBSA)
C. How deep the deepest burns are
D. The severity of the burns on a 1 to 5 burn scale
89. The school nurse notices a child who is wearing old, dirty, poor-fitting clothes; is always
hungry; has no lunch money; and is always tired. When the nurse asks the boy his tiredness, he
talks of playing outside until midnight. The nurse will suspect that this child is:

A. Being raised by a parent of low intelligence (IQ)


B. An orphan
C. A victim of child neglect
D. The victim of poverty
90. Which of the following indicates the type(s) of acute renal failure?
A. Four types: hemorrhagic with and without clotting, and nonhemorrhagic with and without clotting
B. One type: Acute
C. Three types: Prerenal, intrarenal, postrenal
D. Two types: Acute and subacute
Situation 13: Milo 16 y/o has been diagnosed to have AIDS, he worked as entertainer in a cruise ship:
91. Which method of transmission is common to contract AIDS:
A. Syringe and needles
B. Body fluids
C. Sexual contact
D. Transfusion
92. Causative organism in AIDS is one of the following:
A. Fungus
B. Bacteria
C. Retrovirus
D. Parasites
93. You are assigned in a private room of Milo. Which procedure should be of utmost
importance:
A. Alcohol wash
B. Universal precaution
C. Washing isolation
D. Gloving technique

94. What primary health teaching would you give to Milo?


A. Daily exercise
B. Prevent infection
C. Reversal Isolation
D. Proper nutrition
95. Exercise precaution must be taken to protect health worker dealing with the AIDS patients,
which among these must be done as priority?
A. Boil used syringes and needles
B. Use gloves when handling specimen
C. Label personal belonging
D. Avoid accidental wound
Situation 14: Michelle is a 6 year old preschooler. She was reported by her sister to have measles but
she was at home because of fever, upper respiratory problem and white sports in her mouth.
96. Rubeola is an Arabic term meaning Red, the rash appears on the skin in invasive stage
prior to eruption. As a nurse, your physical examination must determine complication
especially:
A. Otitis media
B. Bronchial pneumonia
C. Inflammatory conjunctiva
D. Membranous laryngitis
97. To render comfort measure is one of the priorities, which includes care of the skin, eyes,
ears, mouth and nose. To clean the mouth, your antiseptic is in some form of which one below?
A. Water
B. Sulfur
C. Alkaline
D. Salt

98. As a public health nurse, you teach mother and family members the prevention of
complication of measles. Which of the following should be closely watched?
A. Temperature fails to drop
B. Inflammation of the conjunctiva
C. Inflammation of the nasopharynx
D. Ulcerative stomatitis
99. Source of infection of measles is secretion of nose and throat of infection person. Filterable
of measles is transmitted by:
A. Water supply
B. Droplet
C. Food ingestion
D. Sexual contact
100. Method of prevention is to avoid exposure to an infection person. Nursing responsibility
for rehabilitation of patient includes the provision of:
A. Terminal disinfection
B. Injection of gamma globulin
C. Immunization
D. Comfort measures

Answers
Here are the answers for the exam. Unfortunately, rationales are not given. If you need clarifications
or disputes, please direct them to the comments section and well be glad to give you an explanation.
1.

D. 31

2.

A. 1,2,4

3.

D. BBT is lowest during the secretory phase

4.

B. Progesterone

5.

B. Trophoblastic cells of the embryo

6.

A. Fetal development are felt by Mariah

7.

D. Umbilicus

8.

A. The size of your breast will not affect your lactation.

9.

D. Knowledge deficit regarding nutritional requirements pregnancies


related to lack of information sources.

10.

A. Incorporate her food preferences that are adequately nutritious in her

meal plan.
11.

C. Thelarche

12.

B. The vas deferens

13.

D. Herniation of the bladder into the vaginal wall.

14.

C. Day 15

15.

D. It is due to learning patterns of the female client where she views sex

as bad or sinful.
16.

B. She should assess whether her cervical mucus is thin colour, clear and

watery.
17.

D. Migraine headache

18.

C. Five years

19.

B. She should insert the condom before any penile penetration

20.

A. She has patent fallopian tubes, so fertilized ova can be implanted on

them.
21.

C. Sperm can no longer reach the ova, because the fallopian tubes are

blocked
22.

C. a couple has been trying to conceive for 1 year

23.

A. endometrial implants can block the fallopian tubes

24.

D. She may feel some cramping when the dye is inserted

25.

A. Donor sperm are introduced vaginally into the uterus or cervix

26.

A. Experienced reduce sensory perception

27.

A. Are more sensitive drugs

28.

C. Decreased bladder capacity

29.

D. Change in the mental status

30.

D. Airway, breathing, Circulation

31.

D. Only their own

32.

C. Both parties involved are committed in solving the conflict

33.

A. The presence of a family crisis

34.

B. Difficulty swallowing, diminished or absent gag reflex and respiratory

distress
35.

D. Call the poison control center

36.

C. The pill should cause a normal menstrual period every month. It

sounds like your friend has not been taking the pills properly.
37.

B. Failure to pass meconium during the first 24 hours after birth

38.

B. Avoid intercourse for three days

39.

B. Assess for Prolapse of the umbilical cord

40.

A. Avoid covering the area of the topical medication with the diaper

41.

C. Effective communication

42.

D. increased bleeding can occur from uterine pressure on leg veins

43.

A. Teach care daily and let the caregivers do a return demonstration just

before discharge
44.

D. Pediatric nurse practitioner

45.

C. Subject Matter

46.

B. A urine dipstick measurement of 2+ proteinuria or more for 3 days or

the child found to have 3-4+ proteinuria plus edema.


47.

D. Identity

48.

D. Competence

49.

D. Left occipito-Posterior Position

50.

A. Focus, Organize, Clarify, Understand and Solution

51.

B. Some dehydration

52.

B. Some dehydration

53.

D. Severe Persistent diarrhea

54.

C. Give Vitamin A supplement

55.

D. All of the above

56.

C. Explaining to the individuals, families, groups and community the

nature of the disease and its causation.


57.

C. Cough of 30 days

58.

C. Observing closely the patient for vital signs leading to shock

59.

B. Appearance of at least 20 petechiae within 1 cm square

60.

B. Replacement of body fluids

61.

A. Identify the myths and misconceptions prevailing in the community

62.

A. 80 %

63.

D. 90 %

64.

C. Give DT instead of DPT

65.

C. OPV, Hep. B, DPT

66.

A. Elimination of conditions causing cancer

67.

A. Conduct community assemblies

68.

A. Those under early case detection

69.

B. Those under supportive care

70.

D. Nursing process

71.

C. Severe pneumonia

72.

A. 40 breaths per minute or more

73.

A. Some dehydration

74.

D. Do not give any other foods to the child for home treatment

75.

C. Pneumonia

76.

B. very severe febrile disease

77.

C. they do not have as many fat stored as other infants

78.

A. headache and vomiting

79.

C. give aspirin

80.

A. ask what are the childs problem

81.

B. skin Petechiae

82.

A. give drugs every 4 hours

83.

C respect for person

84.

C. beneficence and nonmaleficence

85.

C. Miss, it is better to use a pick up forceps/ bread tong

86.

A. It may affect Pap smear results

87.

D. Message, sender, channel, Receiver and Feedback

88.

B. Percentages of total body surface area (TBSA)

89.

C. A victim of child neglect

90.

C. Three types: Prerenal, intrarenal, postrenal

91.

C. Sexual contact

92.

C. Retrovirus

93.

B. Universal precaution

94.

B. Prevent infection

95.

B. Use gloves when handling specimen

96.

B. Bronchial pneumonia

97.

D. Salt

98.

C. Inflammation of the nasopharynx

99.

B. Droplet

100. D. Comfort measures


1. When assessing the adequacy of sperm for conception to occur, which of the following is the
most useful criterion?
A. Sperm count
B. Sperm motility
C. Sperm maturity
D. Semen volume
2. A couple who wants to conceive but has been unsuccessful during the last 2 years has
undergone many diagnostic procedures. When discussing the situation with the nurse, one
partner states, We know several friends in our age group and all of them have their own child
already, Why cant we have one?. Which of the following would be the most pertinent nursing
diagnosis for this couple?
A. Fear related to the unknown
B. Pain related to numerous procedures.
C. Ineffective family coping related to infertility.
D. Self-esteem disturbance related to infertility.
3. Which of the following urinary symptoms does the pregnant woman most frequently
experience during the first trimester?
A. Dysuria
B. Frequency
C. Incontinence
D. Burning

4. Heartburn and flatulence, common in the second trimester, are most likely the result of
which of the following?
A. Increased plasma HCG levels
B. Decreased intestinal motility
C. Decreased gastric acidity
D. Elevated estrogen levels
5. On which of the following areas would the nurse expect to observe chloasma?
A. Breast, areola, and nipples
B. Chest, neck, arms, and legs
C. Abdomen, breast, and thighs
D. Cheeks, forehead, and nose
6. A pregnant client states that she waddles when she walks. The nurses explanation is based
on which of the following as the cause?
A. The large size of the newborn
B. Pressure on the pelvic muscles
C. Relaxation of the pelvic joints
D. Excessive weight gain
7. Which of the following represents the average amount of weight gained during pregnancy?
A. 12 to 22 lb
B 15 to 25 lb
C. 24 to 30 lb
D. 25 to 40 lb
8. When talking with a pregnant client who is experiencing aching swollen, leg veins, the nurse
would explain that this is most probably the result of which of the following?
A. Thrombophlebitis
B. Pregnancy-induced hypertension

C. Pressure on blood vessels from the enlarging uterus


D. The force of gravity pulling down on the uterus
9. Cervical softening and uterine souffle are classified as which of the following?
A. Diagnostic signs
B. Presumptive signs
C. Probable signs
D. Positive signs
10. Which of the following would the nurse identify as a presumptive sign of pregnancy?
A. Hegar sign
B. Nausea and vomiting
C. Skin pigmentation changes
D. Positive serum pregnancy test
11. Which of the following common emotional reactions to pregnancy would the nurse expect to
occur during the first trimester?
A. Introversion, egocentrism, narcissism
B. Awkwardness, clumsiness, and unattractiveness
C. Anxiety, passivity, extroversion
D. Ambivalence, fear, fantasies
12. During which of the following would the focus of classes be mainly on physiologic changes,
fetal development, sexuality, during pregnancy, and nutrition?
A. Prepregnant period
B. First trimester
C. Second trimester
D. Third trimester
13. Which of the following would be disadvantage of breast feeding?

A. Involution occurs more rapidly


B. The incidence of allergies increases due to maternal antibodies
C. The father may resent the infants demands on the mothers body
D. There is a greater chance for error during preparation
14. Which of the following would cause a false-positive result on a pregnancy test?
A. The test was performed less than 10 days after an abortion
B. The test was performed too early or too late in the pregnancy
C. The urine sample was stored too long at room temperature
D. A spontaneous abortion or a missed abortion is impending
15. FHR can be auscultated with a fetoscope as early as which of the following?
A. 5 weeks gestation
B. 10 weeks gestation
C. 15 weeks gestation
D. 20 weeks gestation
16. A client LMP began July 5. Her EDD should be which of the following?
A. January 2
B. March 28
C. April 12
D. October 12
17. Which of the following fundal heights indicates less than 12 weeks gestation when the date
of the LMP is unknown?
A. Uterus in the pelvis
B. Uterus at the xiphoid
C. Uterus in the abdomen
D. Uterus at the umbilicus

18. Which of the following danger signs should be reported promptly during the antepartum
period?
A. Constipation
B. Breast tenderness
C. Nasal stuffiness
D. Leaking amniotic fluid
19. Which of the following prenatal laboratory test values would the nurse consider as
significant?
A. Hematocrit 33.5%
B. Rubella titer less than 1:8
C. White blood cells 8,000/mm3
D. One hour glucose challenge test 110 g/dL
20. Which of the following characteristics of contractions would the nurse expect to find in a
client experiencing true labor?
A. Occurring at irregular intervals
B. Starting mainly in the abdomen
C. Gradually increasing intervals
D. Increasing intensity with walking
21. During which of the following stages of labor would the nurse assess crowning?
A. First stage
B. Second stage
C. Third stage
D. Fourth stage
22. Barbiturates are usually not given for pain relief during active labor for which of the
following reasons?

A. The neonatal effects include hypotonia, hypothermia, generalized drowsiness, and reluctance to
feed for the first few days.
B. These drugs readily cross the placental barrier, causing depressive effects in the newborn 2 to 3
hours after intramuscular injection.
C. They rapidly transfer across the placenta, and lack of an antagonist make them generally
inappropriate during labor.
D. Adverse reactions may include maternal hypotension, allergic or toxic reaction or partial or total
respiratory failure
23. Which of the following nursing interventions would the nurse perform during the third
stage of labor?
A. Obtain a urine specimen and other laboratory tests.
B. Assess uterine contractions every 30 minutes.
C. Coach for effective client pushing
D. Promote parent-newborn interaction.
24. Which of the following actions demonstrates the nurses understanding about the
newborns thermoregulatory ability?
A. Placing the newborn under a radiant warmer.
B. Suctioning with a bulb syringe
C. Obtaining an Apgar score
D. Inspecting the newborns umbilical cord
25. Immediately before expulsion, which of the following cardinal movements occur?
A. Descent
B. Flexion
C. Extension
D. External rotation
26. Before birth, which of the following structures connects the right and left auricles of the
heart?

A. Umbilical vein
B. Foramen ovale
C. Ductus arteriosus
D. Ductus venosus
27. Which of the following when present in the urine may cause a reddish stain on the diaper of
a newborn?
A. Mucus
B. Uric acid crystals
C. Bilirubin
D. Excess iron
28. When assessing the newborns heart rate, which of the following ranges would be
considered normal if the newborn were sleeping?
A. 80 beats per minute
B. 100 beats per minute
C. 120 beats per minute
D. 140 beats per minute
29. Which of the following is true regarding the fontanels of the newborn?
A. The anterior is triangular shaped; the posterior is diamond shaped.
B. The posterior closes at 18 months; the anterior closes at 8 to 12 weeks.
C. The anterior is large in size when compared to the posterior fontanel.
D. The anterior is bulging; the posterior appears sunken.
30. Which of the following groups of newborn reflexes below are present at birth and remain
unchanged through adulthood?
A. Blink, cough, rooting, and gag
B. Blink, cough, sneeze, gag
C. Rooting, sneeze, swallowing, and cough
D. Stepping, blink, cough, and sneeze

31. Which of the following describes the Babinski reflex?


A. The newborns toes will hyperextend and fan apart from dorsiflexion of the big toe when one side
of foot is stroked upward from the ball of the heel and across the ball of the foot.
B. The newborn abducts and flexes all extremities and may begin to cry when exposed to sudden
movement or loud noise.
C. The newborn turns the head in the direction of stimulus, opens the mouth, and begins to suck
when cheek, lip, or corner of mouth is touched.
D. The newborn will attempt to crawl forward with both arms and legs when he is placed on his
abdomen on a flat surface
32. Which of the following statements best describes hyperemesis gravidarum?
A. Severe anemia leading to electrolyte, metabolic, and nutritional imbalances in the absence of other
medical problems.
B. Severe nausea and vomiting leading to electrolyte, metabolic, and nutritional imbalances in the
absence of other medical problems.
C. Loss of appetite and continuous vomiting that commonly results in dehydration and ultimately
decreasing maternal nutrients
D. Severe nausea and diarrhea that can cause gastrointestinal irritation and possibly internal bleeding
33. Which of the following would the nurse identify as a classic sign of PIH?
A. Edema of the feet and ankles
B. Edema of the hands and face
C. Weight gain of 1 lb/week
D. Early morning headache
34. In which of the following types of spontaneous abortions would the nurse assess dark brown
vaginal discharge and a negative pregnancy tests?
A. Threatened
B. Imminent
C. Missed
D. Incomplete

35. Which of the following factors would the nurse suspect as predisposing a client to placenta
previa?
A. Multiple gestation
B. Uterine anomalies
C. Abdominal trauma
D. Renal or vascular disease
36. Which of the following would the nurse assess in a client experiencing abruptio placenta?
A. Bright red, painless vaginal bleeding
B. Concealed or external dark red bleeding
C. Palpable fetal outline
D. Soft and nontender abdomen
37. Which of the following is described as premature separation of a normally implanted
placenta during the second half of pregnancy, usually with severe hemorrhage?
A. Placenta previa
B. Ectopic pregnancy
C. Incompetent cervix
D. Abruptio placentae
38. Which of the following may happen if the uterus becomes overstimulated by oxytocin
during the induction of labor?
A. Weak contraction prolonged to more than 70 seconds
B. Tetanic contractions prolonged to more than 90 seconds
C. Increased pain with bright red vaginal bleeding
D. Increased restlessness and anxiety
39. When preparing a client for cesarean delivery, which of the following key concepts should
be considered when implementing nursing care?

A. Instruct the mothers support person to remain in the family lounge until after the delivery
B. Arrange for a staff member of the anesthesia department to explain what to expect postoperatively
C. Modify preoperative teaching to meet the needs of either a planned or emergency cesarean birth
D. Explain the surgery, expected outcome, and kind of anesthetics
40. Which of the following best describes preterm labor?
A. Labor that begins after 20 weeks gestation and before 37 weeks gestation
B. Labor that begins after 15 weeks gestation and before 37 weeks gestation
C. Labor that begins after 24 weeks gestation and before 28 weeks gestation
D. Labor that begins after 28 weeks gestation and before 40 weeks gestation
41. When PROM occurs, which of the following provides evidence of the nurses understanding
of the clients immediate needs?
A. The chorion and amnion rupture 4 hours before the onset of labor.
B. PROM removes the fetus most effective defense against infection
C. Nursing care is based on fetal viability and gestational age.
D. PROM is associated with malpresentation and possibly incompetent cervix
42. Which of the following factors is the underlying cause of dystocia?
A. Nurtional
B. Mechanical
C. Environmental
D. Medical
43. When uterine rupture occurs, which of the following would be the priority?
A. Limiting hypovolemic shock
B. Obtaining blood specimens
C. Instituting complete bed rest
D. Inserting a urinary catheter
44. Which of the following is the nurses initial action when umbilical cord prolapse occurs?

A. Begin monitoring maternal vital signs and FHR


B. Place the client in a knee-chest position in bed
C. Notify the physician and prepare the client for delivery
D. Apply a sterile warm saline dressing to the exposed cord
45. Which of the following amounts of blood loss following birth marks the criterion for
describing postpartum hemorrhage?
A. More than 200 ml
B. More than 300 ml
C. More than 400 ml
D. More than 500 ml
46. Which of the following is the primary predisposing factor related to mastitis?
A. Epidemic infection from nosocomial sources localizing in the lactiferous glands and ducts
B. Endemic infection occurring randomly and localizing in the periglandular connective tissue
C. Temporary urinary retention due to decreased perception of the urge to avoid
D. Breast injury caused by overdistention, stasis, and cracking of the nipples
47. Which of the following best describes thrombophlebitis?
A. Inflammation and clot formation that result when blood components combine to form an
aggregate body
B. Inflammation and blood clots that eventually become lodged within the pulmonary blood vessels
C. Inflammation and blood clots that eventually become lodged within the femoral vein
D. Inflammation of the vascular endothelium with clot formation on the vessel wall
48. Which of the following assessment findings would the nurse expect if the client develops
DVT?
A. Midcalf pain, tenderness and redness along the vein
B. Chills, fever, malaise, occurring 2 weeks after delivery
C. Muscle pain the presence of Homans sign, and swelling in the affected limb
D. Chills, fever, stiffness, and pain occurring 10 to 14 days after delivery

49. Which of the following are the most commonly assessed findings in cystitis?
A. Frequency, urgency, dehydration, nausea, chills, and flank pain
B. Nocturia, frequency, urgency dysuria, hematuria, fever and suprapubic pain
C. Dehydration, hypertension, dysuria, suprapubic pain, chills, and fever
D. High fever, chills, flank pain nausea, vomiting, dysuria, and frequency
50. Which of the following best reflects the frequency of reported postpartum blues?
A. Between 10% and 40% of all new mothers report some form of postpartum blues
B. Between 30% and 50% of all new mothers report some form of postpartum blues
C. Between 50% and 80% of all new mothers report some form of postpartum blues
D. Between 25% and 70% of all new mothers report some form of postpartum blues

Answers & Rationale


Here are the answers and rationale for this exam. Compare your answers to those below and be sure
to read the rationales for additional learning. If you have any disputes or clarifications, please direct
them to the comments section.
1.

B. Although all of the factors listed are important, sperm motility is the
most significant criterion when assessing male infertility. Sperm count,
sperm maturity, and semen volume are all significant, but they are not as
significant sperm motility.

2.

D. Based on the partners statement, the couple is verbalizing feelings of


inadequacy and negative feelings about themselves and their capabilities.
Thus, the nursing diagnosis of self-esteem disturbance is most appropriate.
Fear, pain, and ineffective family coping also may be present but as
secondary nursing diagnoses.

3.

B. Pressure and irritation of the bladder by the growing uterus during the
first trimester is responsible for causing urinary frequency. Dysuria,
incontinence, and burning are symptoms associated with urinary tract
infections.

4.

C. During the second trimester, the reduction in gastric acidity in


conjunction with pressure from the growing uterus and smooth muscle

relaxation, can cause heartburn and flatulence. HCG levels increase in the
first, not the second, trimester. Decrease intestinal motility would most
likely be the cause of constipation and bloating. Estrogen levels decrease in
the second trimester.
5.

D. Chloasma, also called the mask of pregnancy, is an irregular


hyperpigmented area found on the face. It is not seen on the breasts,
areola, nipples, chest, neck, arms, legs, abdomen, or thighs.

6.

C. During pregnancy, hormonal changes cause relaxation of the pelvic


joints, resulting in the typical waddling gait. Changes in posture are
related to the growing fetus. Pressure on the surrounding muscles causing
discomfort is due to the growing uterus. Weight gain has no effect on gait.

7.

C. The average amount of weight gained during pregnancy is 24 to 30 lb.


This weight gain consists of the following: fetus 7.5 lb; placenta and
membrane 1.5 lb; amniotic fluid 2 lb; uterus 2.5 lb; breasts 3 lb; and
increased blood volume 2 to 4 lb; extravascular fluid and fat 4 to 9 lb. A
gain of 12 to 22 lb is insufficient, whereas a weight gain of 15 to 25 lb is
marginal. A weight gain of 25 to 40 lb is considered excessive.

8.

C. Pressure of the growing uterus on blood vessels results in an increased


risk for venous stasis in the lower extremities. Subsequently, edema and
varicose vein formation may occur. Thrombophlebitis is an inflammation of
the veins due to thrombus formation. Pregnancy-induced hypertension is not
associated with these symptoms. Gravity plays only a minor role with these
symptoms.

9.

C. Cervical softening (Goodell sign) and uterine souffl are two probable
signs of pregnancy. Probable signs are objective findings that strongly
suggest pregnancy. Other probable signs include Hegar sign, which is
softening of the lower uterine segment; Piskacek sign, which is enlargement
and softening of the uterus; serum laboratory tests; changes in skin
pigmentation; and ultrasonic evidence of a gestational sac. Presumptive
signs are subjective signs and include amenorrhea; nausea and vomiting;
urinary frequency; breast tenderness and changes; excessive fatigue;
uterine enlargement; and quickening.

10.

B. Presumptive signs of pregnancy are subjective signs. Of the signs

listed, only nausea and vomiting are presumptive signs. Hegar sign,skin
pigmentation changes, and a positive serum pregnancy test are considered
probably signs, which are strongly suggestive of pregnancy.
11.

D. During the first trimester, common emotional reactions include

ambivalence, fear, fantasies, or anxiety. The second trimester is a period of


well-being accompanied by the increased need to learn about fetal growth
and development. Common emotional reactions during this trimester
include narcissism, passivity, or introversion. At times the woman may seem
egocentric and self-centered. During the third trimester, the woman
typically feels awkward, clumsy, and unattractive, often becoming more
introverted or reflective of her own childhood.
12.

B. First-trimester classes commonly focus on such issues as early

physiologic changes, fetal development, sexuality during pregnancy, and


nutrition. Some early classes may include pregnant couples. Second and
third trimester classes may focus on preparation for birth, parenting, and
newborn care.
13.

C. With breast feeding, the fathers body is not capable of providing the

milk for the newborn, which may interfere with feeding the newborn,
providing fewer chances for bonding, or he may be jealous of the infants
demands on his wifes time and body. Breast feeding is advantageous
because uterine involution occurs more rapidly, thus minimizing blood loss.
The presence of maternal antibodies in breast milk helps decrease the
incidence of allergies in the newborn. A greater chance for error is
associated with bottle feeding. No preparation is required for breast feeding.
14.

A. A false-positive reaction can occur if the pregnancy test is performed

less than 10 days after an abortion. Performing the tests too early or too
late in the pregnancy, storing the urine sample too long at room
temperature, or having a spontaneous or missed abortion impending can all
produce false-negative results.
15.

D. The FHR can be auscultated with a fetoscope at about 20 weeks

gestation. FHR usually is ausculatated at the midline suprapubic region with

Doppler ultrasound transducer at 10 to 12 weeks gestation. FHR, cannot be


heard any earlier than 10 weeks gestation.
16.

C. To determine the EDD when the date of the clients LMP is known use

Nagele rule. To the first day of the LMP, add 7 days, subtract 3 months, and
add 1 year (if applicable) to arrive at the EDD as follows: 5 + 7 = 12 (July)
minus 3 = 4 (April). Therefore, the clients EDD is April 12.
17.

A. When the LMP is unknown, the gestational age of the fetus is estimated

by uterine size or position (fundal height). The presence of the uterus in the
pelvis indicates less than 12 weeks gestation. At approximately 12 to 14
weeks, the fundus is out of the pelvis above the symphysis pubis. The
fundus is at the level of the umbilicus at approximately 20 weeks gestation
and reaches the xiphoid at term or 40 weeks.
18.

D. Danger signs that require prompt reporting leaking of amniotic fluid,

vaginal bleeding, blurred vision, rapid weight gain, and elevated blood
pressure. Constipation, breast tenderness, and nasal stuffiness are common
discomforts associated with pregnancy.
19.

B. A rubella titer should be 1:8 or greater. Thurs, a finding of a titer less

than 1:8 is significant, indicating that the client may not possess immunity
to rubella. A hematocrit of 33.5% a white blood cell count of 8,000/mm3,
and a 1 hour glucose challenge test of 110 g/dl are with normal parameters.
20.

D. With true labor, contractions increase in intensity with walking. In

addition, true labor contractions occur at regular intervals, usually starting


in the back and sweeping around to the abdomen. The interval of true labor
contractions gradually shortens.
21.

B. Crowing, which occurs when the newborns head or presenting part

appears at the vaginal opening, occurs during the second stage of labor.
During the first stage of labor, cervical dilation and effacement occur. During
the third stage of labor, the newborn and placenta are delivered. The fourth
stage of labor lasts from 1 to 4 hours after birth, during which time the
mother and newborn recover from the physical process of birth and the
mothers organs undergo the initial readjustment to the nonpregnant state.

22.

C. Barbiturates are rapidly transferred across the placental barrier, and

lack of an antagonist makes them generally inappropriate during active


labor. Neonatal side effects of barbiturates include central nervous system
depression, prolonged drowsiness, delayed establishment of feeding (e.g.
due to poor sucking reflex or poor sucking pressure). Tranquilizers are
associated with neonatal effects such as hypotonia, hypothermia,
generalized drowsiness, and reluctance to feed for the first few days.
Narcotic analgesic readily cross the placental barrier, causing depressive
effects in the newborn 2 to 3 hours after intramuscular injection. Regional
anesthesia is associated with adverse reactions such as maternal
hypotension, allergic or toxic reaction, or partial or total respiratory failure.
23.

D. During the third stage of labor, which begins with the delivery of the

newborn, the nurse would promote parent-newborn interaction by placing


the newborn on the mothers abdomen and encouraging the parents to
touch the newborn. Collecting a urine specimen and other laboratory tests is
done on admission during the first stage of labor. Assessing uterine
contractions every 30 minutes is performed during the latent phase of the
first stage of labor. Coaching the client to push effectively is appropriate
during the second stage of labor.
24.

A. The newborns ability to regulate body temperature is poor. Therefore,

placing the newborn under a radiant warmer aids in maintaining his or her
body temperature. Suctioning with a bulb syringe helps maintain a patent
airway. Obtaining an Apgar score measures the newborns immediate
adjustment to extrauterine life. Inspecting the umbilical cord aids in
detecting cord anomalies.
25.

D. Immediately before expulsion or birth of the rest of the body, the

cardinal movement of external rotation occurs. Descent flexion, internal


rotation, extension, and restitution (in this order) occur before external
rotation.
26.

B. The foramen ovale is an opening between the right and left auricles

(atria) that should close shortly after birth so the newborn will not have a

murmur or mixed blood traveling through the vascular system. The umbilical
vein, ductus arteriosus, and ductus venosus are obliterated at birth.
27.

B. Uric acid crystals in the urine may produce the reddish brick dust

stain on the diaper. Mucus would not produce a stain. Bilirubin and iron are
from hepatic adaptation.
28.

B. The normal heart rate for a newborn that is sleeping is approximately

100 beats per minute. If the newborn was awake, the normal heart rate
would range from 120 to 160 beats per minute.
29.

C. The anterior fontanel is larger in size than the posterior fontanel.

Additionally, the anterior fontanel, which is diamond shaped, closes at 18


months, whereas the posterior fontanel, which is triangular shaped, closes
at 8 to 12 weeks. Neither fontanel should appear bulging, which may
indicate increased intracranial pressure, or sunken, which may indicate
dehydration.
30.

B. Blink, cough, sneeze, swallowing and gag reflexes are all present at

birth and remain unchanged through adulthood. Reflexes such as rooting


and stepping subside within the first year.
31.

A. With the babinski reflex, the newborns toes hyperextend and fan apart

from dorsiflexion of the big toe when one side of foot is stroked upward form
the heel and across the ball of the foot. With the startle reflex, the newborn
abducts and flexes all extremities and may begin to cry when exposed to
sudden movement of loud noise. With the rooting and sucking reflex, the
newborn turns his head in the direction of stimulus, opens the mouth, and
begins to suck when the cheeks, lip, or corner of mouth is touched. With the
crawl reflex, the newborn will attempt to crawl forward with both arms and
legs when he is placed on his abdomen on a flat surface.
32.

B. The description of hyperemesis gravidarum includes severe nausea and

vomiting, leading to electrolyte, metabolic, and nutritional imbalances in the


absence of other medical problems. Hyperemesis is not a form of anemia.
Loss of appetite may occur secondary to the nausea and vomiting of
hyperemesis, which, if it continues, can deplete the nutrients transported to
the fetus. Diarrhea does not occur with hyperemesis.

33.

B. Edema of the hands and face is a classic sign of PIH. Many healthy

pregnant woman experience foot and ankle edema. A weight gain of 2 lb or


more per week indicates a problem. Early morning headache is not a classic
sign of PIH.
34.

C. In a missed abortion, there is early fetal intrauterine death, and

products of conception are not expelled. The cervix remains closed; there
may be a dark brown vaginal discharge, negative pregnancy test, and
cessation of uterine growth and breast tenderness. A threatened abortion is
evidenced with cramping and vaginal bleeding in early pregnancy, with no
cervical dilation. An incomplete abortion presents with bleeding, cramping,
and cervical dilation. An incomplete abortion involves only expulsion of part
of the products of conception and bleeding occurs with cervical dilation.
35.

A. Multiple gestation is one of the predisposing factors that may cause

placenta previa. Uterine anomalies abdominal trauma, and renal or vascular


disease may predispose a client to abruptio placentae.
36.

B. A client with abruptio placentae may exhibit concealed or dark red

bleeding, possibly reporting sudden intense localized uterine pain. The


uterus is typically firm to board-like, and the fetal presenting part may be
engaged. Bright red, painless vaginal bleeding, a palpable fetal outline and
a soft non-tender abdomen are manifestations of placenta previa.
37.

D. Abruptio placentae is described as premature separation of a normally

implanted placenta during the second half of pregnancy, usually with severe
hemorrhage. Placenta previa refers to implantation of the placenta in the
lower uterine segment, causing painless bleeding in the third trimester of
pregnancy. Ectopic pregnancy refers to the implantation of the products of
conception in a site other than the endometrium. Incompetent cervix is a
conduction characterized by painful dilation of the cervical os without
uterine contractions.
38.

B. Hyperstimulation of the uterus such as with oxytocin during the

induction of labor may result in tetanic contractions prolonged to more than


90seconds, which could lead to such complications as fetal distress,
abruptio placentae, amniotic fluid embolism, laceration of the cervix, and

uterine rupture. Weak contractions would not occur. Pain, bright red vaginal
bleeding, and increased restlessness and anxiety are not associated with
hyperstimulation.
39.

C. A key point to consider when preparing the client for a cesarean

delivery is to modify the preoperative teaching to meet the needs of either a


planned or emergency cesarean birth, the depth and breadth of instruction
will depend on circumstances and time available. Allowing the mothers
support person to remain with her as much as possible is an important
concept, although doing so depends on many variables. Arranging for
necessary explanations by various staff members to be involved with the
clients care is a nursing responsibility. The nurse is responsible for
reinforcing the explanations about the surgery, expected outcome, and type
of anesthetic to be used. The obstetrician is responsible for explaining about
the surgery and outcome and the anesthesiology staff is responsible for
explanations about the type of anesthesia to be used.
40.

A. Preterm labor is best described as labor that begins after 20 weeks

gestation and before 37 weeks gestation. The other time periods are
inaccurate.
41.

B. PROM can precipitate many potential and actual problems; one of the

most serious is the fetus loss of an effective defense against infection. This
is the clients most immediate need at this time. Typically, PROM occurs
about 1 hour, not 4 hours, before labor begins. Fetal viability and gestational
age are less immediate considerations that affect the plan of care.
Malpresentation and an incompetent cervix may be causes of PROM.
42.

B. Dystocia is difficult, painful, prolonged labor due to mechanical factors

involving the fetus (passenger), uterus (powers), pelvis (passage), or


psyche. Nutritional, environment, and medical factors may contribute to the
mechanical factors that cause dystocia.
43.

A. With uterine rupture, the client is at risk for hypovolemic shock.

Therefore, the priority is to prevent and limit hypovolemic shock. Immediate


steps should include giving oxygen, replacing lost fluids, providing drug
therapy as needed, evaluating fetal responses and preparing for surgery.

Obtaining blood specimens, instituting complete bed rest, and inserting a


urinary catheter are necessary in preparation for surgery to remedy the
rupture.
44.

B. The immediate priority is to minimize pressure on the cord. Thus the

nurses initial action involves placing the client on bed rest and then placing
the client in a knee-chest position or lowering the head of the bed, and
elevating the maternal hips on a pillow to minimize the pressure on the
cord. Monitoring maternal vital signs and FHR, notifying the physician and
preparing the client for delivery, and wrapping the cord with sterile saline
soaked warm gauze are important. But these actions have no effect on
minimizing the pressure on the cord.
45.

D. Postpartum hemorrhage is defined as blood loss of more than 500 ml

following birth. Any amount less than this not considered postpartum
hemorrhage.
46.

D. With mastitis, injury to the breast, such as overdistention, stasis, and

cracking of the nipples, is the primary predisposing factor. Epidemic and


endemic infections are probable sources of infection for mastitis. Temporary
urinary retention due to decreased perception of the urge to void is a
contributory factor to the development of urinary tract infection, not
mastitis.
47.

D. Thrombophlebitis refers to an inflammation of the vascular

endothelium with clot formation on the wall of the vessel. Blood components
combining to form an aggregate body describe a thrombus or thrombosis.
Clots lodging in the pulmonary vasculature refers to pulmonary embolism; in
the femoral vein, femoral thrombophlebitis.
48.

C. Classic symptoms of DVT include muscle pain, the presence of Homans

sign, and swelling of the affected limb. Midcalf pain, tenderness, and
redness, along the vein reflect superficial thrombophlebitis. Chills, fever and
malaise occurring 2 weeks after delivery reflect pelvic thrombophlebitis.
Chills, fever, stiffness and pain occurring 10 to 14 days after delivery
suggest femoral thrombophlebitis.

49.

B. Manifestations of cystitis include, frequency, urgency, dysuria,

hematuria nocturia, fever, and suprapubic pain. Dehydration, hypertension,


and chills are not typically associated with cystitis. High fever chills, flank
pain, nausea, vomiting, dysuria, and frequency are associated with
pvelonephritis.
50.

C. According to statistical reports, between 50% and 80% of all new

mothers report some form of postpartum blues. The ranges of 10% to 40%,
30% to 50%, and 25% to 70% are incorrect.

1. Which of the following conditions will lead to a small-for-gestational age fetus due to less
blood supply to the fetus?
A. Diabetes in the mother
B. Maternal cardiac condition
C. Premature labor
D. Abruptio placenta
2. The lower limit of viability for infants in terms of age of gestation is:
A. 21-24 weeks
B. 25-27 weeks
C. 28-30 weeks
D. 38-40 weeks
3. Which provision of our 1987 constitution guarantees the right of the unborn child to life
from conception is
A. Article II section 12
B. Article II section 15
C. Article XIII section 11
D. Article XIII section 15

4. In the Philippines, if a nurse performs abortion on the mother who wants it done and she
gets paid for doing it, she will be held liable because
A. Abortion is immoral and is prohibited by the church
B. Abortion is both immoral and illegal in our country
C. Abortion is considered illegal because you got paid for doing it
D. Abortion is illegal because majority in our country are catholics and it is prohibited by the church
5. The preferred manner of delivering the baby in a gravido-cardiac is vaginal delivery assisted
by forceps under epidural anesthesiA. The main rationale for this is:
A. To allow atraumatic delivery of the baby
B. To allow a gradual shifting of the blood into the maternal circulation
C. To make the delivery effort free and the mother does not need to push with contractions
D. To prevent perineal laceration with the expulsion of the fetal head
6. When giving narcotic analgesics to mother in labor, the special consideration to follow is:
A. The progress of labor is well established reaching the transitional stage
B. Uterine contraction is progressing well and delivery of the baby is imminent
C. Cervical dilatation has already reached at least 8 cm. and the station is at least (+)2
D. Uterine contractions are strong and the baby will not be delivered yet within the next 3 hours.
7. The cervical dilatation taken at 8:00 AM in a G1P0 patient was 6 centimeters. A repeat I.E.
done at 10 A. M. showed that cervical dilation was 7 cm. The correct interpretation of this
result is:
A. Labor is progressing as expected
B. The latent phase of Stage 1 is prolonged
C. The active phase of Stage 1 is protracted
D. The duration of labor is normal
8. Which of the following techniques during labor and delivery can lead to uterine inversion?

A. Fundal pressure applied to assist the mother in bearing down during delivery of the fetal head
B. Strongly tugging on the umbilical cord to deliver the placenta and hasten placental separation
C. Massaging the fundus to encourage the uterus to contract
D. Applying light traction when delivering the placenta that has already detached from the uterine
wall
9. The fetal heart rate is checked following rupture of the bag of waters in order to:
A. Check if the fetus is suffering from head compression
B. Determine if cord compression followed the rupture
C. Determine if there is utero-placental insufficiency
D. Check if fetal presenting part has adequately descended following the rupture
10. Upon assessment, the nurse got the following findings: 2 perineal pads highly saturated
with blood within 2 hours post partum, PR= 80 bpm, fundus soft and boundaries not well
defineD. The appropriate nursing diagnosis is:
A. Normal blood loss
B. Blood volume deficiency
C. Inadequate tissue perfusion related to hemorrhage
D. Hemorrhage secondary to uterine atony
11. The following are signs and symptoms of fetal distress EXCEPT:
A. Fetal heart rate (FHR) decreased during a contraction and persists even after the uterine
contraction ends
B. The FHR is less than 120 bpm or over 160 bpm
C. The pre-contraction FHR is 130 bpm, FHR during contraction is 118 bpm and FHR after uterine
contraction is 126 bpm
D. FHR is 160 bpm, weak and irregular
12. If the labor period lasts only for 3 hours, the nurse should suspect that the following
conditions may occur:

1.Laceration of cervix
2.Laceration of perineum
3.Cranial hematoma in the fetus
4.Fetal anoxia
A. 1 & 2
B. 2 & 4
C. 2,3,4
D. 1,2,3,4
13. The primary power involved in labor and delivery is
A. Bearing down ability of mother
B. Cervical effacement and dilatation
C. Uterine contraction
D. Valsalva technique
14. The proper technique to monitor the intensity of a uterine contraction is
A. Place the palm of the hands on the abdomen and time the contraction
B. Place the finger tips lightly on the suprapubic area and time the contraction
C. Put the tip of the fingers lightly on the fundal area and try to indent the abdominal wall at the
height of the contraction
D. Put the palm of the hands on the fundal area and feel the contraction at the fundal area
15. To monitor the frequency of the uterine contraction during labor, the right technique is to
time the contraction
A. From the beginning of one contraction to the end of the same contraction
B. From the beginning of one contraction to the beginning of the next contraction
C. From the end of one contraction to the beginning of the next contraction
D. From the deceleration of one contraction to the acme of the next contraction
16. The peak point of a uterine contraction is called the

A. Acceleration
B. Acme
C. Deceleration
D. Axiom
17. When determining the duration of a uterine contraction the right technique is to time it
from
A. The beginning of one contraction to the end of the same contraction
B. The end of one contraction to the beginning of another contraction
C. The acme point of one contraction to the acme point of another contraction
D. The beginning of one contraction to the end of another contraction
18. When the bag of waters ruptures, the nurse should check the characteristic of the amniotic
fluiD. The normal color of amniotic fluid is
A. Clear as water
B. Bluish
C. Greenish
D. Yellowish
19. When the bag of waters ruptures spontaneously, the nurse should inspect the vaginal
introitus for possible cord prolapse. If there is part of the cord that has prolapsed into the
vaginal opening the correct nursing intervention is:
A. Push back the prolapse cord into the vaginal canal
B. Place the mother on semifowlers position to improve circulation
C. Cover the prolapse cord with sterile gauze wet with sterile NSS and place the woman on
Trendelenburg position
D. Push back the cord into the vagina and place the woman on sims position
20. The fetal heart beat should be monitored every 15 minutes during the 2nd stage of labor.
The characteristic of a normal fetal heart rate is

A. The heart rate will decelerate during a contraction and then go back to its pre-contraction rate after
the contraction
B. The heart rate will accelerate during a contraction and remain slightly above the pre-contraction
rate at the end of the contraction
C. The rate should not be affected by the uterine contraction.
D. The heart rate will decelerate at the middle of a contraction and remain so for about a minute after
the contraction
21. The mechanisms involved in fetal delivery is
A. Descent, extension, flexion, external rotation
B. Descent, flexion, internal rotation, extension, external rotation
C. Flexion, internal rotation, external rotation, extension
D. Internal rotation, extension, external rotation, flexion
22. The first thing that a nurse must ensure when the babys head comes out is
A. The cord is intact
B. No part of the cord is encircling the babys neck
C. The cord is still attached to the placenta
D. The cord is still pulsating
23. To ensure that the baby will breath as soon as the head is delivered, the nurses priority
action is to
A. Suction the nose and mouth to remove mucous secretions
B. Slap the babys buttocks to make the baby cry
C. Clamp the cord about 6 inches from the base
D. Check the babys color to make sure it is not cyanotic
24. When doing perineal care in preparation for delivery, the nurse should observe the
following EXCEPT

A. Use up-down technique with one stroke


B. Clean from the mons veneris to the anus
C. Use mild soap and warm water
D. Paint the inner thighs going towards the perineal area
25. What are the important considerations that the nurse must remember after the placenta is
delivered?
1.Check if the placenta is complete including the membranes
2.Check if the cord is long enough for the baby
3.Check if the umbilical cord has 3 blood vessels
4.Check if the cord has a meaty portion and a shiny portion
A. 1 and 3
B. 2 and 4
C. 1, 3, and 4
D. 2 and 3
26. The following are correct statements about false labor EXCEPT
A. The pain is irregular in intensity and frequency.
B. The duration of contraction progressively lengthens over time
C. There is no vaginal bloody discharge
D. The cervix is still closeD.
27. The passageway in labor and deliver of the fetus include the following EXCEPT
A. Distensibility of lower uterine segment
B. Cervical dilatation and effacement
C. Distensibility of vaginal canal and introitus
D. Flexibility of the pelvis
28. The normal umbilical cord is composed of:

A. 2 arteries and 1 vein


B. 2 veins and 1 artery
C. 2 arteries and 2 veins
D. none of the above
29. At what stage of labor and delivery does a primigravida differ mainly from a multigravida?
A. Stage 1
B. Stage 2
C. Stage 3
D. Stage 4
30. The second stage of labor begins with ___ and ends with __?
A. Begins with full dilatation of cervix and ends with delivery of placenta
B. Begins with true labor pains and ends with delivery of baby
C. Begins with complete dilatation and effacement of cervix and ends with delivery of baby
D. Begins with passage of show and ends with full dilatation and effacement of cervix
31. The following are signs that the placenta has detached EXCEPT:
A. Lengthening of the cord
B. Uterus becomes more globular
C. Sudden gush of blood
D. Mother feels like bearing down
32. When the shiny portion of the placenta comes out first, this is called the ___ mechanism.
A. Schultze
B. Ritgens
C. Duncan
D. Marmets
33. When the babys head is out, the immediate action of the nurse is

A. Cut the umbilical cord


B. Wipe the babys face and suction mouth first
C. Check if there is cord coiled around the neck
D. Deliver the anterior shoulder
34. When delivering the babys head the nurse supports the mothers perineum to prevent tear.
This technique is called
A. Marmets technique
B. Ritgens technique
C. Duncan maneuver
D. Schultze maneuver
35. The basic delivery set for normal vaginal delivery includes the following
instruments/articles EXCEPT:
A. 2 clamps
B. Pair of scissors
C. Kidney basin
D. Retractor
36. As soon as the placenta is delivered, the nurse must do which of the following actions?
A. Inspect the placenta for completeness including the membranes
B. Place the placenta in a receptacle for disposal
C. Label the placenta properly
D. Leave the placenta in the kidney basin for the nursing aide to dispose properly
37. In vaginal delivery done in the hospital setting, the doctor routinely orders an oxytocin to be
given to the mother parenterally. The oxytocin is usually given after the placenta has been
delivered and not before because:
A. Oxytocin will prevent bleeding
B. Oxytocin can make the cervix close and thus trap the placenta inside

C. Oxytocin will facilitate placental delivery


D. Giving oxytocin will ensure complete delivery of the placenta
38. In a gravido-cardiac mother, the first 2 hours postpartum (4th stage of labor and delivery)
particularly in a cesarean section is a critical period because at this stage
A. There is a fluid shift from the placental circulation to the maternal circulation which can overload
the compromised heart.
B. The maternal heart is already weak and the mother can die
C. The delivery process is strenuous to the mother
D. The mother is tired and weak which can distress the heart
39. The drug usually given parentally to enhance uterine contraction is:
A. Terbutalline
B. Pitocin
C. Magnesium sulfate
D. Lidocaine
40. The partograph is a tool used to monitor labor. The maternal parameters
measured/monitored are the following EXCEPT:
A. Vital signs
B. Fluid intake and output
C. Uterine contraction
D. Cervical dilatation
41. The following are natural childbirth procedures EXCEPT:
A. Lamaze method
B. Dick-Read method
C. Ritgens maneuver
D. Psychoprophylactic method

42. The following are common causes of dysfunctional labor. Which of these can a nurse, on her
own manage?
A. Pelvic bone contraction
B. Full bladder
C. Extension rather than flexion of the head
D. Cervical rigidity
43. At what stage of labor is the mother is advised to bear down?
A. When the mother feels the pressure at the rectal area
B. During a uterine contraction
C. In between uterine contraction to prevent uterine rupture
D. Anytime the mother feels like bearing down
44. The normal dilatation of the cervix during the first stage of labor in a nullipara is
A. 1.2 cm./hr
B. 1.5 cm./hr.
C. 1.8 cm./hr
D. 2.0 cm./hr
45. When the fetal head is at the level of the ischial spine, it is said that the station of the head is
A. Station 1
B. Station 0
C. Station +1
D. Station +2
46. During an internal examination, the nurse palpated the posterior fontanel to be at the left
side of the mother at the upper quadrant. The interpretation is that the position of the fetus is:
A. LOA
B. ROP

C. LOP
D. ROA
47. The following are types of breech presentation EXCEPT:
A. Footling
B. Frank
C. Complete
D. Incomplete
48. When the nurse palpates the suprapubic area of the mother and found that the presenting
part is still movable, the right term for this observation that the fetus is
A. Engaged
B. Descended
C. Floating
D. Internal Rotation
49. The placenta should be delivered normally within ___ minutes after the delivery of the
baby.
A. 5 minutes
B. 30 minutes
C. 45 minutes
D. 60 minutes
50. When shaving a woman in preparation for cesarean section, the area to be shaved should be
from ___ to ___
A. Under breast to mid-thigh including the pubic area
B. The umbilicus to the mid-thigh
C. Xyphoid process to the pubic area
D. Above the umbilicus to the pubic area

Answers and Rationale


Here are the answers and rationale for this exam. Counter check your answers to those below and tell
us your scores. If you have any disputes or need more clarification to a certain question, please direct
them to the comments section.
1. Answer: (B) Maternal cardiac condition
In general, when the heart is compromised such as in maternal cardiac condition, the condition can
lead to less blood supply to the uterus consequently to the placenta which provides the fetus with the
essential nutrients and oxygen. Thus if the blood supply is less, the baby will suffer from chronic
hypoxia leading to a small-for-gestational age condition.
2. Answer: (A) 21-24 weeks
Viability means the capability of the fetus to live/survive outside of the uterine environment. With the
present technological and medical advances, 21 weeks AOG is considered as the minimum fetal age
for viability.
3. Answer: (A) Article II section 12
The Philippine Constitution of 1987 guarantees the right of the unborn child from conception equal
to the mother as stated in Article II State Policies, Section 12.
4. Answer: (B) Abortion is both immoral and illegal in our country
Induced Abortion is illegal in the country as stated in our Penal Code and any person who performs
the act for a fee commits a grave offense punishable by 10-12 years of imprisonment.
5. Answer: (C) To make the delivery effort free and the mother does not need to push with
contractions
Forceps delivery under epidural anesthesia will make the delivery process less painful and require
less effort to push for the mother. Pushing requires more effort which a compromised heart may not
be able to endure.
6. Answer: (D) Uterine contractions are strong and the baby will not be delivered yet within the
next 3 hours.
Narcotic analgesics must be given when uterine contractions are already well established so that it

will not cause stoppage of the contraction thus protracting labor. Also, it should be given when
delivery of fetus is imminent or too close because the fetus may suffer respiratory depression as an
effect of the drug that can pass through placental barrier.
7. Answer: (C) The active phase of Stage 1 is protracted
The active phase of Stage I starts from 4cm cervical dilatation and is expected that the uterus will
dilate by 1cm every hour. Since the time lapsed is already 2 hours, the dilatation is expected to be
already 8 cm. Hence, the active phase is protracted.
8. Answer: (B) Strongly tugging on the umbilical cord to deliver the placenta and hasten
placental separation
When the placenta is still attached to the uterine wall, tugging on the cord while the uterus is relaxed
can lead to inversion of the uterus. Light tugging on the cord when placenta has detached is alright in
order to help deliver the placenta that is already detached.
9. Answer: (B) Determine if cord compression followed the rupture
After the rupture of the bag of waters, the cord may also go with the water because of the pressure of
the rupture and flow. If the cord goes out of the cervical opening, before the head is delivered
(cephalic presentation), the head can compress on the cord causing fetal distress. Fetal distress can be
detected through the fetal heart tone. Thus, it is essential do check the FHB right after rupture of bag
to ensure that the cord is not being compressed by the fetal head.
10. Answer: (D) Hemorrhage secondary to uterine atony
All the signs in the stem of the question are signs of hemorrhage. If the fundus is soft and boundaries
not well defined, the cause of the hemorrhage could be uterine atony.
61. Answer: (C) The pre-contraction FHR is 130 bpm, FHR during contraction is 118 bpm and
FHR after uterine contraction is 126 bpm
The normal range of FHR is 120-160 bpm, strong and regular. During a contraction, the FHR usually
goes down but must return to its pre-contraction rate after the contraction ends.
12. Answer: (D) 1,2,3,4
all the above conditions can occur following a precipitate labor and delivery of the fetus because
there was little time for the baby to adapt to the passageway. If the presentation is cephalic, the fetal

head serves as the main part of the fetus that pushes through the birth canal which can lead to cranial
hematoma, and possible compression of cord may occur which can lead to less blood and oxygen to
the fetus (hypoxia). Likewise the maternal passageway (cervix, vaginal canal and perineum) did not
have enough time to stretch which can lead to laceration.
13. Answer: (C) Uterine contraction
Uterine contraction is the primary force that will expel the fetus out through the birth canal Maternal
bearing down is considered the secondary power/force that will help push the fetus out.
14. Answer: (C) Put the tip of the fingers lightly on the fundal area and try to indent the
abdominal wall at the height of the contraction
In monitoring the intensity of the contraction the best place is to place the fingertips at the fundal
area. The fundus is the contractile part of the uterus and the fingertips are more sensitive than the
palm of the hand.
15. Answer: (B) From the beginning of one contraction to the beginning of the next contraction
Frequency of the uterine contraction is defined as from the beginning of one contraction to the
beginning of another contraction.
16. Answer: (B) Acme
Acme is the technical term for the highest point of intensity of a uterine contraction.
17. Answer: (A) The beginning of one contraction to the end of the same contraction
Duration of a uterine contraction refers to one contraction. Thus it is correctly measure from the
beginning of one contraction to the end of the same contraction and not of another contraction.
18. Answer: (A) Clear as water
The normal color of amniotic fluid is clear like water. If it is yellowish, there is probably Rh
incompatibility. If the color is greenish, it is probably meconium stained.
19. Answer: (C) Cover the prolapse cord with sterile gauze wet with sterile NSS and place the
woman on Trendelenburg position
The correct action of the nurse is to cover the cord with sterile gauze wet with sterile NSS. Observe
strict asepsis in the care of the cord to prevent infection. The cord has to be kept moist to prevent it

from drying. Dont attempt to put back the cord into the vagina but relieve pressure on the cord by
positioning the mother either on Trendelenburg or Sims position
20.Answer: (A) The heart rate will decelerate during a contraction and then go back to its precontraction rate after the contraction
The normal fetal heart rate will decelerate (go down) slightly during a contraction because of the
compression on the fetal head. However, the heart rate should go back to the pre-contraction rate as
soon as the contraction is over since the compression on the head has also ended.
21. Answer: (B) Descent, flexion, internal rotation, extension, external rotation
The mechanism of fetal delivery begins with descent into the pelvic inlet which may occur several
days before true labor sets in the primigravida. Flexion, internal rotation and extension are
mechanisms that the fetus must perform as it accommodates through the passageway/birth canal.
Eternal rotation is done after the head is delivered so that the shoulders will be easily delivered
through the vaginal introitus.
22. Answer: (B) No part of the cord is encircling the babys neck
The nurse should check right away for possible cord coil around the neck because if it is present, the
baby can be strangulated by it and the fetal head will have difficulty being delivered.
23.Answer: (A) Suction the nose and mouth to remove mucous secretions
Suctioning the nose and mouth of the fetus as soon as the head is delivered will remove any
obstruction that maybe present allowing for better breathing. Also, if mucus is in the nose and mouth,
aspiration of the mucus is possible which can lead to aspiration pneumonia. (Remember that only the
babys head has come out as given in the situation.)
24. Answer: (D) Paint the inner thighs going towards the perineal area
Painting of the perineal area in preparation for delivery of the baby must always be done but the
stroke should be from the perineum going outwards to the thighs. The perineal area is the one being
prepared for the delivery and must be kept clean
25. Answer: (A) 1 and 3
The nurse after delivering the placenta must ensure that all the cotyledons and the membranes of the

placenta are complete. Also, the nurse must check if the umbilical cord is normal which means it
contains the 3 blood vessels: 1 vein and 2 arteries.
26. Answer: (B) The duration of contraction progressively lengthens over time
In false labor, the contractions remain to be irregular in intensity and duration while in true labor, the
contractions become stronger, longer and more frequent.
27. Answer: (D) Flexibility of the pelvis
The pelvis is a bony structure that is part of the passageway but is not flexible. The lower uterine
segment including the cervix as well as the vaginal canal and introitus are all part of the passageway
in the delivery of the fetus.
28. Answer: (A) 2 arteries and 1 vein
The umbilical cord is composed of 2 arteries and 1 vein.
29. Answer: (A) Stage 1
In stage 1 during a normal vaginal delivery of a vertex presentation, the multigravida may have about
8 hours labor while the primigravida may have up to 12 hours labor.
30. Answer: (C) Begins with complete dilatation and effacement of cervix and ends with
delivery of baby
Stage 2 of labor and delivery process begins with full dilatation of the cervix and ends with the
delivery of baby. Stage 1 begins with true labor pains and ends with full dilatation and effacement of
the cervix.
31. Answer: (D) Mother feels like bearing down
Placental detachment does not require the mother to bear down. A normal placenta will detach by
itself without any effort from the mother.
32. Answer: (A) Schultze
There are 2 mechanisms possible during the delivery of the placenta. If the shiny portion comes out
first, it is called the Schultze mechanism; while if the meaty portion comes out first, it is called the
Duncan mechanism.

33. Answer: (C) Check if there is cord coiled around the neck
The nurse should check if there is a cord coil because the baby will not be delivered safely if the cord
is coiled around its neck. Wiping of the face should be done seconds after you have ensured that there
is no cord coil but suctioning of the nose should be done after the mouth because the baby is a nasal
obligate breather. If the nose is suctioned first before the mouth, the mucus plugging the mouth can
be aspirated by the baby.
34. Answer: (B) Ritgens technique
Ritgens technique is done to prevent perineal tear. This is done by the nurse by support the perineum
with a sterile towel and pushing the perineum downard with one hand while the other hand is
supporting the babys head as it goes out of the vaginal opening.
35. Answer: (D) Retractor
For normal vaginal delivery, the nurse needs only the instruments for cutting the umbilical cord such
as: 2 clamps (straight or curve) and a pair of scissors as well as the kidney basin to receive the
placenta. The retractor is not part of the basic set. In the hospital setting, needle holder and tissue
forceps are added especially if the woman delivering the baby is a primigravida wherein episiotomy
is generally done.
36. Answer: (A) Inspect the placenta for completeness including the membranes
The placenta must be inspected for completeness to include the membranes because an incomplete
placenta could mean that there is retention of placental fragments which can lead to uterine atony. If
the uterus does not contract adequately, hemorrhage can occur.
37. Answer: (B) Oxytocin can make the cervix close and thus trap the placenta inside
The action of oxytocin is to make the uterus contract as well make the cervix close. If it is given prior
to placental delivery, the placenta will be trapped inside because the action of the drug is almost
immediate if given parentally.
38. Answer: (A) There is a fluid shift from the placental circulation to the maternal circulation
which can overload the compromised heart.
During the pregnancy, there is an increase in maternal blood volume to accommodate the need of the
fetus. When the baby and placenta have been delivered, there is a fluid shift back to the maternal

circulation as part of physiologic adaptation during the postpartum period. In cesarean section, the
fluid shift occurs faster because the placenta is taken out right after the baby is delivered giving it
less time for the fluid shift to gradually occur.
39. Answer: (B) Pitocin
The common oxytocin given to enhance uterine contraction is pitocin. This is also the drug given to
induce labor.
40. Answer: (B) Fluid intake and output
Partograph is a monitoring tool designed by the World Health Organization for use by health workers
when attending to mothers in labor especially the high risk ones. For maternal parameters all of the
above is placed in the partograph except the fluid intake since this is placed in a separate monitoring
sheet.
41. Answer: (C) Ritgens maneuver
Ritgens method is used to prevent perineal tear/laceration during the delivery of the fetal head.
Lamaze method is also known as psychoprophylactic method and Dick-Read method are commonly
known natural childbirth procedures which advocate the use of non-pharmacologic measures to
relieve labor pain.
42. Answer: (B) Full bladder
Full bladder can impede the descent of the fetal head. The nurse can readily manage this problem by
doing a simple catheterization of the mother.
43. Answer: (B) During a uterine contraction
The primary power of labor and delivery is the uterine contraction. This should be augmented by the
mothers bearing down during a contraction.
44. Answer: (A) 1.2 cm./hr
For nullipara the normal cervical dilatation should be 1.2 cm/hr. If it is less than that, it is considered
a protracted active phase of the first stage. For multipara, the normal cervical dilatation is 1.5 cm/hr.
45. Answer: (B) Station 0
Station is defined as the relationship of the fetal head and the level of the ischial spine. At the level of

the ischial spine, the station is 0. Above the ischial spine it is considered (-) station and below the
ischial spine it is (+) station.
46. Answer: (A) LOA
The landmark used in determine fetal position is the posterior fontanel because this is the nearest to
the occiput. So if the nurse palpated the occiput (O) at the left (L) side of the mother and at the
upper/anterior (A) quadrant then the fetal position is LOA.
47. Answer: (D) Incomplete
Breech presentation means the buttocks of the fetus is the presenting part. If it is only the foot/feet, it
is considered footling. If only the buttocks, it is frank breech. If both the feet and the buttocks are
presenting it is called complete breech.
48. Answer: (C) Floating
The term floating means the fetal presenting part has not entered/descended into the pelvic inlet. If
the fetal head has entered the pelvic inlet, it is said to be engaged.
49. Answer: (B) 30 minutes
The placenta is delivered within 30 minutes from the delivery of the baby. If it takes longer, probably
the placenta is abnormally adherent and there is a need to refer already to the obstetrician.
50. Answer: (A) Under breast to mid-thigh including the pubic area
Shaving is done to prevent infection and the area usually shaved should sufficiently cover the area for
surgery, cesarean section. The pubic hair is definitely to be included in the shaving
1. Postpartum Period:
The fundus of the uterus is expected to go down normally postpartally about __ cm per day.
A. 1.0 cm
B. 2.0 cm
C. 2.5 cm
D. 3.0 cm
2. The lochia on the first few days after delivery is characterized as

A. Pinkish with some blood clots


B. Whitish with some mucus
C. Reddish with some mucus
D. Serous with some brown tinged mucus
3. Lochia normally disappears after how many days postpartum?
A. 5 days
B. 7-10 days
C. 18-21 days
D. 28-30 days
4. After an Rh(-) mother has delivered her Rh (+) baby, the mother is given RhoGam. This is
done in order to:
A. Prevent the recurrence of Rh(+) baby in future pregnancies
B. Prevent the mother from producing antibodies against the Rh(+) antigen that she may have gotten
when she delivered to her Rh(+) baby
C. Ensure that future pregnancies will not lead to maternal illness
D. To prevent the newborn from having problems of incompatibility when it breastfeeds
5. To enhance milk production, a lactating mother must do the following interventions
EXCEPT:
A. Increase fluid intake including milk
B. Eat foods that increases lactation which are called galactagues
C. Exercise adequately like aerobics
D. Have adequate nutrition and rest
6. The nursing intervention to relieve pain in breast engorgement while the mother continues to
breastfeed is
A. Apply cold compress on the engorged breast
B. Apply warm compress on the engorged breast

C. Massage the breast


D. Apply analgesic ointment
7. A woman who delivered normally per vagina is expected to void within ___ hours after
delivery.
A. 3 hrs
B. 4 hrs.
C. 6-8 hrs
D. 12-24 hours
8. To ensure adequate lactation the nurse should teach the mother to:
A. Breast feed the baby on self-demand day and night
B. Feed primarily during the day and allow the baby to sleep through the night
C. Feed the baby every 3-4 hours following a strict schedule
D. Breastfeed when the breast are engorged to ensure adequate supply
9. An appropriate nursing intervention when caring for a postpartum mother with
thrombophlebitis is:
A. Encourage the mother to ambulate to relieve the pain in the leg
B. Instruct the mother to apply elastic bondage from the foot going towards the knee to improve
venous return flow
C. Apply warm compress on the affected leg to relieve the pain
D. Elevate the affected leg and keep the patient on bedrest
10. The nurse should anticipate that hemorrhage related to uterine atony may occur
postpartally if this condition was present during the delivery:
A. Excessive analgesia was given to the mother
B. Placental delivery occurred within thirty minutes after the baby was born
C. An episiotomy had to be done to facilitate delivery of the head
D. The labor and delivery lasted for 12 hours

11. According to Rubins theory of maternal role adaptation, the mother will go through 3
stages during the post partum perioD. These stages are:
A. Going through, adjustment period, adaptation period
B. Taking-in, taking-hold and letting-go
C. Attachment phase, adjustment phase, adaptation phase
D. Taking-hold, letting-go, attachment phase
12. The neonate of a mother with diabetes mellitus is prone to developing hypoglycemia
because:
A. The pancreas is immature and unable to secrete the needed insulin
B. There is rapid diminution of glucose level in the babys circulating blood and his pancreas is
normally secreting insulin
C. The baby is reacting to the insulin given to the mother
D. His kidneys are immature leading to a high tolerance for glucose
13. Which of the following is an abnormal vital sign in postpartum?
A. Pulse rate between 50-60/min
B. BP diastolic increase from 80 to 95mm Hg
C. BP systolic between 100-120mm Hg
D. Respiratory rate of 16-20/min
14. The uterine fundus right after delivery of placenta is palpable at
A. Level of Xyphoid process
B. Level of umbilicus
C. Level of symphysis pubis
D. Midway between umbilicus and symphysis pubis
15. After how many weeks after delivery should a woman have her postpartal check-up based
on the protocol followed by the DOH Philippines?

A. 2 weeks
B. 3 weeks
C. 6 weeks
D. 12 weeks
16. In a woman who is not breastfeeding, menstruation usually occurs after how many weeks?
A. 2-4 weeks
B. 6-8 weeks
C. 6 months
D. 12 months
17. The following are nursing measures to stimulate lactation EXCEPT
A. Frequent regular breast feeding
B. Breast pumping
C. Breast massage
D. Application of cold compress on the breast
18. When the uterus is firm and contracted after delivery but there is vaginal bleeding, the
nurse should suspect
A. Laceration of soft tissues of the cervix and vagina
B. Uterine atony
C. Uterine inversion
D. Uterine hypercontractility
19. The following are interventions to make the fundus contract postpartally EXCEPT
A. Make the baby suck the breast regularly
B. Apply ice cap on fundus
C. Massage the fundus vigorously for 15 minutes until contracted
D. Give oxytocin as ordered

20. The following are nursing interventions to relieve episiotomy wound pain EXCEPT
A. Giving analgesic as ordered
B. Sitz bath
C. Perineal heat
D. Perineal care
21. Postpartum blues is said to be normal provided that the following characteristics are
present. These are
1. Within 3-10 days only;
2. Woman exhibits the following symptoms- episodic tearfulness, fatigue, oversensitivity, poor
appetite;
3. Maybe more severe symptoms in primpara
A. All of the above
B. 1 and 2
C. 2 only
D. 2 and 3
22. The neonatal circulation differs from the fetal circulation because
A. The fetal lungs are non-functioning as an organ and most of the blood in the fetal circulation is
mixed blooD.
B. The blood at the left atrium of the fetal heart is shunted to the right atrium to facilitate its passage
to the lungs
C. The blood in left side of the fetal heart contains oxygenated blood while the blood in the right side
contains unoxygenated blooD.
D. None of the above
23. The normal respiration of a newborn immediately after birth is characterized as:
A. Shallow and irregular with short periods of apnea lasting not longer than 15 seconds, 30-60
breaths per minute

B. 20-40 breaths per minute, abdominal breathing with active use of intercostals muscles
C. 30-60 breaths per minute with apnea lasting more than 15 seconds, abdominal breathing
D. 30-50 breaths per minute, active use of abdominal and intercostal muscles
24. The anterior fontanelle is characterized as:
A. 3-4 cm antero-posterior diameter and 2-3 cm transverse diameter, diamond shape
B. 2-3 cm antero-posterior diameter and 3-4 cm transverse diameter and diamond shape
C. 2-3 cm in both antero-posterior and transverse diameter and diamond shape
D. none of the above
25. The ideal site for vitamin K injection in the newborn is:
A. Right upper arm
B. Left upper arm
C. Either right or left buttocks
D. Middle third of the thigh
26. At what APGAR score at 5 minutes after birth should resuscitation be initiated?
A. 1-3
B. 7-8
C. 9-10
D. 6-7
27. Right after birth, when the skin of the babys trunk is pinkish but the soles of the feet and
palm of the hands are bluish this is called:
A. Syndactyly
B. Acrocyanosis
C. Peripheral cyanosis
D. Cephalo-caudal cyanosis
28. The minimum birth weight for full term babies to be considered normal is:

A. 2,000gms
B. 1,500gms
C. 2,500gms
D. 3,000gms
29. The procedure done to prevent ophthalmia neonatorum is:
A. Marmets technique
B. Credes method
C. Ritgens method
D. Ophthalmic wash
30. Which of the following characteristics will distinguish a postmature neonate at birth?
A. Plenty of lanugo and vernix caseosa
B. Lanugo mainly on the shoulders and vernix in the skin folds
C. Pinkish skin with good turgor
D. Almost leather-like, dry, cracked skin, negligible vernix caseosa
31. According to the Philippine Nursing Law, a registered nurse is allowed to handle mothers in
labor and delivery with the following considerations:
1. The pregnancy is normal.;
2. The labor and delivery is uncomplicated;
3. Suturing of perineal laceration is allowed provided the nurse had special training;
4. As a delivery room nurse she is not allowed to insert intravenous fluid unless she had special
training for it.
A. 1 and 2
B. 1, 2, and 3
C. 3 and 4
D. 1, 2, and 4

32. Birth Control Methods and Infertility:


In basal body temperature (BBT) technique, the sign that ovulation has occurred is an
elevation of body temperature by
A. 1.0-1.4 degrees centigrade
B. 0.2-0.4 degrees centigrade
C. 2.0-4.0 degrees centigrade
D. 1.0-4.0 degrees centigrade
33. Lactation Amenorrhea Method(LAM) can be an effective method of natural birth control if
A. The mother breast feeds mainly at night time when ovulation could possibly occur
B. The mother breastfeeds exclusively and regularly during the first 6 months without giving
supplemental feedings
C. The mother uses mixed feeding faithfully
D. The mother breastfeeds regularly until 1 year with no supplemental feedings
34. Intra-uterine device prevents pregnancy by the ff. mechanism EXCEPT
A. Endometrium inflames
B. Fundus contracts to expel uterine contents
C. Copper embedded in the IUD can kill the sperms
D. Sperms will be barred from entering the fallopian tubes
35. Oral contraceptive pills are of different types. Which type is most appropriate for mothers
who are breastfeeding?
A. Estrogen only
B. Progesterone only
C. Mixed type- estrogen and progesterone
D. 21-day pills mixed type

36. The natural family planning method called Standard Days (SDM), is the latest type and
easy to use methoD. However, it is a method applicable only to women with regular menstrual
cycles between ___ to ___ days.
A. 21-26 days
B. 26-32 days
C. 28-30 days
D. 24- 36 days
37. Which of the following are signs of ovulation?
1. Mittelschmerz;
2. Spinnabarkeit;
3. Thin watery cervical mucus;
4. Elevated body temperature of 4.0 degrees centigrade
A. 1 & 2
B. 1, 2, & 3
C. 3 & 4
D. 1, 2, 3, 4
38. The following methods of artificial birth control works as a barrier device EXCEPT:
A. Condom
B. Cervical cap
C. Cervical Diaphragm
D. Intrauterine device (IUD)
39. Which of the following is a TRUE statement about normal ovulation?
A. It occurs on the 14th day of every cycle
B. It may occur between 14-16 days before next menstruation
C. Every menstrual period is always preceded by ovulation
D. The most fertile period of a woman is 2 days after ovulation

40. If a couple would like to enhance their fertility, the following means can be done:
1. Monitor the basal body temperature of the woman everyday to determine peak period of
fertility;
2. Have adequate rest and nutrition;
3. Have sexual contact only during the dry period of the woman;
4. Undergo a complete medical check-up to rule out any debilitating disease
A. 1 only
B. 1 & 4
C. 1,2,4
D. 1,2,3,4
41. In sympto-thermal method, the parameters being monitored to determine if the woman is
fertile or infertile are:
A. Temperature, cervical mucus, cervical consistency
B. Release of ovum, temperature and vagina
C. Temperature and wetness
D. Temperature, endometrial secretion, mucus
42. The following are important considerations to teach the woman who is on low dose (minipill) oral contraceptive EXCEPT:
A. The pill must be taken everyday at the same time
B. If the woman fails to take a pill in one day, she must take 2 pills for added protection
C. If the woman fails to take a pill in one day, she needs to take another temporary method until she
has consumed the whole pack
D. If she is breast feeding, she should discontinue using mini-pill and use the progestin-only type
43. To determine if the cause of infertility is a blockage of the fallopian tubes, the test to be done
is

A. Huhners test
B. Rubins test
C. Postcoital test
D. None of the above
44. Infertility can be attributed to male causes such as the following EXCEPT:
A. Cryptorchidism
B. Orchitis
C. Sperm count of about 20 million per milliliter
D. Premature ejaculation
45. Spinnabarkeit is an indicator of ovulation which is characterized as:
A. Thin watery mucus which can be stretched into a long strand about 10 cm
B. Thick mucus that is detached from the cervix during ovulation
C. Thin mucus that is yellowish in color with fishy odor
D. Thick mucus vaginal discharge influence by high level of estrogen
46. Vasectomy is a procedure done on a male for sterilization. The organ involved in this
procedure is
A. Prostate gland
B. Seminal vesicle
C. Testes
D. Vas deferens
47. Breast self examination is best done by the woman on herself every month during
A. The middle of her cycle to ensure that she is ovulating
B. During the menstrual period
C. Right after the menstrual period so that the breast is not being affected by the increase in
hormones particularly estrogen
D. Just before the menstrual period to determine if ovulation has occurred

48. A woman is considered to be menopause if she has experienced cessation of her menses for a
period of
A. 6 months
B. 12 months
C. 18 months
D. 24 months
49. Which of the following is the correct practice of self breast examination in a menopausal
woman?
A. She should do it at the usual time that she experiences her menstrual period in the past to ensure
that her hormones are not at its peak
B. Any day of the month as long it is regularly observed on the same day every month
C. Anytime she feels like doing it ideally every day
D. Menopausal women do not need regular self breast exam as long as they do it at least once every 6
months
50. In assisted reproductive technology (ART), there is a need to stimulate the ovaries to
produce more than one mature ovA. The drug commonly used for this purpose is:
A. Bromocriptine
B. Clomiphene
C. Provera
D. Estrogen

Answers and Rationale


Here are the answers and rationale for this exam. Counter check your answers to those below and tell
us your scores. If you have any disputes or need more clarification to a certain question, please direct
them to the comments section.

1. Answer: (A) 1.0 cm


The uterus will begin involution right after delivery. It is expected to regress/go down by 1 cm. per
day and becomes no longer palpable about 1 week after delivery.
2. Answer: (C) Reddish with some mucus
Right after delivery, the vaginal discharge called lochia will be reddish because there is some blood,
endometrial tissue and mucus. Since it is not pure blood it is non-clotting.
3. Answer: (B) 7-10 days
Normally, lochia disappears after 10 days postpartum. Whats important to remember is that the color
of lochia gets to be lighter (from reddish to whitish) and scantier everyday.
4. Answer: (B) Prevent the mother from producing antibodies against the Rh(+) antigen that
she may have gotten when she delivered to her Rh(+) baby
In Rh incompatibility, an Rh(-) mother will produce antibodies against the fetal Rh (+) antigen which
she may have gotten because of the mixing of maternal and fetal blood during labor and delivery.
Giving her RhoGam right after birth will prevent her immune system from being permanently
sensitized to Rh antigen.
5. Answer: (C) Exercise adequately like aerobics
All the above nursing measures are needed to ensure that the mother is in a healthy state. However,
aerobics does not necessarily enhance lactation.
6. Answer: (B) Apply warm compress on the engorged breast
Warm compress is applied if the purpose is to relieve pain but ensure lactation to continue. If the
purpose is to relieve pain as well as suppress lactation, the compress applied is cold.
7. Answer: (C) 6-8 hrs
A woman who has had normal delivery is expected to void within 6-8 hrs. If she is unable to do so
after 8 hours, the nurse should stimulate the woman to void. If nursing interventions to stimulate
spontaneous voiding dont work, the nurse may decide to catheterize the woman.

8. Answer: (A) Breast feed the baby on self-demand day and night
Feeding on self-demand means the mother feeds the baby according to babys need. Therefore, this
means there will be regular emptying of the breasts, which is essential to maintain adequate lactation.
9. Answer: (D) Elevate the affected leg and keep the patient on bedrest
If the mother already has thrombophlebitis, the nursing intervention is bedrest to prevent the possible
dislodging of the thrombus and keeping the affected leg elevated to help reduce the inflammation.
10. Answer: (A) Excessive analgesia was given to the mother
Excessive analgesia can lead to uterine relaxation thus lead to hemorrhage postpartally. Both B and D
are normal and C is at the vaginal introitus thus will not affect the uterus.
11. Answer: (B) Taking-in, taking-hold and letting-go
Rubins theory states that the 3 stages that a mother goes through for maternal adaptation are: takingin, taking-hold and letting-go. In the taking-in stage, the mother is more passive and dependent on
others for care. In taking-hold, the mother begins to assume a more active role in the care of the child
and in letting-go, the mother has become adapted to her maternal role.
12. Answer: (B) There is rapid diminution of glucose level in the babys circulating blood and
his pancreas is normally secreting insulin
If the mother is diabetic, the fetus while in utero has a high supply of glucose. When the baby is born
and is now separate from the mother, it no longer receives a high dose of glucose from the mother. In
the first few hours after delivery, the neonate usually does not feed yet thus this can lead to
hypoglycemia.
13. Answer: (B) BP diastolic increase from 80 to 95mm Hg
All the vital signs given in the choices are within normal range except an increase of 15mm Hg in the
diastolic which is a possible sign of hypertension in pregnancy.
14. Answer: (B) Level of umbilicus
Immediately after the delivery of the placenta, the fundus of the uterus is expected to be at the level
of the umbilicus because the contents of the pregnancy have already been expelled. The fundus is
expected to recede by 1 fingerbreadths (1cm) everyday until it becomes no longer palpable above the
symphysis pubis.

15. Answer: (C) 6 weeks


According to the DOH protocol postpartum check-up is done 6-8 weeks after delivery to make sure
complete involution of the reproductive organs has be achieved.
16. Answer: (B) 6-8 weeks
When the mother does not breastfeed, the normal menstruation resumes about 6-8 weeks after
delivery. This is due to the fact that after delivery, the hormones estrogen and progesterone gradually
decrease thus triggering negative feedback to the anterior pituitary to release the Folicle-Stimulating
Hormone (FSH) which in turn stimulates the ovary to again mature a graafian follicle and the
menstrual cycle post pregnancy resumes.
17. Answer: (D) Application of cold compress on the breast
To stimulate lactation, warm compress is applied on the breast. Cold application will cause
vasoconstriction thus reducing the blood supply consequently the production of milk.
18. Answer: (A) Laceration of soft tissues of the cervix and vagina
When uterus is firm and contracted it means that the bleeding is not in the uterus but other parts of
the passageway such as the cervix or the vagina.
19. Answer: (C) Massage the fundus vigorously for 15 minutes until contracted
Massaging the fundus of the uterus should not be vigorous and should only be done until the uterus
feel firm and contracted. If massaging is vigorous and prolonged, the uterus will relax due to over
stimulation.
20. Answer: (D) Perineal care
Perineal care is primarily done for personal hygiene regardless of whether there is pain or not;
episiotomy wound or not.
21. Answer: (A) All of the above
All the symptoms 1-3 are characteristic of postpartal blues. It will resolve by itself because it is
transient and is due to a number of reasons like changes in hormonal levels and adjustment to
motherhood. If symptoms lasts more than 2 weeks, this could be a sign of abnormality like
postpartum depression and needs treatment.

22. Answer: (A) The fetal lungs are non-functioning as an organ and most of the blood in the
fetal circulation is mixed blood.
The fetal lungs is fluid-filled while in utero and is still not functioning. It only begins to function in
extra uterine life. Except for the blood as it enters the fetus immediately from the placenta, most of
the fetal blood is mixed blood.
23. Answer: (A) Shallow and irregular with short periods of apnea lasting not longer than 15
seconds, 30-60 breaths per minute
A newly born baby still is adjusting to xtra uterine life and the lungs are just beginning to function as
a respiratory organ. The respiration of the baby at this time is characterized as usually shallow and
irregular with short periods of apnea, 30-60 breaths per minute. The apneic periods should be brief
lasting not more than 15 seconds otherwise it will be considered abnormal.
24. Answer: (A) 3-4 cm antero-posterior diameter and 2-3 cm transverse diameter, diamond
shape
The anterior fontanelle is diamond shape with the antero-posterior diameter being longer than the
transverse diameter. The posterior fontanelle is triangular shape.
25.Answer: (D) Middle third of the thigh
Neonates do not have well developed muscles of the arm. Since Vitamin K is given intramuscular,
the site must have sufficient muscles like the middle third of the thigh.
26.Answer: (A) 1-3
An APGAR of 1-3 is a sign of fetal distress which requires resuscitation. The baby is alright if the
score is 8-10.
27. Answer: (B) Acrocyanosis
Acrocyanosis is the term used to describe the babys skin color at birth when the soles and palms are
bluish but the trunk is pinkish.
28. Answer: (C) 2,500gms
According to the WHO standard, the minimum normal birth weight of a full term baby is 2,500 gms
or 2.5 Kg.

29. Answer: (B) Credes method


Credes method/prophylaxis is the procedure done to prevent ophthalmia neonatorum which the baby
can acquire as it passes through the birth canal of the mother. Usually, an ophthalmic ointment is
used.
30. Answer: (D) Almost leather-like, dry, cracked skin, negligible vernix caseosa
A post mature fetus has the appearance of an old person with dry wrinkled skin and the vernix
caseosa has already diminished.
31. Answer: (B) 1, 2, and 3
To be allowed to handle deliveries, the pregnancy must be normal and uncomplicated. And in
RA9172, the nurse is now allowed to suture perineal lacerations provided s/he has had the special
training. Also, in this law, there is no longer an explicit provision stating that the nurse still needs
special training for IV insertion.
32. Answer: (B) 0.2-0.4 degrees centigrade
The release of the hormone progesterone in the body following ovulation causes a slight elevation of
basal body temperature of about 0.2 0.4 degrees centigrade
33. Answer: (B) The mother breastfeeds exclusively and regularly during the first 6 months
without giving supplemental feedings
A mother who breastfeeds exclusively and regularly during the first 6 months benefits from lactation
amenorrhea. There is evidence to support the observation that the benefits of lactation amenorrhea
lasts for 6 months provided the woman has not had her first menstruation since delivery of the baby.
34. Answer: (D) Sperms will be barred from entering the fallopian tubes
An intrauterine device is a foreign body so that if it is inserted into the uterine cavity the initial
reaction is to produce inflammatory process and the uterus will contract in order to try to expel the
foreign body. Usually IUDs are coated with copper to serve as spermicide killing the sperms
deposited into the female reproductive tract. But the IUD does not completely fill up the uterine
cavity thus sperms which are microscopic is size can still pass through.

35. Answer: (B) Progesterone only


If mother is breastfeeding, the progesterone only type is the best because estrogen can affect
lactation.
36. Answer: (B) 26-32 days
Standard Days Method (SDM) requires that the menstrual cycles are regular between 26-32 days.
There is no need to monitor temperature or mucus secretion. This natural method of family planning
is very simple since all that the woman pays attention to is her cycle. With the aid of CycleBeads, the
woman can easily monitor her cycles.
37. Answer: (B) 1, 2, & 3
Mittelschmerz, spinnabarkeit and thin watery cervical mucus are signs of ovulation. When ovulation
occurs, the hormone progesterone is released which can cause a slight elevation of temperature
between 0.2-0.4 degrees centigrade and not 4 degrees centigrade.
38. Answer: (D) Intrauterine device (IUD)
Intrauterine device prevents pregnancy by not allowing the fertilized ovum from implanting on the
endometrium. Some IUDs have copper added to it which is spermicidal. It is not a barrier since the
sperms can readily pass through and fertilize an ovum at the fallopian tube.
39. Answer: (B) It may occur between 14-16 days before next menstruation
Not all menstrual cycles are ovulatory. Normal ovulation in a woman occurs between the 14th to the
16th day before the NEXT menstruation. A common misconception is that ovulation occurs on the
14th day of the cycle. This is a misconception because ovulation is determined NOT from the first
day of the cycle but rather 14-16 days BEFORE the next menstruation.
40. Answer: (C) 1,2,4
All of the above are essential for enhanced fertility except no. 3 because during the dry period the
woman is in her infertile period thus even when sexual contact is done, there will be no ovulation,
thus fertilization is not possible.
41.Answer: (A) Temperature, cervical mucus, cervical consistency
The 3 parameters measured/monitored which will indicate that the woman has ovulated are-

temperature increase of about 0.2-0.4 degrees centigrade, softness of the cervix and cervical mucus
that looks like the white of an egg which makes the woman feel wet.
42. Answer: (B) If the woman fails to take a pill in one day, she must take 2 pills for added
protection
If the woman fails to take her usual pill for the day, taking a double dose does not give additional
protection. What she needs to do is to continue taking the pills until the pack is consumed and use at
the time another temporary method to ensure that no pregnancy will occur. When a new pack is
started, she can already discontinue using the second temporary method she employed.
43.Answer: (B) Rubins test
Rubins test is a test to determine patency of fallopian tubes. Huhners test is also known as postcoital test to determine compatibility of the cervical mucus with sperms of the sexual partner.
44. Answer: (C) Sperm count of about 20 million per milliliter
Sperm count must be within normal in order for a male to successfully sire a child. The normal sperm
count is 20 million per milliliter of seminal fluid or 50 million per ejaculate.
45. Answer: (A) Thin watery mucus which can be stretched into a long strand about 10 cm
At the midpoint of the cycle when the estrogen level is high, the cervical mucus becomes thin and
watery to allow the sperm to easily penetrate and get to the fallopian tubes to fertilize an ovum. This
is called spinnabarkeit. And the woman feels wet. When progesterone is secreted by the ovary, the
mucus becomes thick and the woman will feel dry.
46. Answer: (D) Vas deferens
Vasectomy is a procedure wherein the vas deferens of the male is ligated and cut to prevent the
passage of the sperms from the testes to the penis during ejaculation.
47. Answer: (C) Right after the menstrual period so that the breast is not being affected by the
increase in hormones particularly estrogen
The best time to do self breast examination is right after the menstrual period is over so that the
hormonal level is low thus the breasts are not tender.

48. Answer: (B) 12 months


If a woman has not had her menstrual period for 12 consecutive months, she is considered to be in
her menopausal stage.
49. Answer: (B) Any day of the month as long it is regularly observed on the same day every
month
Menopausal women still need to do self examination of the breast regularly. Any day of the month is
alright provided that she practices it monthly on the same day that she has chosen. The hormones
estrogen and progesterone are already diminished during menopause so there is no need to consider
the time to do it in relation to the menstrual cycle.
50. Answer: (B) Clomiphene
Clomiphene or Clomid acts as an ovarian stimulant to promote ovulation. The mature ova are
retrieved and fertilized outside the fallopian tube (in-vitro fertilization) and after 48 hours the
fertilized ovum is inserted into the uterus for implantation.
1. You performed the Leopolds maneuver and found the following: breech presentation, fetal
back at the right side of the mother. Based on these findings, you can hear the fetal heart beat
(PMI) BEST in which location?
A. Left lower quadrant
B. Right lower quadrant
C. Left upper quadrant
D. Right upper quadrant
2. In Leopolds maneuver step #1, you palpated a soft broad mass that moves with the rest of
the mass. The correct interpretation of this finding is:
A. The mass palpated at the fundal part is the head part.
B. The presentation is breech.
C. The mass palpated is the back
D. The mass palpated is the buttocks.

3. In Leopolds maneuver step # 3 you palpated a hard round movable mass at the supra pubic
area. The correct interpretation is that the mass palpated is:
A. The buttocks because the presentation is breech.
B. The mass palpated is the head.
C. The mass is the fetal back.
D. The mass palpated is the fetal small part
4. The hormone responsible for a positive pregnancy test is:
A. Estrogen
B. Progesterone
C. Human Chorionic Gonadotropin
D. Follicle Stimulating hormone
5. The hormone responsible for the maturation of the graafian follicle is:
A. Follicle stimulating hormone
B. Progesterone
C. Estrogen
D. Luteinizing hormone
6. The most common normal position of the fetus in utero is:
A. Transverse position
B. Vertical position
C. Oblique position
D. None of the above
7. In the later part of the 3rd trimester, the mother may experience shortness of breath. This
complaint maybe explained as:
A. A normal occurrence in pregnancy because the fetus is using more oxygen
B. The fundus of the uterus is high pushing the diaphragm upwards

C. The woman is having allergic reaction to the pregnancy and its hormones
D. The woman maybe experiencing complication of pregnancy
8. Which of the following findings in a woman would be consistent with a pregnancy of two
months duration?
A. Weight gain of 6-10 lbs. and presence of striae gravidarum
B. Fullness of the breast and urinary frequency
C. Braxton Hicks contractions and quickening
D. Increased respiratory rate and ballottement
9. Which of the following is a positive sign of pregnancy?
A. Fetal movement felt by mother
B. Enlargement of the uterus
C. (+) pregnancy test
D. (+) ultrasound
10. What event occurring in the second trimester helps the expectant mother to accept the
pregnancy?
A. Lightening
B. Ballotment
C. Pseudocyesis
D. Quickening
11. Shoes with low, broad heels, plus a good posture will prevent which prenatal discomfort?
A. Backache
B. Vertigo
C. Leg cramps
D. Nausea

12. When a pregnant woman experiences leg cramps, the correct nursing intervention to relieve
the muscle cramps is:
A. Allow the woman to exercise
B. Let the woman walk for a while
C. Let the woman lie down and dorsiflex the foot towards the knees
D. Ask the woman to raise her legs
13. From the 33rd week of gestation till full term, a healthy mother should have prenatal check
up every:
A. week
B. 2 weeks
C. 3 weeks
D. 4 weeks
14. The expected weight gain in a normal pregnancy during the 3rd trimester is
A. 1 pound a week
B. 2 pounds a week
C. 10 lbs a month
D. 10 lbs total weight gain in the 3rd trimester
15. In the Bartholomews rule of 4, when the level of the fundus is midway between the
umbilicus and xyphoid process the estimated age of gestation (AOG) is:
A. 5th month
B. 6th month
C. 7th month
D. 8th month
16. The following are ways of determining expected date of delivery (EDD) when the LMP is
unknown EXCEPT:

A. Naegeles rule
B. Quickening
C. McDonalds rule
D. Batholomews rule of 4
17. If the LMP is Jan. 30, the expected date of delivery (EDD) is
A. Oct. 7
B. Oct. 24
C. Nov. 7
D. Nov. 8
18. Kegels exercise is done in pregnancy in order to:
A. Strengthen perineal muscles
B. Relieve backache
C. Strengthen abdominal muscles
D. Prevent leg varicosities and edema
19. Pelvic rocking is an appropriate exercise in pregnancy to relieve which discomfort?
A. Leg cramps
B. Urinary frequency
C. Orthostatic hypotension
D. Backache
20. The main reason for an expected increased need for iron in pregnancy is:
A. The mother may have physiologic anemia due to the increased need for red blood cell mass as
well as the fetal requires about 350-400 mg of iron to grow
B. The mother may suffer anemia because of poor appetite
C. The fetus has an increased need for RBC which the mother must supply
D. The mother may have a problem of digestion because of pica

21. The diet that is appropriate in normal pregnancy should be high in


A. Protein, minerals and vitamins
B. Carbohydrates and vitamins
C. Proteins, carbohydrates and fats
D. Fats and minerals
22. Which of the following signs will require a mother to seek immediate medical attention?
A. When the first fetal movement is felt
B. No fetal movement is felt on the 6th month
C. Mild uterine contraction
D. Slight dyspnea on the last month of gestation
23. You want to perform a pelvic examination on one of your pregnant clients. You prepare
your client for the procedure by:
A. Asking her to void
B. Taking her vital signs and recording the readings
C. Giving the client a perineal care
D. Doing a vaginal prep
24. When preparing the mother who is on her 4th month of pregnancy for abdominal
ultrasound, the nurse should instruct her to:
A. Observe NPO from midnight to avoid vomiting
B. Do perineal flushing properly before the procedure
C. Drink at least 2 liters of fluid 2 hours before the procedure and not void until the procedure is done
D. Void immediately before the procedure for better visualization
25. The nursing intervention to relieve morning sickness in a pregnant woman is by giving
A. Dry carbohydrate food like crackers
B. Low sodium diet

C. Intravenous infusion
D. Antacid
26. The common normal site of nidation/implantation in the uterus is
A. Upper uterine portion
B. Mid-uterine area
C. Lower uterine segment
D. Lower cervical segment
27. Mrs. Santos is on her 5th pregnancy and has a history of abortion in the 4th pregnancy and
the first pregnancy was a twin. She is considered to be
A. G 4 P 3
B. G 5 P 3
C. G 5 P 4
D. G 4 P 4
28. The following are skin changes in pregnancy EXCEPT:
A. Chloasma
B. Striae gravidarum
C. Linea negra
D. Chadwicks sign
29. Which of the following statements is TRUE of conception?
A. Within 2-4 hours after intercourse conception is possible in a fertile woman
B. Generally, fertilization is possible 4 days after ovulation
C. Conception is possible during menstruation in a long menstrual cycle
D. To avoid conception, intercourse must be avoided 5 days before and 3 days after menstruation
30. Which of the following are the functions of amniotic fluid?

1. Cushions the fetus from abdominal trauma


2. Serves as the fluid for the fetus
3. Maintains the internal temperature
4. Facilitates fetal movement
A. 1 & 3
B. 1, 3, 4
C. 1, 2, 3
D. All of the above
31. You are performing abdominal exam on a 9th month pregnant woman. While lying supine,
she felt breathless, had pallor, tachycardia, and cold clammy skin. The correct assessment of
the womans condition is that she is:
A. Experiencing the beginning of labor
B. Having supine hypotension
C. Having sudden elevation of BP
D. Going into shock
32. Smoking is contraindicated in pregnancy because
A. Nicotine causes vasodilation of the mothers blood vessels
B. Carbon monoxide binds with the hemoglobin of the mother reducing available hemoglobin for the
fetus
C. The smoke will make the fetus and the mother feel dizzy
D. Nicotine will cause vasoconstriction of the fetal blood vessels
33. Which of the following is the most likely effect on the fetus if the woman is severely anemic
during pregnancy?
A. Large for gestational age (LGA) fetus
B. Hemorrhage
C. Small for gestational age (SGA) baby
D. Erythroblastosis fetalis

34. Which of the following signs and symptoms will most likely make the nurse suspect that the
patient is having hydatidiform mole?
A. Slight bleeding
B. Passage of clear vesicular mass per vagina
C. Absence of fetal heart beat
D. Enlargement of the uterus
35. Upon assessment the nurse found the following: fundus at 2 fingerbreadths above the
umbilicus, last menstrual period (LMP) 5 months ago, fetal heart beat (FHB) not appreciated.
Which of the following is the most possible diagnosis of this condition?
A. Hydatidiform mole
B. Missed abortion
C. Pelvic inflammatory disease
D. Ectopic pregnancy
36. When a pregnant woman goes into a convulsive seizure, the MOST immediate action of the
nurse to ensure safety of the patient is:
A. Apply restraint so that the patient will not fall out of bed
B. Put a mouth gag so that the patient will not bite her tongue and the tongue will not fall back
C. Position the mother on her side to allow the secretions to drain from her mouth and prevent
aspiration
D. Check if the woman is also having a precipitate labor
37. A gravidocardiac mother is advised to observe bed rest primarily to
A. Allow the fetus to achieve normal intrauterine growth
B. Minimize oxygen consumption which can aggravate the condition of the compromised heart of the
mother
C. Prevent perinatal infection
D. Reduce incidence of premature labor

38. A pregnant mother is admitted to the hospital with the chief complaint of profuse vaginal
bleeding, AOG 36 wks, not in labor. The nurse must always consider which of the following
precautions:
A. The internal exam is done only at the delivery under strict asepsis with a double set-up
B. The preferred manner of delivering the baby is vaginal
C. An emergency delivery set for vaginal delivery must be made ready before examining the patient
D. Internal exam must be done following routine procedure
39. Which of the following signs will distinguish threatened abortion from imminent abortion?
A. Severity of bleeding
B. Dilation of the cervix
C. Nature and location of pain
D. Presence of uterine contraction
40. The nursing measure to relieve fetal distress due to maternal supine hypotension is:
A. Place the mother on semi-fowlers position
B. Put the mother on left side lying position
C. Place mother on a knee chest position
D. Any of the above
41. To prevent preterm labor from progressing, drugs are usually prescribed to halt the labor.
The drugs commonly given are:
A. Magnesium sulfate and terbutaline
B. Prostaglandin and oxytocin
C. Progesterone and estrogen
D. Dexamethasone and prostaglandin
42. In placenta praevia marginalis, the placenta is found at the:

A. Internal cervical os partly covering the opening


B. External cervical os slightly covering the opening
C. Lower segment of the uterus with the edges near the internal cervical os
D. Lower portion of the uterus completely covering the cervix
43. In which of the following conditions can the causative agent pass through the placenta and
affect the fetus in utero?
A. Gonorrhea
B. Rubella
C. Candidiasis
D. moniliasis
44. Which of the following can lead to infertility in adult males?
A. German measles
B. Orchitis
C. Chicken pox
D. Rubella
45. Papanicolaou smear is usually done to determine cancer of
A. Cervix
B. Ovaries
C. Fallopian tubes
D. Breast
46. Which of the following causes of infertility in the female is primarily psychological in
origin?
A. Vaginismus
B. Dyspareunia
C. Endometriosis
D. Impotence

47. Before giving a repeat dose of magnesium sulfate to a pre-eclamptic patient, the nurse
should assess the patients condition. Which of the following conditions will require the nurse to
temporarily suspend a repeat dose of magnesium sulfate?
A. 100 cC. urine output in 4 hours
B. Knee jerk reflex is (+)2
C. Serum magnesium level is 10mEg/L.
D. Respiratory rate of 16/min
48. Which of the following is TRUE in Rh incompatibility?
A. The condition can occur if the mother is Rh(+) and the fetus is Rh(-)
B. Every pregnancy of an Rh(-) mother will result to erythroblastosis fetalis
C. On the first pregnancy of the Rh(-) mother, the fetus will not be affected
D. RhoGam is given only during the first pregnancy to prevent incompatibility
49. Which of the following are the most commonly assessed findings in cystitis?
A. Frequency, urgency, dehydration, nausea, chills, and flank pain
B. Nocturia, frequency, urgency dysuria, hematuria, fever and suprapubic pain
C. Dehydration, hypertension, dysuria, suprapubic pain, chills, and fever
D. High fever, chills, flank pain nausea, vomiting, dysuria, and frequency
50. Which of the following best reflects the frequency of reported postpartum blues?
A. Between 10% and 40% of all new mothers report some form of postpartum blues
B. Between 30% and 50% of all new mothers report some form of postpartum blues
C. Between 50% and 80% of all new mothers report some form of postpartum blues
D. Between 25% and 70% of all new mothers report some form of postpartum blues

Answers and Rationale

Here are the answers and rationale for this exam. Counter check your answers to those below and tell
us your scores. If you have any disputes or need more clarification to a certain question, please direct
them to the comments section.
1. Answer: (B) Right lower quadrant
Right lower quadrant. The landmark to look for when looking for PMI is the location of the fetal
back in relation to the right or left side of the mother and the presentation, whether cephalic or
breech. The best site is the fetal back nearest the head.
2. Answer: (D) The mass palpated is the buttocks.
The palpated mass is the fetal buttocks since it is broad and soft and moves with the rest of the mass.
3. Answer: (B) The mass palpated is the head.
When the mass palpated is hard round and movable, it is the fetal head.
4. Answer: (C) Human Chorionic Gonadotropin
Human chorionic gonadotropin (HCG) is the hormone secreted by the chorionic villi which is the
precursor of the placenta. In the early stage of pregnancy, while the placenta is not yet fully
developed, the major hormone that sustains the pregnancy is HCG.
5. Answer: (A) Follicle stimulating hormone
The hormone that stimulates the maturation if the of the graafian follicle is the Follicle Stimulating
Hormone which is released by the anterior pituitary gland.
6. Answer: (B) Vertical position
Vertical position means the fetal spine is parallel to the maternal spine thus making it easy for the
fetus to go out the birth canal. If transverse or oblique, the fetus cant be delivered normally per
vagina.
7. Answer: (B) The fundus of the uterus is high pushing the diaphragm upwards
From the 32nd week of the pregnancy, the fundus of the enlarged uterus is pushing the respiratory
diaphragm upwards. Thus, the lungs have reduced space for expansion consequently reducing the
oxygen supply.

8. Answer: (B) Fullness of the breast and urinary frequency


Fullness of the breast is due to the increased amount of progesterone in pregnancy. The urinary
frequency is caused by the compression of the urinary bladder by the gravid uterus which is still
within the pelvic cavity during the first trimester.
9. Answer: (D) (+) ultrasound
A positive ultrasound will definitely confirm that a woman is pregnant since the fetus in utero is
directly visualized.
10. Answer: (D) Quickening
Quickening is the first fetal movement felt by the mother makes the woman realize that she is truly
pregnant. In early pregnancy, the fetus is moving but too weak to be felt by the mother. In the 18th20th week of gestation, the fetal movements become stronger thus the mother already feels the
movements.
11. Answer: (A) Backache
Backache usually occurs in the lumbar area and becomes more problematic as the uterus enlarges.
The pregnant woman in her third trimester usually assumes a lordotic posture to maintain balance
causing an exaggeration of the lumbar curvature. Low broad heels provide the pregnant woman with
a good support.
12. Answer: (C) Let the woman lie down and dorsiflex the foot towards the knees
Leg cramps is caused by the contraction of the gastrocnimeus (leg muscle). Thus, the intervention is
to stretch the muscle by dosiflexing the foot of the affected leg towards the knee.
13. Answer: (A) week
In the 9th month of pregnancy the mother needs to have a weekly visit to the prenatal clinic to
monitor fetal condition and to ensure that she is adequately prepared for the impending labor and
delivery.
14. Answer: (A) 1 pound a week
During the 3rd trimester the fetus is gaining more subcutaneous fat and is growing fast in preparation
for extra uterine life. Thus, one pound a week is expected.

15. Answer: (C) 7th month


In Bartholomews Rule of 4, the landmarks used are the symphysis pubis, umbilicus and xyphoid
process. At the level of the umbilicus, the AOG is approximately 5 months and at the level of the
xyphoid process 9 months. Thus, midway between these two landmarks would be considered as 7
months AOG.
16. Answer: (A) Naegeles rule
Naegeles Rule is determined based on the last menstrual period of the woman.
17. Answer: (C) Nov. 7
Based on the last menstrual period, the expected date of delivery is Nov. 7. The formula for the
Naegeles Rule is subtract 3 from the month and add 7 to the day.
18. Answer: (A) Strengthen perineal muscles
Kegels exercise is done by contracting and relaxing the muscles surrounding the vagina and anus in
order to strengthen the perineal muscles
19.Answer: (D) Backache
Backache is caused by the stretching of the muscles of the lower back because of the pregnancy.
Pelvic rocking is good to relieve backache.
20. Answer: (A) The mother may have physiologic anemia due to the increased need for red
blood cell mass as well as the fetal requires about 350-400 mg of iron to grow
About 400 mgs of Iron is needed by the mother in order to produce more RBC mass to be able to
provide the needed increase in blood supply for the fetus. Also, about 350-400 mgs of iron is need for
the normal growth of the fetus. Thus, about 750-800 mgs iron supplementation is needed by the
mother to meet this additional requirement.
21. Answer: (A) Protein, minerals and vitamins
In normal pregnancy there is a higher demand for protein (body building foods), vitamins (esp.
vitamin A, B, C, folic acid) and minerals (esp. iron, calcium, phosphorous, zinc, iodine, magnesium)
because of the need of the growing fetus.

22. Answer: (B) No fetal movement is felt on the 6th month


Fetal movement is usually felt by the mother during 4.5 5 months. If the pregnancy is already in its
6th month and no fetal movement is felt, the pregnancy is not normal either the fetus is already dead
intra-uterine or it is an H-mole.
23. Answer: (A) Asking her to void
A pelvic examination includes abdominal palpation. If the pregnant woman has a full bladder, the
manipulation may cause discomfort and accidental urination because of the pressure applied during
the abdominal palpation. Also, a full bladder can impede the accuracy of the examination because the
bladder (which is located in front of the uterus) can block the uterus.
24. Answer: (C) Drink at least 2 liters of fluid 2 hours before the procedure and not void until
the procedure is done
Drinking at least 2 liters of water 2 hours before the procedure will result to a distended bladder. A
full bladder is needed when doing an abdominal ultrasound to serve as a window for the ultrasonic
sound waves to pass through and allow visualization of the uterus (located behind the urinary
bladder).
25. Answer: (A) Dry carbohydrate food like crackers
Morning sickness maybe caused by hypoglycemia early in the morning thus giving carbohydrate
food will help.
26. Answer: (A) Upper uterine portion
The embryos normal nidation site is the upper portion of the uterus. If the implantation is in the
lower segment, this is an abnormal condition called placenta previa.
27. Answer: (B) G 5 P 3
Gravida refers to the total number of pregnancies including the current one. Para refers to the number
of pregnancies that have reached viability. Thus, if the woman has had one abortion, she would be
considered Para 3. Twin pregnancy is counted only as 1.
28. Answer: (D) Chadwicks sign
Chadwicks sign is bluish discoloration of the vaginal mucosa as a result of the increased
vascularization in the area.

29.Answer: (A) Within 2-4 hours after intercourse conception is possible in a fertile woman
The sperms when deposited near the cervical os will be able to reach the fallopian tubes within 4
hours. If the woman has just ovulated (within 24hours after the rupture of the graafian follicle),
fertilization is possible.
30. Answer: (D) All of the above
All the four functions enumerated are true of amniotic fluid.
31. Answer: (B) Having supine hypotension
Supine hypotension is characterized by breathlessness, pallor, tachycardia and cold clammy skin.
This is due to the compression of the abdominal aorta by the gravid uterus when the woman is on a
supine position.
32. Answer: (B) Carbon monoxide binds with the hemoglobin of the mother reducing available
hemoglobin for the fetus
Carbon monoxide is one of the substances found in cigarette smoke. This substance diminishes the
ability of the hemoglobin to bind with oxygen thus reducing the amount of oxygenated blood
reaching the fetus.
33. Answer: (C) Small for gestational age (SGA) baby
Anemia is a condition where there is a reduced amount of hemoglobin. Hemoglobin is needed to
supply the fetus with adequate oxygen. Oxygen is needed for normal growth and development of the
fetus.
34. Answer: (B) Passage of clear vesicular mass per vagina
Hydatidiform mole (H-mole) is characterized by the degeneration of the chorionic villi wherein the
villi becomes vesicle-like. These vesicle-like substances when expelled per vagina and is a definite
sign that the woman has H-mole.
35. Answer: (A) Hydatidiform mole
Hydatidiform mole begins as a pregnancy but early in the development of the embryo degeneration
occurs. The proliferation of the vesicle-like substances is rapid causing the uterus to enlarge bigger
than the expected size based on ages of gestation (AOG). In the situation given, the pregnancy is only
5 months but the size of the uterus is already above the umbilicus which is compatible with 7 months

AOG. Also, no fetal heart beat is appreciated because the pregnancy degenerated thus there is no
appreciable fetal heart beat.
36. Answer: (C) Position the mother on her side to allow the secretions to drain from her mouth
and prevent aspiration
Positioning the mother on her side will allow the secretions that may accumulate in her mouth to
drain by gravity thus preventing aspiration pneumonia. Putting a mouth gag is not safe since during
the convulsive seizure the jaw will immediately lock. The mother may go into labor also during the
seizure but the immediate concern of the nurse is the safety of the baby. After the seizure, check the
perineum for signs of precipitate labor.
37. Answer: (B) Minimize oxygen consumption which can aggravate the condition of the
compromised heart of the mother
Activity of the mother will require more oxygen consumption. Since the heart of a gravido-cardiac is
compromised, there is a need to put a mother on bedrest to reduce the need for oxygen.
38. Answer: (A) The internal exam is done only at the delivery under strict asepsis with a
double set-up
Painless vaginal bleeding during the third trimester maybe a sign of placenta praevia. If internal
examination is done in this kind of condition, this can lead to even more bleeding and may require
immediate delivery of the baby by cesarean section. If the bleeding is due to soft tissue injury in the
birth canal, immediate vaginal delivery may still be possible so the set up for vaginal delivery will be
used. A double set-up means there is a set up for cesarean section and a set-up for vaginal delivery to
accommodate immediately the necessary type of delivery needed. In both cases, strict asepsis must
be observed.
39. Answer: (B) Dilation of the cervix
In imminent abortion, the pregnancy will definitely be terminated because the cervix is already open
unlike in threatened abortion where the cervix is still closed.
40. Answer: (B) Put the mother on left side lying position
When a pregnant woman lies on supine position, the weight of the gravid uterus would be
compressing on the vena cava against the vertebrae obstructing blood flow from the lower

extremities. This causes a decrease in blood return to the heart and consequently immediate
decreased cardiac output and hypotension. Hence, putting the mother on side lying will relieve the
pressure exerted by the gravid uterus on the vena cava.
41. Answer: (A) Magnesium sulfate and terbutaline
Magnesium sulfate acts as a CNS depressant as well as a smooth muscle relaxant. Terbutaline is a
drug that inhibits the uterine smooth muscles from contracting. On the other hand, oxytocin and
prostaglandin stimulates contraction of smooth muscles.
42. Answer: (C) Lower segment of the uterus with the edges near the internal cervical os
Placenta marginalis is a type of placenta previa wherein the placenta is implanted at the lower
segment of the uterus thus the edges of the placenta are touching the internal cervical opening/os.
The normal site of placental implantation is the upper portion of the uterus.
43. Answer: (B) Rubella
Rubella is caused by a virus and viruses have low molecular weight thus can pass through the
placental barrier. Gonorrhea, candidiasis and moniliasis are conditions that can affect the fetus as it
passes through the vaginal canal during the delivery process.
44. Answer: (B) Orchitis
Orchitis is a complication that may accompany mumps in adult males. This condition is characterized
by unilateral inflammation of one of the testes which can lead to atrophy of the affected testis. About
20-30% of males who gets mumps after puberty may develop this complication.
45. Answer: (A) Cervix
Papanicolaou (Paps) smear is done to detect cervical cancer. It cant detect cancer in ovaries and
fallopian tubes because these organs are outside of the uterus and the abnormal cells from these
organs will not be detected from a smear done on the cervix.
46. Answer: (A) Vaginismus
Vaginismus is primarily psychological in origin. Endometriosis is a condition that is caused by
organic abnormalities. Dyspareunia is usually caused by infection, endometriosis or hormonal
changes in menopause although may sometimes be psychological in origin.

47. Answer: (A) 100 cc. urine output in 4 hours


The minimum urine output expected for a repeat dose of MgSO4 is 30 cc/hr. If in 4 hours the urine
output is only 100 cc this is low and can lead to poor excretion of Magnesium with a possible
cumulative effect, which can be dangerous to the mother.
48. Answer: (C) On the first pregnancy of the Rh(-) mother, the fetus will not be affected
On the first pregnancy, the mother still has no contact with Rh(+) blood thus it has not antibodies
against Rh(+). After the first pregnancy, even if terminated into an abortion, there is already the
possibility of mixing of maternal and fetal blood so this can trigger the maternal blood to produce
antibodies against Rh(+) blood. The fetus takes its blood type usually form the father.
49. Answer: (B) Manifestations of cystitis include, frequency, urgency, dysuria, hematuria
nocturia, fever, and suprapubic pain.
Dehydration, hypertension, and chills are not typically associated with cystitis. High fever chills,
flank pain, nausea, vomiting, dysuria, and frequency are associated with pvelonephritis.
50. Answer: (C) According to statistical reports, between 50% and 80% of all new mothers
report some form of postpartum blues. The ranges of 10% to 40%, 30% to 50%, and 25% to 70%
are incorrect.

You might also like